199
ACCOUNTING STANDARDS Accounting Standards are the defined accounting policies issued by Government or expert institute. These standards are issued to bring harmonization in follow up of accounting policies. Presently, Institute of Chartered Accountants of India has issued 29 Accounting Standards as listed below. AS 1. Disclosure of Accounting Policies AS 2. Valuation of Inventories AS 3. Cash Flow Statements AS 4. Contingencies and Events Occurring After the Balance Sheet Date AS 5. Net Profit or Loss for the Period, Prior Period Items and Changes in Accounting Policies AS 6. Depreciation Accounting AS 7. Construction Contracts AS 8. Accounting for Research and Development (Not Applicable now) AS 9. Revenue Recognition AS 10. Accounting for Fixed Assets AS 11. Accounting for the Effects of Changes in Foreign Exchange Rates AS 12. Accounting for Government Grants AS 13. Accounting for Investments AS 14. Accounting for Amalgamation AS 15. Accounting for Retirement Benefits in the financial Statements of Employers AS 16. Borrowing Costs AS 17. Segment Reporting AS 18. Related Party Disclosure AS 19. Leases AS 20. Earning Per Share AS 21. Consolidated Financial Statements AS 22. Accounting for Taxes on Income AS 23. Accounting for Investments in Associates in Consolidated Financial Statements AS 24. Discontinuing Operations AS 25. Interim Financial Reporting AS 26. Intangible Assets AS 27. Financial Reporting of Interests in Joint Ventures AS 28. Impairment of Assets AS 29. Provisions, Contingent Liabilities & Contingent Assets Procedure for Issuing Accounting Standards 1. Accounting Standard Board (ASB) determines the broad areas in which Accounting Standards need to be formulated. 2. In the preparation of AS, ASB is assisted by Study Groups. 3. ASB also holds discussions with representative of Government, Public Sector Undertakings, Industry and other organizations (ICSI/ICWAI) for ascertaining their views. 4. An exposure draft of the proposed standard is prepared and issued for comments by members of ICAI and the public at large. 5. After taking into consideration the comments received, the draft of the proposed standard will be finalized by ASB and submitted to the council of the Institute. 6. The council of the Institute will consider the final draft of the proposed Standard and If found necessary, modify the same in consultation with ASB. The AS on the relevant subject will then be issued under the authority of the council.

cacwacs.files.wordpress.com · ACCOUNTING STANDARDS Accounting Standards are the defined accounting policies issued by Government or expert institute. These standards are issued to

  • Upload
    others

  • View
    10

  • Download
    0

Embed Size (px)

Citation preview

ACCOUNTING STANDARDS

Accounting Standards are the defined accounting policies issued by Government or expert institute. These

standards are issued to bring harmonization in follow up of accounting policies.

Presently, Institute of Chartered Accountants of India has issued 29 Accounting Standards as listed below.

AS 1. Disclosure of Accounting Policies

AS 2. Valuation of Inventories

AS 3. Cash Flow Statements

AS 4. Contingencies and Events Occurring After the Balance Sheet Date

AS 5. Net Profit or Loss for the Period, Prior Period Items and Changes in Accounting Policies

AS 6. Depreciation Accounting

AS 7. Construction Contracts

AS 8. Accounting for Research and Development (Not Applicable now)

AS 9. Revenue Recognition

AS 10. Accounting for Fixed Assets

AS 11. Accounting for the Effects of Changes in Foreign Exchange Rates

AS 12. Accounting for Government Grants

AS 13. Accounting for Investments

AS 14. Accounting for Amalgamation

AS 15. Accounting for Retirement Benefits in the financial Statements of Employers

AS 16. Borrowing Costs

AS 17. Segment Reporting

AS 18. Related Party Disclosure

AS 19. Leases

AS 20. Earning Per Share

AS 21. Consolidated Financial Statements

AS 22. Accounting for Taxes on Income

AS 23. Accounting for Investments in Associates in Consolidated Financial Statements

AS 24. Discontinuing Operations

AS 25. Interim Financial Reporting

AS 26. Intangible Assets

AS 27. Financial Reporting of Interests in Joint Ventures

AS 28. Impairment of Assets

AS 29. Provisions, Contingent Liabilities & Contingent Assets

Procedure for Issuing Accounting Standards

1. Accounting Standard Board (ASB) determines the broad areas in which Accounting Standards

need to be formulated.

2. In the preparation of AS, ASB is assisted by Study Groups.

3. ASB also holds discussions with representative of Government, Public Sector Undertakings,

Industry and other organizations (ICSI/ICWAI) for ascertaining their views.

4. An exposure draft of the proposed standard is prepared and issued for comments by members of

ICAI and the public at large.

5. After taking into consideration the comments received, the draft of the proposed standard will be

finalized by ASB and submitted to the council of the Institute.

6. The council of the Institute will consider the final draft of the proposed Standard and If found

necessary, modify the same in consultation with ASB. The AS on the relevant subject will then be

issued under the authority of the council.

AS – 1

DISCLOSURE OF ACOUNTING POLICY

Accounting policies are the specific accounting principles and the methods of applying those principles

adopted by an enterprise in the preparation and presentation of financial statements.

- All significant accounting policies should be disclosed.

- Such disclosure form part of financial statements.

- All disclosures should be made at one place.

- Specific disclosure for the adoption of fundamental accounting assumptions is not

required.

- Disclosure of accounting policies cannot remedy a wrong or inappropriate

treatment of the item in the accounts.

Any change in accounting policies which has a material effect in the current period or which is reasonably

expected to have material effect in later periods should be disclosed.

In the case of a change in accounting policies, which has a material effect in the current period, the amount by

which any item in the financial statements is affected by such change should also be disclosed to the extent

ascertainable. Where such amount is not ascertainable, the fact should be indicated.

Fundamental Accounting Assumption: (GCA) :

1] Going Concern

2] Consistency

3] Accrual

Major considerations governing the selection of accounting policies:

1] Prudence

2] Substance over form (Logic over Law)

3] Materiality

The following are examples of the areas in which different accounting policies may be adopted by different

enterprises:

- Methods of depreciation

- Methods of translation of foreign currency

- Valuation of inventories

- Valuation of investments

- Treatment of retirement benefits

- Treatment of contingent liabilities etc.

AS – 2

VALUATION OF INVENTORY

Inventories are assets:

(a) held for sale in ordinary course of business;

(b) in the process of production fro such sale (WIP);

(c) in the form of materials or supplies to be consumed in the production process or in the rendering

of services.

However, this standard does not apply to the valuation of following inventories:

(a) WIP arising under construction contract (Refer AS – 7);

(b) WIP arising in the ordinary course of business of service providers;

(c) Shares, debentures and other financial instruments held as stock in trade; and

(d) Producers‟ inventories of livestock, agricultural and forest products, and mineral oils, ores and

gases to the extent that they are measured at net realizable value in accordance with well

established practices in those industries.

Inventories should be valued at the lower of cost and net realizable value.

The cost of inventories should comprise

(a) all costs of purchase

(b) costs of conversion

(c) other costs incurred in bringing the inventories to their present location and condition.

The costs of purchase consist of

(a) the purchase price

(b) duties and taxes ( other than those subsequently recoverable by the enterprise from the taxing

authorities like CENVAT credit)

(c) freight inwards and other expenditure directly attributable to the acquisition.

Trade discounts (but not cash discounts), rebates, duty drawbacks and other similar items are

deducted in determining the costs of purchase.

The costs of conversion include direct costs and systematic allocation of fixed and variable

production overhead.

Allocation of fixed overheads is based on the normal capacity of the production facilities. Normal

capacity is the production, expected to be achieved on an average over a number of periods or

seasons under normal circumstances, taking into account the loss of capacity resulting from planned

maintenance.

Under Recovery: Unallocated overheads are recognized as an expense in the period in which they

are incurred.

Example: Normal capacity = 20000 units

Production = 18000 units

Sales = 16000 units

Closing Stock = 2000 units

Fixed Overheads = Rs. 60000

Then, Recovery rate = Rs60000/20000 = Rs 3 per unit

Fixed Overheads will be bifurcated into three parts:

Cost of sales : 16000*3 = 48000

Closing stock : 2000 *3 = 6000

Under recovery : Rs 6000 ( to be charged to P/L)

(Apparently it seems that fixed cost element in closing stock should be

60000/18000*2000 =Rs 6666.67. but this is wrong as per AS-2)

Over Recovery: In period of high production, the amount of fixed production overheads is

allocated to each unit of production is decreased so that inventories.

Example: Normal capacity = 20000 units

Production = 25000 units

Sales = 23000 units

Closing Stock = 2000 units

Fixed Overheads = Rs 60000

Recovery Rate = Rs 60000/20000 = Rs 3 per unit

But, Revised Recovery rate = Rs 60000/25000 = Rs. 2.40 per unit

Cost of sales : 23000*2.4 = Rs 55200

Closing Stock : 2000 *2.4 = Rs. 4800

Joint or by products:

In case of joint or by products, the costs incurred up to the stage of split off should be allocated on

a rational and consistent basis. The basis of allocation may be sale value at split off point or sale

value at the completion of production. In case of the by products of negligible value or wastes,

valuation may be taken at net realizable value. The cost of main product is then joint cost minus

net realizable value of by product or waste.

The other costs are also included in the cost of inventory to the extent they contribute in bringing

the inventory to its present location and condition.

Interest and other borrowing costs are usually not included in cost of inventory. However, AS-16

recommends the areas where borrowing costs are taken as cost of inventory.

Certain costs are strictly not taken as cost of inventory.

(a) Abnormal amounts of wasted materials, labour, or other production costs;

(b) Storage costs, unless those costs are necessary in the production process prior to a further

production stage;

(c) Administrative overheads that do not contribute to bringing the inventories to their present

location and condition; and

(d) Selling and Distribution costs.

Cost Formula:

Specific identification method for determining cost of inventories

Specific identification method means directly linking the cost with specific item of

inventories. This method has application in following conditions:

In case of purchase of item specifically segregated for specific project and is

not ordinarily interchangeable.

In case of goods of services produced and segregated for specific project.

Where Specific Identification method is not applicable

The cost of inventories is valued by the following methods;

FIFO ( First In First Out) Method

Weighted Average Cost

Cost of inventories in certain conditions:

The following methods may be used for convenience if the results approximate actual cost.

Standard Cost: It takes into account normal level of consumption of material and

supplies, labour, efficiency and capacity utilization. It must be regularly reviewed taking

into consideration the current condition.

Retail Method: Normally applicable for retail trade

Cost of inventory is determined by reducing the gross margin from the

sale

value of inventory.

Net Realisable Value means the estimated selling price in ordinary course of business, at the time of

valuation, less estimated cost of completion and estimated cost necessary to make the sale.

Comparison between net realizable value and cost of inventory

The comparison between cost and net realizable value should be made on item-by-item basis. (In

some cases, group of items-by-group of item basis)

For Example:

Cost NRV Inventory Value as per AS-2

Item A 100 90 90

Item B 100 115 100

Total 200 205 200 190

Raw material valuation

If the finished goods to which raw material is applied, is sold at profit, RAW MATERIAL is valued at

cost irrespective of its NRV level being lower to its costs.

AS – 3

CASH FLOW STATEMENT

Definitions:

Cash comprises cash on hand and cash at bank. (Demand Deposits with bank)

Cash Equivalents are

Short Term

Highly Liquid Investments (Maturity around 3 months)

Subject to insignificant risk of changes in value.

Cash Flows are inflows and outflows of cash and cash equivalents.

Cash Flow Statement represents the cash flows during the specified period by operating, investing

and financing activities.

Operating Activities are the principal revenue-producing activities of the enterprise and other

activities that are not investing activities and financing activities.

Example:

1] Cash receipts from sales of goods/services

2] Cash receipts from royalties, fees and other revenue items

3] Cash payments for salaries, wages and rent

4] Cash payment to suppliers for goods

5] Cash payments or refunds of Income Tax unless they can be specifically identified with financing

or investing activities

6] Cash receipts and payments to future contracts, forward contracts when the contracts are held for

trading purposes.

Cash from operating activities can be disclosed either by DIRECT METHOD OR BY INDIRECT

METHOD.

Investing Activities are the acquisition and disposal of long-term assets and other investments not

included in cash equivalents.

Example:

1] Cash payments/receipts to acquire/sale of fixed assets including intangible assets

2] Cash payments to acquire shares or interest in joint ventures (other than the cases where

instruments are considered as cash equivalents)

3] Cash advances and loans made to third parties (Loan sanctioned by a financial enterprise is

operating activity)

4] Dividends and Interest received

5] Cash flows from acquisitions and disposal of subsidiaries

Financing Activities are activities that result in changes in the size and composition of the owners‟

capital (including preference share capital in the case of a company) and borrowing of the

enterprise.

Example:

1] Cash proceeds from issue of shares and debentures

2] Buy back of shares

3] Redemption of Preference shares or debentures

4] Cash repayments of amount borrowed.

5] Dividend and Interest paid

An enterprise should report separately major classes of gross cash receipts and gross cash payments

arising from investing and financing activities.

However, cash flows from following activities may be reported on a net basis.

Cash receipts and payments on behalf of customers

For example: Cash collected on behalf of, and paid over to, the owners of properties.

Cash flows from items in which turnover is quick, the amounts are large and the maturities

are short.

For example: Purchase and sale of investments

For financial enterprise: Cash receipts and payments for the acceptance and repayment of

deposits with a fixed maturity date.

For financial enterprise: Deposits placed/withdrawn from other financial enterprises

For financial enterprise: Cash advances and loans made to customers and the repayment of

those advances and loans.

Foreign Currency Cash Flows:

Cash flows arising in foreign currency should be recorded in enterprise‟ reporting currency

applying the exchange conversion rate existing on the date of cash flow.

The effect of changes in exchange rates of cash and cash equivalents held in foreign currency should

be reported as separate part of the reconciliation of the changes in cash and cash equivalents during

the period.

Extraordinary Items: These items should be separately shown under respective heads of cash from

operating, investing and financing activities.

Investing and financing transactions that do not require the use of cash and cash equivalents should

be excluded from a cash flow statement. For Example

A] The conversion of debt to equity

B] Acquisition of an enterprise by means of issue of shares

Other Disclosure:

Components of cash and cash equivalents.

Reconciliation of closing cash and cash equivalents with items of balance sheet.

The amount of significant cash and cash equivalent balances held by the enterprise, which

are not available for use by it.

AS - 4

CONTINGENCIES AND EVENTS OCCURRING AFTER THE BALANCE SHEET DATE

Contingency : A contingency is a condition or situation, the ultimate outcome of which, gain or loss, will be

known or determined only on the occurrence, or non-occurrence, of one or more uncertain

future events.

Accounting Treatment:

If it is likely that a contingency will result in

LOSS: It is prudent to provide for that loss in the financial statements.

PROFIT: Not recognized as revenue (However, when the realization of a gain is virtually certain, then such

gain is not a contingency and accounting for the gain is appropriate.)

The estimates of the outcome and of the financial effect of contingencies are determined

- by the judgement of the management

- by review of events occurring after the balance sheet date

- by experience of the enterprise in similar transaction

- by reviewing reports from independent experts.

If estimation cannot be made, disclosure is made of the existence and nature of the contingency.

Provision for contingencies are not made in respect of general or unspecified risks.

The existence and amount of guarantees and obligations arising from discounted bills of exchange are

generally disclosed by way of note even though the possibility of loss is remote.

The amount of a contingent loss should be provided for by a charge in the statement of profit and loss if:

(a) it is probable that future events will confirm that, after taking into account any related probable

recovery, an asset has been impaired or a liability has been incurred as at the balance sheet date, and

(b) a reasonable estimate of the amount of the resulting loss can be made.

If either of aforesaid two conditions are not met, e.g where a reasonable estimate of the loss is not practicable,

the existence of the contingency should be disclosed by way of note unless the possibility of loss is

remote.Such disclosure should provide following information:

(a) the nature of the contingency;

(b) the uncertainities which may affect the future outcome;

(c) an estimate of the financial effect, or a statement that such an estimate cannot be made.

Events Occurring after the Balance Sheet Date:

Events occurring after the balance sheet date are those significant events, both favourable and unfavourable,

that occur between the balance sheet date and the date on which the financial statements are approved by the

Board of Directors in case of a company, and, by the corresponding approving authority in the case of any

other entity.

Two types of events can be identified:

Adjusting Event:

Those, which provide further evidence of conditions that, existed at the balance sheet date

Actual adjustments in financial statements are required for adjusting event.

Exceptions:

1] Although, not adjusting event, Proposed dividend are adjusted in books of account.

2] Adjustments are required for the events, which occur after balance sheet date that indicates that

fundamental accounting assumption of going concern is no longer, appropriate.

Non-Adjusting Events:

Those, which are indicative of conditions that arose subsequent to the balance sheet date.

No adjustments are required to be made for such events. But, disclosures should be made in the report of the

approving authority of those events occurring after the balance sheet date that represent material changes

and commitments affecting the financial position of the enterprise. Such disclosure should provide following

information:

(a) the nature of the events

(b) an estimate of the financial effect, or a statement that such an estimate cannot be made.

AS-5

NET PROFIT OR LOSS FOR THE PERIOD, PRIOR PERIOD ITEMS AND CHANGES IN ACCOUNTING

POLICIES

All items of income and expense, which are recognized in a period, should be included in the determination of

net profit or loss for the period unless an Accounting Standard requires or permits otherwise.

The net profit or loss for the period comprises the following components, each of which should be disclosed on

the face of the statement of profit and loss:

(a) profit or loss from ordinary activities; and

(b) extraordinary items.

Ordinary Activities are any activities, which are undertaken by an enterprise as part of its business, and such

related activities in which the enterprise engages in furtherance of, incidental to, or arising from, these

activities.

When items of income and expenses within profit or loss from ordinary activities are of such size, nature that

their disclosure is relevant to explain the performance of the enterprise for the period, the nature and amount

of such items should be disclosed properly. Examples of such circumstances are:

(Exceptional Items)

- disposal of items of fixed assets

- litigation settlements

- legislative changes having retrospective application

- disposal of long term investments

- reversal of provisions

Extraordinary items are income or expense that arise from events or transactions that are clearly distinct

from the ordinary activities of the enterprise and, therefore, are not expected to recur frequently or regularly.

Examples of events or transactions that generally give rise to extraordinary items for most enterprises are:

- attachment of property of the enterprise;

- an earthquake

However, claims from policyholders arising from an earthquake do not qualify as an extraordinary item for

an insurance enterprise that insures against such risks.

Extraordinary items should be disclosed in the statement of profit and loss as a part of net profit or loss for

the period. The nature and the amount of each extraordinary item should be separately disclosed in the

statement of profit and loss in a manner that its impact on current profit or loss can be perceived.

Prior Period Items:

Prior period items are income or expenses that arise in the current period as a result of ERROR or

OMMISSIONS in the preparation of the financial statements of one or more prior periods.

The nature and amount of prior period items should be separately disclosed in the statement of profit and loss

in a manner that their impact on the current profit or loss can be perceived.

Changes in Accounting Policy:

Accounting policies are the specific accounting principles and the methods of applying those principles

adopted by an enterprise in the preparation and presentation of financial statements.

A change in an accounting policy should be made only if the adoption of a different accounting policy is

required:

(a) by statute

(b) for compliance with an accounting standard

(c) if it is considered that the change would result in a more appropriate presentation of the financial

statements of the enterprise.

Any change in accounting policy which has a material effect, should be disclosed. Such changes should be

disclosed in the statement of profit and loss in a manner that their impact on profit or loss can be perceived.

Where the effect of such change is not ascertainable, the fact should be indicated.

If a change is made in the accounting policies which has no material effect on the financial statements for the

current period but which is reasonably expected to have material effect in later periods, the fact of such

change should be appropriately disclosed in the period in which the change is adopted.

The following are not changes in accounting policies:

(a) the adoption of an accounting policy for events which differ in substance from previously occurring events

e.g. introduction of a formal retirement gratuity scheme by an employer in place of ad hoc ex-gratia

payments to employees on retirement; and

(b) the adoption of a new accounting policy for events or transactions which did not occur previously or that

were immaterial.

Change in Accounting Estimate:

The nature and amount of a change in an accounting estimate which has a material effect in the current

period, or which is expected to have a material effect in subsequent periods, should be disclosed. If it is

impracticable to quantify the amount, this fact should be disclosed.

The effect of a change in an accounting estimate should be classified using the same classification in the

statement of profit and loss as was previously for the estimate.

For example, the effect of a change in an accounting estimate that was previously included as an

extraordinary item is reported as an extraordinary item.

Clarifications:

(a) Change in accounting estimate does not bring the adjustment within the definitions of an extraordinary

item or a prior period item.

(b) Sometimes, it is difficult to distinguish between a change in an accounting policy and a change in

accounting estimate. In such cases, the change is treated as a change in an accounting estimate, with

appropriate disclosures.

AS – 6

DEPRECIATION ACCOUNTING

Depreciation is a measure of the wearing out, consumption or other loss of value of a

depreciable asset arising from use, passage of time or obsolescence through technology

and market changes. Depreciation is allocated so as to charge a fair proportion of the

depreciable amount in each accounting period during the expected useful life of the asset.

Depreciation includes amortisation of assets whose useful life is predetermined.

The depreciable amount of a depreciable asset should be allocated on a systematic

basis to each accounting period during the useful life of the asset.

Depreciable assets are assets which

[1] are expected to be used during more than one accounting period; and

[2] have a limited useful life; and

[3] are held by an enterprise for use in the production or supply or for administrative

purposes

Depreciable amount of a depreciable asset is its historical cost, or other amount

substituted for historical cost less the estimated residual value.

Useful life is the period over which a depreciable asset is expected to be used by the

enterprise.

The useful life of a depreciable asset is shorter than its physical life.

There are two method of depreciation:

1] Straight Line Method (SLM)

2] Written Down Value Method (WDVM)

Note: A combination of more than one method may be used.

The depreciation method selected should be applied consistently from period to

period. The change in method of depreciation should be made only if;

The adoption of the new method is required by statute; or

For compliance with an accounting standard; or

If it is considered that change would result in a more appropriate

preparation of financial statement; or

When there is change in method of depreciation, depreciation should be

recalculated in accordance with the new method from the date of the assets coming

into use. (i.e RETROSPECTIVELY)

The deficiency or surplus arising from such recomputation should be adjusted in the

year of change through profit and loss account.

Such change should be treated as a change in accounting policy and its effect

should be quantified and disclosed.

The useful lives of major depreciable assets may be reviewed periodically. Where

there is a revision of the estimated useful life, the unamortised depreciable amount

should be charged over the revised remaining useful life. (i.e. PROSPECTIVELY)

Any addition or extension which becomes an integral part of the existing asset

should be depreciated over the remaining useful life of that asset.

The depreciation on such addition may also be applied at the rate applied to the

existing asset.

Where an addition or extension retains a separate identity and is capable of being

used after the existing asset is disposed of, depreciation should be provided

independently on the basis of estimate of its own useful life.

Where the historical cost of a depreciable asset has undergone a change due to

increase or decrease in the long term liability on account of exchange fluctuations,

price adjustments, changes in duties or similar factors, the depreciation on the revised

unamortised depreciable amount should be provided prospectively over the residual

useful life of the asset.

This accounting standard is not applied on the following items.

Forests and plantations

Wasting assets

Research and development expenditure

Goodwill

Live stock

Disclosure requirements

1] the historical cost

2] total depreciation for each class charged during the period

3] the related accumulated depreciation

4] depreciation method used ( Accounting policy)

5] depreciation rates if they are different from those prescribed by the statute

governing the enterprise

AS – 7

CONSTRUCTION CONTRACT

A Construction contract is a contract specifically negotiated for the construction of an asset or a

combination of assets that are closely interrelated or interdependent in terms of their design,

technology and function or their ultimate purpose or use.

Recognition of contract revenue and contract cost

When the outcome of a construction contract can be estimated reliably, contract revenue and contract

cost should be recognized as revenue and expenses by reference to the stage of construction. (This

accounting standard recommends the use of percentage of completion method)

When the outcome of a construction contract cannot be estimated reliably,

Revenue should be recognized only to the extent of contract costs incurred of which recovery is

probable. (i.e. Revenue recognized = Costs Incurred )

Contract costs should be recognized as an expense in the period in which they are incurred.

The outcome of a construction contract can be estimated reliably when all the following conditions

are satisfied:

(a) total contract revenue can be measured reliably;

(b) the receipt of revenue is probable;

(c) the contract costs to complete the contract can be measured reliably;

(d) the stage of completion at the reporting date can be measured reliably;

(e) the contract costs attributable to the contract can be clearly identified.

Contract revenue should comprise:

(a) the initial amount of revenue agreed in the contract; and

(b) variations in amount to be received

- to the extent that it is probable that they will result in revenue; and

- they are capable of being reliably measured.

( Contract can of two kinds: Fixed Price contract and Cost Plus contract)

Contract costs should comprise:

(a) costs that relate directly to the specific contract;

(b) costs that are attributable to contract activity in general and can be allocated to the contract.

At any stage of contract, when it is probable that total contract costs will exceed total contract

revenue, the expected loss should be recognized as an expense immediately. The amount of

such loss is determined irrespective of:

- whether or not work has commenced on the contract;

- the stage of completion of contract activity; or

- whether outcome of contract is estimated or not

When an uncertainty arises about the collectability of an amount already included in contract

revenue, and already recognized in the statement of profit and loss, the uncollectable amount

or the amount in respect of which recovery has ceased to be probable is recognized as an

expense rather than an adjustment of the amount of contract revenue.

Contract costs that relate to future activity, are recognized as an asset provided it is probable

that they will be recovered. Such asset is classified as Contract WIP.

The stage of completion of a contract may be determined by following ways;

- surveys of work done

- completion of physical proportion of the contract work

- the proportion that contract costs incurred for work performed upto

the reporting date bear to the estimated total contract costs

When a contract covers a number of assets, the construction of each asset should be treated as

a separate construction of each asset should be treated as separate construction contract when

- separate proposals have been submitted for each asset;

- each asset has been subject to separate negotiation

- the costs and revenues of each asset can be identified.

A group of contracts, whether with a single customer or with several customers, should be

treated as a single construction contract when

- the group of contracts is negotiated as a single package;

- the contracts are very closely interrelated

- the contracts are performed concurrently or in a continuous sequence.

The recognition of revenue and expenses in construction contract is based on reliable estimate.

This estimate may vary from one accounting year to another accounting year. The effect of

change in estimate should be treated as per AS-5. i.e. It should not be treated as prior period

item or extraordinary item.

Disclosure: - Contract Revenue recognized as revenue

- Method used to determine the contract revenue

- Method used to determine the stage of completion

- Contract costs incurred + Recognised Profit – Recognised Loss

- Amount of advances received

- Amount due from customers

- Amount due to customers

AS – 9

REVENUE RECOGNITION

Revenue is the gross inflow of cash, receivables or other consideration arising in the course of the ordinary

activities of an enterprise from the sale of goods, from the rendering of services, and from the use by others of

enterprise resources yielding interest, royalties and dividends.

Revenue includes: - Proceeds from sale of goods

- Proceeds from rendering of services

- Interest, royalty and dividends.

Sale of goods

Revenue from sales should be recognized when

All significant risks and rewards of ownership have been transferred to the buyer from the seller.

Ultimate realisability of receipt is reasonably certain.

Rendering of Services

Revenue from service transactions is usually recognized as the service is performed, either by proportionate

completion method or by the completed service contract method.

1) Proportionate Completion method – This is a method of accounting, which recognises revenue in the

statement of profit and loss proportionately with degree of completion of services under a contract.

Revenue is recognised by reference to the performance of each act. The revenue recognised under this

method would be determined on the basis of contract value, associated costs, number of acts or other

suitable basis.

2) Completed service contract method – This is a method of accounting, which recognises revenue in the

statement of profit and loss only when the rendering of services under a contract is completed or

substantially completed.

Revenue under this method is recognised on completion or substantial completion of the job.

Revenue from Interest : Recognised on time proportion basis

Revenue from Royalties: Recognised on accrual basis in accordance with the terms of the relevant

agreement.

Revenue from Dividends: Recognised when right to receive is established

Subsequent uncertainty in collection: When the uncertainty relating to collectability arises subsequent to the

time of sale or the rendering of services, it is more appropriate to make a separate provision to reflect the

uncertainty rather than to adjust the amount of revenue originally recorded.

Disclosure: An enterprise should disclose the circumstances in which revenue recognition has been postponed

pending the resolution of significant uncertainties.

EXAMPLES

1] On sale, buyer takes title and accepts billing but delivery is delayed at buyer‟s request

- Revenue should be recognised notwithstanding that physical delivery has not been completed.

2] Delivery subject to installations and inspections

- Revenue should not be recognised until the customer accepts delivery and installation and inspection are

complete. However, when installation process is very simple, revenue should be recognised. For example.

Television sale subject to installation.

3] Sale on approval

- Revenue should not be recognised until the goods have been formally accepted or time for rejection has

elapsed or where no time has been fixed, a reasonable time has elapsed.

4] Sales with the condition of „money back if not completely satisfied‟

- It may be appropriate to recognize the sale but to make suitable provision for returns based on previous

experience.

5] Consignment sales

- Revenue should not be recognised until the goods are sold to a third party.

6] Installment sales

- Revenue of sale price excluding interest should be recognised on the date of sale.

7] Special order and shipments

- Revenue from such sales should be recognized when the goods are identified and ready for delivery.

8] Where seller concurrently agrees to repurchase the same goods at a later date

- The sale should not be recognised, as this is a financial arrangement.

9] Subscriptions received for publications

- Revenue received or billed should be deferred and recognised either on a straight-line basis over time or

where the items delivered vary in value from period to period, revenue should be based on the sales value

of the item delivered.

10] Advertisement commission received

- It is recognised when the advertisement appears before public.

11] Tution fees received

- Should be recognised over the period of instruction.

12] Entrance and membership fees

- Entrance fee is generally capitalized

- If the membership fee permits only membership and all other services or products are paid for

separately, the fee should be recognised when received. If the membership fee entitles the member to

services or publications to be provided during the year, it should be recognised on a systematic and

rational basis having regard to the timing and nature of all services.

13] Sale of show tickets

- Revenue should be recognised when the event takes place.

14] Guaranteed sales of agricultural crops

- When sale is assured under forward contract or government guarantee, the crops can be recognised at

net realizable value although it does not satisfy the criteria of revenue recognition.

The above accounting standard is not applicable for:

- Revenue arising from construction contracts

- Revenue arising from hire purchase, lease agreements

- Revenue arising from Government grants and subsidies

- Revenue of Insurance companies arising from insurance contracts

- Profit or loss on sale of fixed assets

- Realised or unrealized gains resulting from changes in foreign exchange rates

AS-10

ACCOUNTING FOR FIXED ASSETS

Definitions:

Fixed Asset is an asset held with the intention of being used for the purpose of producing or providing goods

or services and is not held for sale in the normal course of business. (It is expected to be used for more than

one accounting period.)

The cost of fixed asset includes:

Purchase price

Import Duties and other non-refundable taxes

Direct cost incurred to bring the asset to its working condition

Installation cost

Professional fees like fees of architects

General overhead of enterprise when these expenses are specifically attributable to

acquisition/preparation of fixed assets

Any expenses before the commercial production, including cost of test run and experimental

production

Any expenses before the asset is ready for use not put to use

Loss on deferred payment arising out of foreign currency liability

Price adjustment, changes in duties and similar factors

The cost of fixed asset is deducted with:

Trade discounts and rebates

Sale proceeds of test run production

Amount of government grants received/receivable against fixed assets (See AS- 12)

Gain on deferred payment arising out of foreign currency liability

Similarly, historical cost of self constructed fixed assets will include:

All cost which are directly related to the specific asset

All costs that are attributable to the construction activity should be allocated to fixed assets

Any internal profit included in the cost should be eliminated.

Any expenses incurred on asset between date of ready for use and put to use is either charged to P&L

A/c or treated as deferred revenue expenditure to be amortised in 3-5 years after commencement of

production.

When fixed asset is acquired in exchange for another asset, the cost of the asset acquired should be

recorded

- either at, fair market value

- or at, the net book value of the assets given up

For this purpose, fair market value may be determined by reference either to the asset given up or to

the asset acquired, whichever is more clearly evident.

Fixed asset acquired in exchange for shares or other securities should be recorded at FMV of assets

given up or asset acquired, whichever is more clearly evident. (i.e the option of recording the asset at

net book value of asset given up is closed)

Fair market value is the price that would be agreed to in an open and unrestricted market between

knowledgeable and willing parties dealing at arm‟s length distance.

Subsequent expenditures related to an item of fixed asset should be added to its book value only if

they increase the future benefits from the existing asset beyond its previously assessed standard of

performance.

Material items retired from active use and held for disposal should be stated at the lower of their

net book value and net realizable value and shown separately. Fixed assets should be eliminated

from the financial statements on disposal or when no further benefit is expected from its use and

disposal. Profit/loss on such disposal or writing off is recognized in the profit and loss account.

REVALUATION When the fixed assets are revalued, these assets are shown at revalued price. Revaluation of fixed

assets should be restricted to the net recoverable amount of fixed asset.

When a fixed asset is revalued, an entire class of assets should be revalued or selection of assets

for revaluation should be made on a systematic basis. That basis must be disclosed.

Accounting treatment of revaluation under different situation:

When revaluation is made upward

Fixed Assets A/c Dr

To Revaluation Reserve

When revaluation is made downward

P&L A/c Dr

To Fixed Assets

When revaluation is made upward subsequent to previous upward revaluation

Fixed Assets A/c Dr

To Revaluation Reserve

When revaluation is made downward subsequent to previous upward revaluation

Revaluation Reserve A/c Dr (To the extent of carrying amount of R.R)

P&L A/c Dr (Balancing Figure)

To Fixed assets

When revaluation is made upward subsequent to previous downward revaluation

Fixed assets A/c Dr

To P&L A/c (To the extent of previous downward

revaluation)

To Revaluation Reserve (Balancing Figure)

When revaluation is made downward subsequent to previous downward revaluation

P& L A/c Dr

To Fixed Assets

Accounting treatment on disposal of Fixed Assets:

On sale of fixed assets

Bank A/c Dr

P & L A/c Dr (If Loss)

To Fixed Assets

To P & L A/c (If Profit)

On sale of fixed assets where upward revaluation has taken place

On disposal of a previously revalued item of fixed asset, the difference between net disposal

proceeds and the net book value is normally charged or credited to the profit and loss account

except that, to the extent such a loss is related to an increase which was previously recorded as a

credit to revaluation reserve and which has not been subsequently reversed or utilized, it is

charged directly to that account. The amount standing in revaluation reserve following the

retirement or disposal of an asset which relates to that asset may be transferred to general reserve.

If Loss If Profit

Bank A/c Dr Bank A/c Dr

Revaluation Reserve A/c Dr To Fixed Assets A/c

P& L A/c Dr To P/L A/c

To Fixed Assets

Revaluation Reserve A/c Dr Revaluation Reserve A/c Dr

To General Reserve To General Reserve

In the case of fixed assets owned by the enterprise jointly with others, the extent of the enterprise‟s

share in such assets, and the proportion of the original cost, accumulated depreciation and WDV

should be stated in the B/S.

Alternatively, the pro rata cost of such jointly owned assets may be grouped together with similar fully

owned assets with an appropriate disclosure thereof.

Only purchased goodwill should be recorded in books.

Disclosure:

Gross and net book value of fixed assets at the beginning and end of period showing additions and

disposals

Revalued amounts substituted for historical costs of fixed assets, the method adopted to compute

the same and whether an external valuer was involved.

AS - 12

ACCOUNTING FOR GOVERNMENT GRANTS

Applicability: Mandatory for all enterprises with respect from 01/04/1994.

Government Grants are assistance by government in cash or kind for past or future compliance with certain

conditions.

Government grants may be received in following ways.

Grants related to acquisition of fixed assets

Grants related to revenue

Grants related to promoter‟s contribution

Grants related to compensation for expenses

Government Grants should be recognised

Where there is reasonable assurance that the enterprise will comply with the conditions attached to

them; and

The grants will be received.

Amount of Grant:

Monetary Grant: Amount earned should be the value of grant.

Non- Monetary Grant:

Where grants are given at concessional rate, then such assets are accounted for at their acquisition

cost.

Where grants are given free of cost, then such assets are recorded at nominal value.

Accounting Treatment:

Grants related to Depreciable assets:

EITHER, Grants are shown as deduction from Gross value of assets

Bank A/c Dr

To Government Grant

Government Grant Dr

To Fixed Assets

(When grant is equal to book value of asset, fixed asset is shown at nominal value.)

OR, Grants are treated as deferred income

Bank A/c Dr

To Grant

[In this case, Grants are recognised as profit in P&L A/c on a systematic and rational basis over the useful

life of assets (i.e. in proportion to the amount of depreciation charged over period)]

Net effect on Profit & Loss A/c will remain same in both cases

Grants related to Non-Depreciable assets:

EITHER, Grants are shown as deduction from Gross value of assets

Bank A/c Dr

To Government Grant

Government Grant Dr

To Fixed Assets

(When grant is equal to book value of asset, fixed asset is shown at nominal value.)

OR, shown as reserves

When no future obligations are to be fulfilled

Bank A/c Dr

To Gov. Grant

Gov. Grant Dr

To Capital Reserve

When grant requires fulfillment of certain obligations:

Bank A/c Dr

To Gov. Grant

Gov. Grant Dr (Should be credited to income over the same period over which

the cost of

To P&L A/c meeting such expense is charged to revenue)

(In respective years)

The deferred income balance should be separately disclosed in the financial statement.

Grants related to revenue

Government grants related to revenue should be recognised on a systematic basis in the profit and loss

account over the periods necessary to match with the related costs, which they are intended to compensate.

Grants related to promoter’s contribution

Grant should be treated as Capital Reserve.

Bank A/c Dr

To Gov Grant

Gov Grant A/c Dr

To Capital Reserve

Grants related to compensation for expenses

Government grants receivable as compensation for expenses or losses (with no further costs) should be

recognised as an income in the year of receivable as an Extra-ordinary item.

REFUND OF GOVERNMENT GRANT

Government grants sometimes become refundable because certain conditions are not fulfilled. The grant

refundable is treated as an extraordinary item.

The amount refundable in respect of a government grant related to a specific asset is recorded by increasing

the book value of the asset or by reducing the capital reserve or the deferred income balance, as appropriate,

by the amount refundable. (Where the book value of asset is increased, the depreciation should be provided on

new asset value prospectively)

Where the amount refundable is in respect of a government grant related to revenue, the refund is applied first

against any unamortised deferred credit remaining in respect of the grant. Rest amount of refund should be

charged to profit and loss account.

Where, the amount refundable is in respect of promoter‟s contribution, the capital reserve should be reduced

by the amount refundable.

Contingency related to Govt. Grant

A contingency related to Govt. grant receivable and refundable should be treated in accordance with AS-4.

Disclosures:

The accounting policy adopted

The nature and extent of gov. grants recognised in the financial statements.

AS - 13

ACCOUNTING FOR INVESTMENTS (Revised in 2003)

Applicability: Mandatory for all enterprises.

Investments are classified as Long Term Investments and Short Term Investments.

Current Investment is intended to be held for not more than one year and readily realisable.

Long term Investment is an investment other than a current investment.

The carrying amount of current investments is lower of cost and fair value.

It is prudent to carry investments individually at the lower of cost and fair value. But, such

comparison can also be made category-wise.

The carrying amount of long-term investments is carried at cost. However, when there is

permanent decline in the value of a long-term investment, the carrying amount is reduced to

recognize the decline. The carrying amount of long-term investments should be determined on

individual basis.

Any reduction or reversal of reduction in value of investment is adjusted through P&L A/c.

Cost of Investments:

The cost of an investment should include acquisition charges such as brokerage, fees and duties.

If an investment is acquired-

- by issue of shares or other securities; then the investments should be valued at the

fair value of the issued security. (i.e. Issue price determined by statutory authority)

- By exchange of another asset; then the investments should be valued at fair value of

the asset given up or asset acquired, whichever is more clearly evident.

Investment property is investment in land or buildings that is not intended to be occupied

substantially for use by, or in the operations of, the investing enterprise. An investment

property is classified as long-term investment.

Disposal of Investments : On disposal, the difference between the carrying amount and

the disposal proceeds, net of expenses, is recognized in the profit and loss statement.

Reclassification of investments:

Long-term to short-term: Transfers from one class to another class are made at lower of

cost and carrying amount at the date of transfer.

Current to long-term: Transfers are made at lower of cost and fair value at the date of

transfer.

Disclosure:

1] Accounting policies for determination of carrying amount

2] Classification of Investments

3] The amounts included in Profit and loss statement

- profits or losses on disposal and changes in carrying amount of current and

longterm investments

- interest, dividends (showing separately dividends from subsidiary) and rentals on

investments showing separately such income from current and long term

investments.

- Gross Income should be disclosed (i.e. The amount of TDS should be shown under

advance taxes paid)

4] Aggregate amount of quoted and unquoted investments giving the aggregate market

value of quoted investments.

AS- 15

ACCOUNTING FOR RETIREMENT BENEFITS

(Revised in 2005 & titled as Employees Benefit)

Applicability: It is mandatory for all enterprises.

Retirement Benefits consists of :

1. Provident Fund

2. Superannuation / Pension

3. Gratuity

4. Leave Encashment Benefit

5. Other Retirement Benefits

Accounting treatment under Defined Contribution Scheme/ Provident Fund

Contribution payable by the employer in a year is charged to profit & loss account.

Accounting treatment under Defined Benefit Scheme/ Gratuity/ Leave Encashment

Payment of Retirement Benefit out of its own fund

Appropriate provision for accruing liability is created through profit & loss account. Accruing

liability is calculated by actuarial method.

Note: Actuarial valuation is the process used by an actuary (expert) to estimate the present value

of benefits to be paid under a retirement benefit scheme. Actuarial valuation should normally be

conducted at least once in every three years. Differences arising after fresh actuary valuation

should be adjusted through Profit & Loss account in the year in which fresh actuary valuation is

conducted.

Benefits funded through creation of a trust

Amount to be contributed to the trust every year is provided through profit & loss account. The

amount to be contributed is calculated by actuarial valuation.

Benefits funded through a scheme administrated by the insurer

The premium paid to the insurer is charged to profit & loss account. Such premium is calculated

through actuarial valuation.

Review of Actuarial Method/Assumption

Any alterations in the retirement benefit costs, arising due to change in method/ assumption, are

EITHER, charged to credited to profit and loss account in the year of change in accordance with

Accounting Standard 5. “Prior Period and extra ordinary item and changes in accounting

policies.”

OR, spread over a period not more than the expected remaining working lives of the participating

Employees.

Disclosure:

Method by which retirement benefit costs for the period have been defined

When accounting is made as per actuarial valuation, date on which such valuation was conducted.

AS-17

SEGMENTAL REPORTING

A BUSINESS SEGMENT is a distinguishable component of an enterprise that is engaged in providing an

individual product or service or a group of related products or services and that is subject to risks and

returns that are different from those of other business segments.

A GEOGRAPHICAL SEGMENT is a distinguishable component of an enterprise that is engaged in

providing products or services within a particular economic environment and that is subject to risks and

returns that are different from those of components operating in other economic environments.

The risks and returns of an enterprise are both by the geographical

(1) location of production or service facilities and other assets of an enterprise and

(2) location of its customers.

The definition allows geographical segments to be based on any of the two.

A REPORTABLE SEGMENT is a business segment or a geographical segment identified on the basis of

foregoing definitions for which segment information is required to be disclosed by the standard.

ENTERPRISE REVENUE is revenue is revenue from sales to external customers as reported in the

statement of profit and loss. (i.e. Sales made to external customers by all segments)

SEGMENT REVENUE is the aggregate of

(f) revenue directly attributable to segments

(g) revenue reasonably allocated to segment; and

(h) revenue from transactions with other segments.

SEGMENT EXPENSE is the aggregate of

(a) operating expense directly attributable to segment

(b) expenses reasonably allocated to segment; and

(c) expenses relating to transactions with other segments.

However, SEGMENT REVENUE/EXPENSE does not include

(a) Extraordinary items as defiened in AS-5

(b) Interest or dividend ( including earned/incurred on loans to other segment) unless the operations of

the segment are primarily of a financial nature

(c) Gains on sales of investments or on extinguishments of debt (Capital gain/loss) unless the operations

of the segment are primarily of a financial nature.

(d) General administration expenses, head office expenses and other expenses that arise at the enterprise

level and relate to the enterprise as a whole.

SEGMENT RESULT is segment revenue less segment expenses.

SEGMENT ASSETS are those operating assets that are employed by a segment in its operating activities

and that either are directly attributable the segment or can be allocated to the segment on a reasonable

basis.

SEGMENT LIABILITIES are those operating liabilities that result from operating activities and that

either are directly attributable the segment or can be allocated to the segment on a reasonable basis.

( If the segment result of a segment includes interest expense, its segment liabilities include the related

interest-bearing liabilities and vice versa.)

(Segment liabilities do not include income tax liabilities and vice versa.)

Similarly, if depreciation segment expenses then related assets comes under segment assets.

Primary segment and Secondary segment

One among the two, Business Segment and Geographical Segment, is primary segment and other becomes

secondary segment. The reporting requirements for the primary and secondary segments are different.

Basis for identifying primary and secondary segments

Risks and returns are the main criteria for identifying primary and secondary segments.

If the risks and returns of an enterprise are affected predominantly by differences in the products,

business segments are recognized as primary segments and geographical segments as secondary

segments and vice versa.

If the risks and returns of an enterprise are affected both by differences in the products as well as

differences in the locations in which it operates, then the enterprise should use business segments

as its primary segment and geographical segment as its secondary segment.

If risks and returns of an enterprise are affected neither by differences in products/services nor by

differences in geographical areas of operations, the management may elect any of the two as

primary with other being secondary segment.

(Internal organization and management structure of an enterprise and its system of internal financial

reporting to the board of directors and the CEO should normally be the basis for identifying the

predominant source and nature of risks and differing rates of return facing the enterprise.)

Reportable Segments

A business segment or geographical segment should be identified as reportable segment if:

(a) its revenue from sales to external customers and from transactions with other segments is 10% or

more of the total revenue, external and internal, of all segments; or

(b) its segment result, whether profit or loss, is 10% or more of-

(1) the combined result of all segments in profit, or

(2) the combined result of all segments in loss,

whichever is greater in absolute amount; or

(c ) its segment assets are 10% or more of the total assets of all segments.

A business/reportable segment that is not a reportable segment as per above, may be recognized as

reportable segment despite its size at the discretion of the management of the enterprise.

If total external revenue attributable to reportable segments constitutes less than 75% of the total

enterprise revenue, additional segments should be identified as reportable segments, even if they do not

meet 10% thresholds as above, until at least 75% percent of the total enterprise revenue is included in

reportable segments.

A segment identified as a reportable segment in the immediately preceding period because it satisfied the

relevant 10% thresholds should continue to be a reportable segment for the current period

notwithstanding that its revenue, result, and assets no longer meet the 10% thresholds.

Primary Reporting Format

The following disclosures are required for each reportable segment of primary segment:

(1) Segment revenue, (with classification of external and internal)

(2) Segment result

(3) Total carrying amount of segment assets

(4) Total amount of segment liabilities

(5) Total cost incurred during the period to acquire segment assets

(6) Depreciation and amortisation recognized as expense, and

(7) Total non cash expenses other than Dep. And amortisation recognized as expense.

Disclosers required pursuant to clause (6) and (7) above, need not be made in respect of a segment, if the

enterprise reports cash flows arising from operating, investing and financing activities for such segment.

An enterprise should present a reconciliation between the information disclosed for reportable segments

and aggregated information in the enterprise financial statements (in respect of clause 1 to 4 above)

Secondary Reporting Format

Where primary segments are business segments

(1) Segment revenue from external customers for each geographical segment whose revenue from

sales to external customers is 10% or more of enterprise‟s revenue;

(2) The total carrying amount of segment assets for each geographical segment whose segment

assets is 10% or more of the total assets of all geographical segments;

(3) New assets acquired for each geographical segment whose segment assets is 10% or more of

the total assets of all geographical segments.

Where primary segments are geographical segments based on location of assets

The following information (point “1” to “3”)should be disclosed for each business segment whose

revenue from sales to external customers is 10% or more of enterprise revenue or whose segment

assets are 10% or more of the total assets of all business segments

(1) segment revenue from external customers;

(2) the total carrying amount of segment assets; and

(3) new segment assets acquired;

(4) if location of customers is different from location of its assets, then the enterprise should

also report revenue from sales to external customers for each customers based

geographical segment whose revenue from sales to external customers is 10% or more of

enterprise revenue.

Where primary segments are geographical segments based on location to customers

Points 1 to 3 as above.

(5) If locations of assets are different from locations of customers, then the enterprise is

required to report the following segment information for each asset based geographical

segment whose revenue from sales to external customers is 1% or more of enterprise

revenue or whose segment assets are 10% or more of total enterprise assets

I. The total carrying amount of segment assets by geographical location of the assets;

and

II. New segment assets acquired by location of assets.

Other Disclosures:

In measuring and reporting segment revenue from transactions with other segments, inter-segment

transfers should be measured on the basis that the enterprise actually used to price those transfers.

The transfer-pricing basis should be disclosed in the financial statements.

An enterprise should indicate the types of products and services included in each reported business

segment and indicate the composition of each reported geographical segment, primary and secondary,

if not otherwise disclosed in the financial statements.

AS-16

BORROWING COSTS

Applicability: Mandatory for all enterprises w.e.f. 01/04/2000.

Borrowing Costs include:

1. Interest and commitment charges on borrowings

2. Amortization of discounts or premiums relating to borrowings

3. Amortisation of ancillary costs incurred in connection with the arrangement of

borrowings

4. Exchange difference arising from borrowings to the extent it amounts to interest costs.

Borrowing costs should be recognized as an expense in the period in which they are incurred.

Borrowing costs that are directly attributable to the acquisition, construction or production of a

qualifying asset should be capitalized as part of that asset.

Qualifying Asset is an asset that necessarily takes a substantial period of time to get ready for its

intended use or sale. e.g. Heavy Plant & Machinery.

BORROWING COST ELIGIBLE FOR CAPITALISATION

Specific Borrowing for acquisition of qualifying asset: Borrowing cost to be capitalised

Amount of borrowing cost = Specific Borrowing Cost – Income from temporary investment

General Borrowing and used for acquisition of qualifying asset: Borrowing cost should be capitalised

with the following amount;

Amount of Borrowing Cost = Expenditure cost on asset or Asset cost * Capitalisation rate

Capitalisation Rate = Weighted Average Borrowing costs on general borrowing

(i.e. Excluding cost of specific borrowing)

Note: When with the capitalization of borrowing cost, the cost exceeds the net recoverable amount, the

carrying amount is written down to net recoverable amount as per the recommendation of other

accounting standards.

COMMENCEMENT OF CAPITALISATION

Capitalisation of Borrowing should commence when all the following conditions are satisfied

1. Expenditure for the acquisition of a qualifying asset is being incurred

2. Borrowing costs are being incurred; and

3. Activities that are necessary to prepare the asset for its intended use or sale are in progress

SUSPENSION OF CAPITALISATION

Capitalisation of borrowing costs should be suspended during extended periods in which active

development is interrupted.

However capitalization should not be suspended when a temporary delay is a necessary part of the

process of getting an asset ready for its intended use or sale.

Example:

Borrowing costs incurred while land is under development are capitalized during the period in

which activities related to the development are being undertaken. However, it should not be

capitalized when land acquired for building purposes is held without use.

CESSATION OF CAPITALISATION

Capitalisation of borrowing costs should cease when substantially all the activities necessary to

prepare the qualifying asset for its intended use or sale are complete.

When the construction of a qualifying asset is completed in parts

and a completed part is capable of being used while construction continues for the other parts,

capitalization of borrowing costs in relation to that part should cease

when substantially all the activities necessary to prepare that part for its intended use or sale are

complete.

Example:

Housing complex comprising several buildings: If individual building can be used separately, its

capitalisation should cease.

Disclosure:

1. The Accounting Policies adopted for borrowing costs.

2. The amount of borrowing costs capitalized during the period.

AS-18

RELATED PARTY DISCLOSURES

(revised in 2003)

Applicability: Mandatory for all enterprises with respect from 01/04/2004

Related party is considered to be related if at any time during the reporting period

one party has the ability to control the other party or exercise significant influence over the

other party in making financial and/or operating decisions. The following related party

relationship are covered under AS-18:

1. Enterprises that directly, or indirectly through one or more intermediaries, control, or are

controlled by, or are under common control with the reporting enterprise ( e.g. Holding

companies, subsidiaries

and fellow subsidiaries)

2. Associates and joint ventures of the reporting enterprise

3. Individuals owning, directly or indirectly, an interest in the voting power of the reporting

enterprise that gives them control or significant influence over the enterprise, and relatives of any

such individual;

4. Key management personnel and relatives of such personnel; and

5. Enterprises over which any person described in (4) or (5) is able to exercise significant influence.

Key Management Personnel

Those persons who have the authority and responsibility for planning, directing and controlling the

activities of the reporting enterprise.

The following are not deemed to be related party:

1. Two companies simply because they have common director

2. A single customer/supplier with whom an enterprise transacts a significant volume of business

merely by virtue of the resulting economic dependence

3. Providers of finance

4. Trade Unions

5. Public Utilities

6. Government departments and Government sponsored bodies

DISCLOSURE

If there have been transactions between related parties, during the existence of a related party

relationships, the reporting enterprise should disclose the followings:

1. Name of the transacting related party

2. Description of the relationship

3. Description of nature of transaction

4. Volume of transaction ( Amount wise or proportion wise)

5. Any other information necessary for understanding financial statements

6. Outstanding ( Amount wise or proportion wise) and any provision for doubtful debt due

from such party

7. Amounts written off or written back in respect of debts due from or to related parties.

The followings are the examples of the related party transactions

Purchase or sale of goods

Rendering or receiving services

Purchases or sales of fixed assets

Licence agreements

Leasing or hire purchase agreements

Guarantees and collaterals

Management contracts including for deputation of employees.

AS-19

LEASES

Applicability:- Mandatory for all enterprises w.e.f. 1.04.2001.

It should be applied in accounting for all leases other than:

a) lease agreements to explore for or use natural resources;

b) licensing agreements for items such as plays, manuscripts, patents and copyrights; and

c) lease agreements to use lands.

Lease : A lease is an agreement whereby the lessor conveys to the lessee in return for a payment or

series of payments the right to use an asset for an agreed period of time.

Finance Lease : All risks and rewards incident to ownership of an asset is transferred.

Operating Lease : Lease other than finance lease; i.e. which does not transfer all the risk and

reward incidental to ownership.

Minimum Lease Payments : For lessor – Total Lease rent to be paid over the lease term

+

Any Guaranteed Residual Value by or on behalf of Lessee

+

Residual Value Guaranteed by Third Party

(-)

Contingent Rent

(-)

Cost for Service and tax to be paid by and reimbursed to lessor

For lessee - Total Lease rent to be paid over the lease term

+

Any Guaranteed Residual Value by or on behalf of Lessee

(-)

Contingent Rent

(-)

Cost for Service and tax to be paid by and reimbursed to lessor

Accounting for Finance Lease – In the books of lessee The lessee should recognize the lease as an asset at lower of the following

- Fair Value of the leased asset

- Present value of minimum lease payments

(In calculating the present value of the minimum lease payments, the discount rate is the interest rate

implicit in the lease. If implicit rate is not known, the lessee‟s incremental borrowing rate should be

used.) Entry required to be passed:

Lease Assets A/c Dr

To Lessor

All lease payments should then be apportioned between the finance charge and the reduction of the

outstanding liability. Finance charge should be debited to P&L A/c.

Lessor A/c Dr

P&L A/c Dr (With the amount of finance charge)

To Bank A/c (With the amount of lease payment)

The lessee as per AS-6 should depreciate the leased asset.

Accounting for Finance Lease – In the books of lessor

The lessor should recognize the transaction as sale with the cash price. If artificially low rates of

interest are quoted, profit on sale should be restricted to that which would apply if a commercial rate

of interest were charged.

The cost of sale recognized at the commencement of the lease term is the cost/carrying amount less the

present value of the unguaranteed residual value.

Accounting for Operating Lease – In the books of lessee

Lease payments (excluding costs for services such as insurance and maintenance) are recognized as

an expense in the statement of profit or loss on a straight-line basis unless another systematic basis is

more appropriate.

Accounting for Operating Lease – In the books of lessor

Lease receipts are recognized as an income in the statement of profit or loss on a straight-line basis

unless another systematic basis is more appropriate. The lessor should present an asset given under

operating lease in its balance sheet under fixed assets.

Initial direct costs incurred specifically to earn revenues from an operating lease are

Either, deferred and allocated to income over the lease term in proportion of income

Or, recognized as an expense in the statement of current year profit and loss.

SALE AND LEASEBACK TRANSACTIONS

A sale and leaseback transaction involves the sale of an asset by the vendor and the leasing of the

same asset back to the vendor.

If sale and leaseback transaction results in finance lease:

Excess or deficiency of sale proceeds over the carrying amount should be deferred and amortised over

the lease term in proportion to the depreciation of the leased asset. It should not be immediately

recognized as income or loss in the financial statements.

If sale and leaseback transaction results in operating lease:

If the sale price is equal to fair value

- Any profit or loss should be recognized immediately.

If the sale price is below fair value

- Any profit or loss should be recognized immediately, except that, if the loss is

compensated by future lease payments at below market price

- If the loss is compensated by future lease payments at below market price, the profit

or loss should be deferred and amortised in proportion to the lease payments.

If the sale price is above fair value

- The excess over fair value should be deferred and amortised over the period for

which the asset is expected to be used.

Further, if the fair value at the time of a sale and leaseback transaction is less than the carrying

amount of the asset, a loss equal to the amount of the difference between the carrying amount and fair

value should be recognized immediately.

Disclosure Requirements:

In the books of lessee in case of financial lease

1. Assets acquired under finance lease

2. Reconciliation between the total of minimum lease payments and their present value as at the

balance sheet date with following segregation

- not later than one year

- later than one year and not later than five years

- later than five years

3. Contingent rents recognized as expense

4. Future minimum sublease payments expected to be received under non-cancellable subleases

5. General description of the leasing arrangements

In the books of lessor in case of financial lease

1. General description of the significant leasing arrangement

2. Accounting policy for initial direct cost

3. Reconciliation of total gross investment in lease and present value of minimum lease payment

(MLP) receivable at the balance sheet date

4. MLP receivable in following categories

- not later than one year

- later than one year and not later than five years

- later than five years

In the books of lessee in case of operating lease

1. General description of the significant leasing arrangement

2. Total of future minimum lease payments in the following period

- not later than one year

- later than one year and not later than five years

- later than five years

3. Lease payments recognized in profit & loss account for the period

In the books of lessor in case of operating lease

1. General description of the significant leasing arrangement

2. Accounting policy for the initial direct payment

3. Future lease payments in aggregate classified as :

- not later than one year

- later than one year and not later than five years

- later than five years

AS-20

EARNINGS PER SHARE

(Revised in 2004)

Applicability:- Mandatory w.e.f. 1.04.2001 in respect of enterprises whose equity shares or potential

equity shares are listed on a recognized stock exchange in India.

An enterprise should present BASIC & DILUTED EPS on the face of the statement of profit and loss

account for each class of equity shares that has a different right to share in the net profit for the period.

EPS to be calculated & presented even in case of losses.

Basic EPS = Net profit/loss for the period attributable to equity shareholders

Weighted Average No. of Equity Shares

Diluted EPS =

Adjusted Net profit/loss for the period attributable to equity shareholders

Weighted Average No. of (Equity Shares + Dilutive Potential

Equity Shares)

Where net profit/loss for equity shareholders = PAT less Preference Dividend including CDT

(Preference Dividend should be deducted whether or not provided in case of Cumulative Preference

Shares).

Date from which the shares are included for calculation of weighted no. of shares:

Equity shares issued for cash

Date on which cash is received

Debentures converted to cash

Date of conversion

Equity shares issued in exchange for settlement

Of a liability

Date when settlement becomes

effective

Equity shares issued for rendering of services

Date on which services are

rendered

Equity Shares issued in course of

Amalgamation in the nature of Purchase

Date of the Acquisition

Equity Shares issued in course of

Amalgamation in the nature of Purchase

Beginning of the reporting

period

Partly paid equity shares are treated as a fraction of an equity share to the extent that they were entitled

to participate in dividends relative to a fully paid equity shares.

Where an enterprise has equity shares of different nominal values but with the same dividend rights, the

number of equity shares is calculated by converting all such equity shares into equivalent number of

shares of the same nominal value.

BONUS ISSUE, SHARE SPLIT, REVERSE SHARE SPLIT etc.

In these cases, shares are issued to existing shareholders for no additional consideration. Therefore, the

number of equity shares outstanding is increased without an increase in resources.

RIGHTS ISSUE

In rights issue, the exercise price is often less than the fair value of the shares. Therefore, a right issue

generally includes a bonus element. Hence, number of equity shares to be taken for calculating Basic

EPS should be:

Right Shares + (Equity Shares prior to right * conversion factor)

Where, conversion factor = Fair value per share immediately prior to the exercise of

rights

Theoretical ex-rights fair value per share

Where, Theoretical ex-rights fair value = (Fair value of Prior shares + Right Proceeds)

Post Right total no. of equity shares

DILUTED EARNINGS PER SHARE

Diluted EPS =

Adjusted Net profit/loss for the period attributable to equity shareholders

Weighted Average No. of (Equity Shares + Dilutive Potential Equity

Shares)

In calculating diluted EPS, the net profit (considered for BASIC EPS) is adjusted with the corresponding

changes in profits that shall arise when dilutive potential shares are issued. For example: When

debentures are converted to shares, the net profit should be added with interest amount and further

adjusted with related tax expense.

Potential equity shares should be treated as dilutive when, and only when, their conversion to equity

shares would decrease net profit per share from continuing ordinary operations.

DISCLOSURE

Basic & Diluted EPS

Amount used as numerator & reconciliation with PAT

Number used as denominator for Basic & Diluted EPS & reconciliation thereon

Nominal value of shares along with EPS figures

AS – 22

ACCOUNTING FOR TAXES ON INCOME

Applicability :- a) Mandatory w.e.f. 1-04-2001 in respect of the following:

1.Enterprises whose equity or debt securities are listed on a recognized

stock exchange in India;

2.All the enterprises of a group, if the parent presents consolidated

financial statements.

b) Mandatory w.e.f. 1.04.2002, in respect of companies not covered by a);

c) Mandatory w.e.f. 1.04.2006 in respect of all other enterprises.

Accounting income (loss) is the net profit or loss for a period, as reported in the statement of profit and

loss, before deducting income tax expense or adding income tax saving. (i.e. PBT as per P/L A/c)

Taxable income (tax loss) is the amount of income (loss) for a period, determined in accordance with the

tax laws, based upon which income tax payable (recoverable) is determined. (i.e. GTI)

Tax expense (tax saving) is the aggregate of current tax and deferred tax charged or credited to the

statement of profit and loss for the period. (i.e. tax which is to be debited or credited to P/L A/c)

Current tax is the amount of income tax determined to be payable (recoverable) in respect of the taxable

income (tax loss) for a period. (i.e. tax as per Income tax Act)

Deferred tax is the tax effect of timing differences. Model journal entries to be passed in books of

account should be as under:

Current Tax A/c ………..Dr

To Provision for Current Tax

Deferred Tax A/c ………Dr

To Deferred Tax Liability A/c

OR

Deferred Tax Assets A/c …….Dr

To Deferred Tax A/c

Tax Expense A/c……Dr

Deferred Tax A/c……Dr (In case DTA is created)

To Current Tax A/c

To Deferred Tax A/c (In case DTL is created)

P/L A/c………………Dr

To Current Tax A/c

Permanent differences are the differences between taxable income and accounting income for a period

that originate in one period and do not reverse subsequently.

Examples:

- Expenditure disallowed as per Income Tax Act (Forever)

- Excess expenditure allowed by Income Tax Act, 1961 in respect of Scientific Expenditure

Timing differences are the differences between taxable income and accounting income for a period that

originate in one period and are capable of reversal in one or more subsequent periods.

Examples:

- Depreciation rate/method different as per Accounts and Income tax Calculation

- Expenditure of the nature mentioned in Section 43B (e.g. sales tax charged in account on

accrual basis but not paid; such sales tax will be an allowable expenditure in the year of

payment and a disallowable expenditure in the year in which accrued)

Hints for creation of DTL or DTA

When accounting profit/ loss is higher than taxable profit/loss: Deferred Tax liability is created or

Deferred tax asset is reversed.

When accounting profit/loss is less than taxable profit/loss: Deferred tax asset is created or Deferred

Tax Liability is reversed.

When taxable loss is carried forward for set off: Deferred Tax Asset is created.

When carried forward taxable loss is set off : Deferred Tax Asset is reversed.

However, Deferred Tax Asset (DTA) should be recognized and carried forward only to the extent that

there is a reasonable certainty that sufficient future taxable income will be available against which such

deferred tax assets can be reversed/ realized.

Example: Deferred Tax Asset should be created in respect of taxable loss being carried forward, when

there is reasonable certainty that carried forward taxable loss will be set off. (i.e. Adequate taxable

profit is expected in future)

The carrying amount of deferred tax assets should be reviewed at each balance sheet date. An enterprise

should write down the carrying amount of deferred tax asset to the extent that it is no longer reasonably

certain that sufficient profits will be available.

Such, written down value can be re-stated if it becomes virtually certain that sufficient profits will be

available (for set off).

Also at each balance sheet date, an enterprise re-assesses unrecognized deferred tax assets. The

enterprise recognizes previously unrecognized deferred tax assets to the extent that it has become

reasonably certain that sufficient future taxable income will be available against which such deferred

tax assets can be realized.

Presentation & Disclosure

In the Balance Sheet, a Deferred Tax Asset should be shown after the head “INVESTMENT” and

Deferred Tax Liability should be shown after the head “UNSECURED LOAN”.

Current Tax assets and liabilities should be separately shown with Deferred Tax assets and liabilities.

Deferred Tax asset is set-off with deferred tax liabilities when

- the enterprise has a legally enforceable right to set-off assets against liabilities

representing current tax; and

- the deferred tax assets and the deferred tax liabilities relate to taxes on income levied by

the same governing taxation laws.

The nature of the evidence supporting the recognition of deferred tax assets should be disclosed, if an

enterprise has unabsorbed depreciation or carry forward of losses under tax laws.

AS – 24

DISCONTINUING OPERATIONS

A discontinuing operation is a component of an enterprise:

(a) that the enterprise, pursuant to a single plan, is:

- disposing of substantially in its entirety (example – demerger)

- disposing of piecemeal (selling and settling assets and liabilities one by one)

- terminating through abandonment; and

(b) that represents a separate major line of business or geographical area of operations; and

( c) that can be distinguished operationally and for financial reporting purposes.

Examples of activities that may not satisfy criteria (a) above but that can be discontinuing operations in

combination with other circumstances include:

- gradual or evolutionary phasing out of a product line or class of service;

- discontinuing, even if relatively abruptly, several products within an ongoing line of

business;

- shifting of some production or marketing activities for a particular line of business from

one location to another; and

- closing of a facility to achieve productivity improvements or other cost savings;

- Selling shares of subsidiary whose activities are similar to those of the parent or other

subsidiaries. (In case of Consolidated Financial Statements)

A reportable business segment or geographical segment as defined in AS-17 would normally satisfy

criteria (b) above.

A component that can be distinguished operationally and for financial reporting purposes criteria [c]

above – if all the following conditions are met:

1. the operating assets and liabilities of the component can be directly attributed to it;

2. its revenue can be directly attributed to it;

3. at least a majority of its operating expenses can be directly attributed to it.

Presentation and Disclosure

An enterprise should include the following information relating to a discontinuing operation in its

financial statements beginning with the financial statements for the period in which the initial disclosure

event occurs and up to and including the period in which discontinuance is completed.:

INITIAL DISCLOSURE:

1. A description of the discontinuing operation(s);

2. the business or geographical segment(s) in which it is reported as per AS – 17

3. the date and nature of the initial disclosure event;

4. the date or period in which the discontinuance is expected to be completed if known or

determinable;

5. the carrying amounts, as of the balance sheet date, of the total assets to be disposed of and the

total liabilities to be settled;

6. revenue and expenses from such discontinuing operation in current reporting period;

7. pre-tax profit/loss from discontinuing operation during the current financial reporting period,

and income tax expense.

8. net cash flows attributable to the operating, investing, and financing activities of the

discontinuing operation during the current financial reporting period.

With respect to a discontinuing operation, the initial disclosure event is the occurrence of one of the

following, whichever occurs earlier

(a) the enterprise has entered into a binding sale agreement for substantially all of the assets

of the discontinuing operation; or

(b) the enterprise‟s board of directors or similar governing body has both

(i) approved a formal plan; and

(ii) made an announcement of the plan.

Other Disclosures

When an enterprise disposes of assets or settles liabilities attributable to a discontinuing operation or

enters into binding agreements for the sale of such assets or the settlement of such liabilities, it should

include, in its financial statements, the following information when the event occurs.

(a) Gain or loss recognized on such disposal.

(b) Net selling prices (or range of prices) of those assets for whih the enterprise has

entered into binding contract, the expected timing of receipt of cash flow and the

carrying amount of those assets.

The disclosures required above should be presented in the notes to the financial statements except the

following, which should be shown on the face of the statements of profit or loss;

(a) pre-tax profit/loss from ordinary activities of discontinuing operation and related

income tax expense

(b) pre-tax gain or loss recognized on disposal of assets or settlement of liabilities.

If an enterprise abandons or withdraws from a plan that was previously reported as a discontinuing

operation, that fact, reason therefor and its effect should be disclosed.

Comparative information for prior periods in respect of discontinuing operations should also be deemed

as discontinuing operations.

AS-28

IMPAIRMENT OF ASSETS

Impairment of Assets means weakening in the value of asset. An enterprise should assess at

each balance sheet date whether there is any indication that an asset may be impaired. If

any such indication exists, the enterprise should estimate the recoverable amount of the

asset.

Indication of Impairment of an asset

- External Sources of Information

Market value has declined significantly more than that would be expected as

a result of depreciation.

Adverse effect on the enterprise due to change in technology, market

conditions, etc.

Change in interest rates.

The carrying amount of the net assets of the reporting enterprise is more

than its market capitalization.

- Internal Sources of Information

Physical damage of asset

Significant change in style or extent of use of asset.

Internal Reporting indicates that the economic performance of an asset is, or

will be, worse than expected.

Assets - This AS does not apply to

1. Inventories (AS-2)

2. Assets arising from Const. Contracts (AS-7)

3. Financial Assets including Investments (AS-13)

A Financial Asset is any asset that is :-

Cash ;

A contractual right to receive cash or another financial asset

from another enterprise;

A contractual right to exchange financial instruments with

another enterprise under conditions that are potentially

favorable; or

An ownership interest in another enterprise.

4. Deferred tax Asset (AS-22)

RECOVERABLE AMOUNT : (of asset or cash generating unit)

HIGHER OF

- NET SELLING PRICE OF ASSET

- VALUE IN USE

Net Selling Price : It is the amount obtainable from sale of an asset in an arm‟s length

transaction between knowledgeable, willing parties less the costs of disposal.

Value in use : It is the present value of estimated future Cash flow expected to arise from

continue use of an asset and from its disposal at the end of its useful life.

Estimating the value in use of an asset involves the following steps :- - Estimation of future cash inflows & Outflows.

- Application of appropriate discount rate.

Projection of cash flow should be based on

- Most recent financial budgets/forecasts.

- Reasonable and supportable assumptions on the economic conditions. Giving more

weights to external evidence.

- Steady or declining growth rate for the period beyond the period covered by most

recent budgets/forecasts.

Increasing growth rate can be taken when it can be properly justified. However, growth rate

should not exceed the long-term average growth rate for the products/industries/countries in

which the enterprise operates.

Future cash flows should be estimated for the asset in its current condition.

Estimates of future cash flows should not include estimated future cash inflows or

outflows that are expected to arise from :-

a future restructuring to which an enterprise is not yet committed ; or

future capital expenditure that will improve or enhance the asset in excess of its

originally assessed standard of performance.

Estimates of future cash flows should not include ;

Cash inflows or outflows from financing activities ; or

Income tax receipts or payments.

Foreign Currency Future Cash Flows : Future cash flows are estimated in the currency in

which they will be generated and then discounted using a discount rate appropriate for the

currency. An enterprise translates the present value obtained using the exchange rate at the

balance sheet date (describe in Accounting standard (AS)11, Accounting for the Effects of

changes in Foreign Exchange rates, as the closing rate).

DISCOUNT RATE : The discount rate should be PRE TAX RATE. That reflects current

market assessments of the time value of money & the risk specific to the asset.

As a starting point the enterprise may take into account the following rates :-

- Weighted average cost of capital.

- Market borrowing rate.

- Enterprises incremental Borrowing rates.

IMPAIRMENT LOSS

An Impairment Loss is the amount by which the carrying amount of an asset exceeds its

recoverable amount i.e.

Impairment loss = Carrying amount (-) Recoverable amount.

An impairment loss or a revalued asset is recognized as an expense in the statement

of Profit or Loss. However, an impairment loss on a revalued asset is recognized directly

against any revaluation surplus for the asset to the extent that the impairment loss does not

exceed the amount held in the revaluation surplus for the same asset.

When the amount estimated for an impairment loss is greater than the carrying

amount of the asset to which it relates, an enterprise should recognize a liability if, and

only if, that is required by another Accounting Standards.

Accounting entries required to be passed on recognition of Impairment loss :-

Impairment Loss A/c - Dr.

To Assets A/c.

P/L A/c / revaluation reserve A/c - Dr.

To Impairment Loss.

After such recognition Depreciation should be calculated prospectively considering such

loss :

CASH GENERATING UNIT

Cash generating unit is the

- smallest identifiable group of assets.

- that generates cash inflows.

- That are largely independent of the cash inflows from other assets.

If there is any indication that an asset may be impaired, the recoverable amount should be

estimated for the individual asset.

If it is not possible to estimate the recoverable amount of the individual asset, an enterprise

should determine the recoverable amount of the cash – generating unit to which the asset

belongs (the asset‟s cash – generating unit).

The recoverable amount of an individual asset cannot be determined if:

1) The assets value in use cannot be estimated (to be close to its net selling price) ; and

2) The asset does not generate cash inflow from continuing use that is largely

independent from others.

Example : -

A bus company provides services under contract with a municipality that requires minimum

service on each of five separate routes. Assets devoted to each route and the cash flows

from each route can be identified separately. One of the routes operates at a significant

loss.

Because the enterprise does not have the option to curtail any one bus route, the lowest

level of identifiable cash inflows from continuing use that are largely independent of the

cash inflows from other assets or group of assets is the cash inflows generated by the five

routes together. The cash-generating unit for each route is the bus company as a whole

If an active market exists for the output produced by an asset or a group of asset, this asset

on group of assets should be identified as a separate cash-generating unit, even if output is

used internally.

In such case, future market price should be considered while calculating future cash

inflows and cash outflows.

Cash – generating units should be identified consistently from period to period for the same

asset or types of assets; unless a change is justified.

Allocation of Goodwill & Corporate Assets

Corporate Assets ; are assets other than goodwill that contribute to the future cash flows of

both the cash generating unit under review and other cash generating units.

The Goodwill and Corporate Assets are allocated to cash generating unit on a Reasonable

and Consistent basis to the CGU under review.

Then, the recoverable amount of the CGU under review is compared with the carrying

amount of CGU (including allocated goodwill & corporate assets). (BOTTOM UP TEST).

The impairment loss should be allocated to reduce the carrying amount of the assets of the

unit in the following order :

(a) first, to allocated goodwill, and

(b) then, to the other asset of the CGU on a pro-rata basis based on carrying amount of

individual assets.

Entry to be passed :-

P/L A/c Dr. (with the amount of impairment loss)

To Goodwill (up to the allocated amount)

“ Other Assets (on pro-rata basis)

The carrying amount should not be reduced below to zero. The excess loss (i.e. beyond

zero) should be allocated to other assets on pro-rata basis.

A liability should be recognized for any remaining amount of an impairment loss for a

CGU that is required by another Accounting Standard.

If, in performing the „bottom up test‟ the enterprise could not allocate the carrying amount

of goodwill/Corporate on a reasonable and consistent basis to the CGU under review, the

enterprise should also perform a top-down test :- i.e.

1) Identify the smallest cash generating unit that includes the cash-generating unit

under review and to which the carrying amount of goodwill/Corporate assets can be

allocated on a reasonable and consistent basis (i.e. the large CGU is identified)

2) Then, compare the RA of the larger CGU with CA of larger CGU.

If; CA > RA, identify impairment loss and allocate it first to goodwill and then to

other assets on pro-rata basis.

Example – Application of the „Bottom-Up‟ and „Top-Down‟ Tests to Goodwill

In this example, tax effects are ignored.

At the end of 20X0, enterprise M acquired 100% of enterprise Z for Rs. 3,000 lakhs. Z has 3

cash-generating units A, B and C with net fair values of Rs. 1,200 lakhs, Rs. 800 lakhs and

Rs. 400 lakhs respectively. M recognizes goodwill of rs. 600 lakhs (Rs. 3,000 lakhs less Rs.

2,400 lakhs) that relates to Z.

At the end of 20X4, A makes significant losses. Its recoverable amount is estimated to be

Rs. 1,350 lakhs. Carrying amounts are detailed below.

Carrying amounts at the end of 20X4 (Amount in Rs. lakhs)

End of 20X4 A B C Goodwill

Total

Net carrying amount 1,300 1,200 800 120

3,420

A- Goodwill Can be Allocated on a reasonable and Consistent Basis

At the date of acquisition of Z, the net fair values of A, B and C are considered a

reasonable basis for a pro-rata allocation of the goodwill to A, B and C.

Allocation of goodwill at the end of 20X4

A B C

Total

End of 20X0

Net fair values 1,200 800 400

2,400

Pro-rata 50% 33% 17%

100%

End of 20X4

Net carrying amount 1,300 1,200 800

3,300

Allocation of goodwill

(using the pro-rata above) 60 40 20

120

_______________________________________________

Net carrying amount

(after allocation of goodwill) 1,360 1,240 820

3,420

========================================

In accordance with the „bottom-up‟ test , M computes A‟s recoverable amount to its

carrying amount after the allocation of the carrying amount of goodwill.

Application of „bottom-up‟ test (Amount in Rs. lakhs) End of 20X4

Carrying amount after allocation of goodwill (Schedule 2) 1,360

Recoverable amount 1,350

Impairment loss 10

====

M recognizes an impairment loss of Rs. 10 lakhs for A. The impairment loss is fully

allocated to the goodwill.

B- Goodwill Cannot Be Allocated on a Reasonable and Consistent Basis

There is no reasonable way to allocate the goodwill that arose on the acquisition of Z to A,

B and C. At the end of 20X4, Z‟s recoverable amount is estimated to be Rs. 3,400 lakhs.

At the end of 20X4, M first applies the „bottom-up‟ test. It compares A‟s recoverable

amount to its carrying amount excluding the goodwill.

Application of „bottom-up‟ test (Amount in Rs. lakhs)

End of 20X4 A

Carrying amount 1,300

Recoverable amount 1,350

Impairment loss 0

Therefore, no impairment loss is recognized for A as a result of the „bottom-up‟ test.

Since the goodwill could not be allocated on a reasonable and consistent basis to A, M also

performs a „top-down‟ test. It compares the carrying amount of Z as a whole to its

recoverable amount (Z as a whole is the smallest cash-generating unit that includes A and

to which goodwill can be allocated on a reasonable and consistent basis).

Application of the „top-down‟ test (Amount in Rs. lakhs)

End of 20X4 A B C Goodwill Z

Carrying amount 1,300 1,200 800 120 3,420

Impairment loss arising from the „bottom-up‟ test 0 - - - 0

Carrying amount after the „bottom-up‟ test 1,300 1,200 800 120 3,420

Recoverable amount

3,400

Impairment loss arising from „top-down‟ test

20

Therefore, M recognizes an impairment loss of Rs. 20 lakhs that it allocates fully to

goodwill.

Reversal of an Impairment Loss

An enterprise should assess at each balance sheet date whether there is any indication that

an impairment loss recognized in prior accounting period may no longer exist or may have

decreased.

If any such indication exists, the enterprise should estimate the recoverable amount of that

asset.

Indications of such change are known through External & Internal sources of information.

Impairment loss recognized earlier is reversed.

Such reversal is recognized as income in Profit/loss statement. For Revalued asset, it

should be credited to revaluation reserve(As per AS-10) if earlier revaluation reserve A/c

has been debited on recognition of impairment loss.

Reversal of an Impairment Loss for a Cash Generating Unit

- First, reversal should be made to increase the carrying amount of assets other than

goodwill; and - Then, to goodwill allocated to cash generating unit.

Impairment in case of Discontinuing Operations

If the enterprise wants to sell the discontinuing operation in its entirety, then the

recoverable amount for the entire unit is compared with the carrying amount of entire unit.

If the enterprise wants to dispose the discontinuing operation in piecemeal, then the

recoverable amount of individual assets are determined and compared with carrying

amount of individual assets.

If the enterprise abandons the discontinuing operation, the recoverable amount of

individual assets are determined and compared with carrying amount of individual assets.

AS-29

PROVISIONS, CONTIGENT LIABILITIES AND CONTINGENT ASSETS

PROVISION:

A provision is a liability which can be measured only by using a substantial degree of estimation.

Treatment : A provision should be recognized when:

(a) An enterprise has a present obligation as a result of past event

(b) It is probable that an outflow of resources embodying economic benefits will be

required to settle the obligation; and

(c) A reliable estimate can be made of the amount of the obligation.

Present Obligation: An obligation is a present obligation if, based on the evidence available, its

existence at the balance sheet date is considered Probable, i.e. more

likely than not.

Past Event: A Past event that leads to a present obligation is called an obligating event.

CONTINGENT LIABILITY:

1] A contingent liability is

A possible obligation that arises from past events

And; existence of which will be confirmed by the occurrence or non

occurrence of future events not wholly within the control of the enterprise

2] A contingent liability is

A present obligation that arises from past events

And; not recognized because of lower probability of outflow of resources or

non-availability of reliable estimate

Possible Obligation: An obligation is a present obligation if, based on the evidence

available, its existence at the balance sheet date is considered Not

Probable.

Treatment: An enterprise should not recognize a contingent liability.

It should be disclosed in financial statements unless the possibility of

outflow is remote.

CONTINGENT ASSETS:

A contingent assets is a possible asset that arises from past events of the existence of which

will be confirmed only by the occurrence or non-occurrence of one or more uncertain

future events not wholly within the control of the enterprise.

Treatment: An enterprise should not recognize a contingent asset.

An enterprise should not be disclosed in financial statements.

It may be disclosed in the report of approving authority, where an inflow

is probable.

Other Important Issues:

1. Provisioning is required for only those liabilities that exist at the balance sheet date.

( i.e. No provision is required for costs that need to be incurred to operate in

future.)

2. Where details of a proposed new law have yet to be finalized, an obligation arises

only when the legislation is virtually certain to be enacted. For example, huge

penalty shall be imposed on the enterprise if the proposed law is enacted. No

provisioning is required unless the virtual certainty of the enactment of the law is

established.

3. Where there are a number of similar obligations (e.g. product warranties) the

probability that an outflow will be required in settlement is determined by

considering the class of obligations as a whole.

4. If the reliable estimate of the liability cannot be made, it should be disclosed as a

contingent liability.

5. Where an enterprise is jointly & severally liable for an obligation:

Provision should be made for the portion on which enterprise has direct

liability.

The balance amount should be disclosed as contingent liability.

6. Gains from the expected disposal of assets should not be taken into account in

measuring a provision.

7. Reimbursement for expenditure of which provision is created, should be recognized

when and only when it is virtually certain that the reimbursement shall be received

on settlement of liability.

Such Reimbursement may be shown as a net figure in Profit & Loss statement but

should be presented in balance sheet as a separate asset (i.e. net provision not to be

shown)

8. A provision should be used only for expenditures for which the provision was

originally recognized. Provisions should also be reviewed at each balance sheet

date and if no longer required, it should be reversed.

9. Provision should not be recognized for future operating losses as it neither meets

the criteria of liability nor meets the criteria for recognition of provision.

RESTRUCTURING:

A restructuring is a program that is planned and controlled by management and materially

changes either:

(a) the scope of a business undertaken by an enterprise; or

(b) the manner in which the business is expected.

Restructuring may include the following:

(a) sale or termination of a line of business;

(b) the closure of business location in a region

(c) eliminating a layer of management;

Treatment: A provision for recognition criteria is recognized only when the recognition

criteria for provision is met.

A restructuring provision should include only the direct expenditures arising from the

restructuring, which are those that are both;

(a) necessarily entailed by the restructuring; and

(b) not associated with the ongoing activities of the enterprise.

Restructuring provision does not include costs like

(a) retraining or relocating continuing staff

(b) marketing expenses

(c) investments in new systems and distribution networks.

Identifiable future operating losses up to the date of a restructuring and gains on disposal

of assets (even if it is included as part of restructuring) are not included in provisions.

DISCLOSURES:

The enterprise should disclose for each class of provision:

(a) the carrying amount at the beginning & end of the period

(b) additional provision made during the period

(c) amount used during the period

(d) amount reversed during the period

(e) nature of obligation & and expected time of incurrence

(f) indication about the uncertainties attached to the provisions

The enterprise should disclose for each class of contingent liabilities:

(a) an estimate of its financial effects

(b) an indication of the uncertainties relating to any outflow

(c) the possibility of any reimbursement

Where any of the information required is not disclosed because it is not practicable to do

so, that fact should be stated.

In extremely rare cases, disclosures can be expected to seriously harm the enterprise in a

dispute with other parties. In such cases, instead of detailed information, general nature of

dispute together with the reason of non-disclosures should be disclosed.

Example 1: Warranties

A manufacturer gives warranties et the time of sale to purchasers of its product. Under the

terms of the contract for the manufacturer undertakes to make good, by repairs or

replacement, manufacturer defects that become apparent within three years from the date

of sale. On past experience, it is probable (i.e. more likely than not) that there will be some

claims under the warranties.

Present obligation as a result of a past obligating event- The obligating event is the sale of

the product with a warranty, which gives rise to an obligation

An outflow of resources embodying economics benefits in settlement- Probable for the

warranties as a whole.

Conclusion – A provision is recognized for the best estimate of the costs of making good

under the warranty products sold before the balance sheet date.

Example 2 Contaminated Land- Legislation Virtually Certain to be Enacted

An enterprise in the oil industry causes contaminated but does not clean up because there is

no legislation requiring cleaning up, and the enterprise has been contaminating land for

several years. At 31 March 2005 it is virtually certain that a law requiring a clean up of

land already contaminated will be enacted shortly after the year end.

Present obligation as a result of a past obligating event- he obligating event is the

contamination of the land because of the virtually certainty of legislation requiring

cleaning up.

An outflow of resources embodying economics benefits in settlement- Probable.

Conclusion - A provision is recognized for the best estimate of the costs of the clean up.

Example 3: Offshore Oilfield

An enterprise operates an offshore oilfield where its licensing agreement requires it to

remove the oil rig at the end of production and restore the seabed. Ninety percent of the

eventual cost related to the removal of the oil rig and restoration of damage caused by

building it, and ten percent arise through the extraction of oil. At the balance sheet date,

the rig has been constructed but no oil has been extracted.

Present obligation as a result of past obligating event- The construction of the oil rig

created an obligation under the terms of the license to remove the rig and restore the

seabed and is thus as obligating event. At the balance sheet date, however, there is no

obligation to rectify the damage that will be caused by extraction of oil.

An outflow of resources embodying economics benefits in settlement- Probable.

Conclusion- A provision is recognized for the best estimate of ninety percent of the eventual

costs that relate to the removal of the oil rig and restoration of damage caused by building

it. There coasts are included as part of the cost of the oil rig. The ten percent of costs that

arise through the extraction of oil are recognized as a liability when the oil is extracted.

Example 4: Refunds Policy

A retail store has a policy of refunding purchases by dissatisfied customers, even though it

is under no legal obligation to do so. Its policy of making refunds is generally known.

Present obligation as a result of a past obligating event- The obligating event is the sale of

the product, which gives rise to an obligation because obligating also arise from normal

business practice, custom and a desire to a maintain good business relations or act in an

equitable manner

Outflows of resources embodying economic benefit in settlement–Probable, a proportion of

goods are returned for refund.

Conclusion – A provision is recognized for the best estimate of the costs of refunds.

Example 5: Legal Requirement to Fit Smoke Filters

Under new legislation, an enterprise is required to fit smoke filters to its factories by 30

September 2005. The enterprise has not fitted the smoke filters.

(a) At the balance sheet date of 31 March 2005

Present obligation as a result of a past obligating event – There is no obligation because

there is no obligating event either for the costs of fitting smoke filters or for fines under the

legislation

Conclusion – No provision is recognized for the cost of fitting the smoke filters.

(b) At the balance sheet date of 31 March 2006

Present obligations as a result of a past obligating event – There is still no obligation for

the costs of fitting smoke filters because no obligating event has occurred (the fitting of the

filters). However, an obligation might arise to pay fines o penalties under the legislation

because the obligating event has occurred (the non-complaint operation of the factory).

An outflow of resources embodying economic benefits in settlement - Assessment of

probability of incurring fines and fines and penalties by non-compliant operation depends

on the details of the legislation and the stringency of the enforcement regime.

Conclusion – No provision is recognized for the costs of fitting smoke filters. However, a

provision is recognized for the best estimate of any fines and penalties that are more likely

than not to be imposed.

Example 6: Staff retraining as a Result of Changes in the Income Tax System

The government introduces a number of changes to the income tax system. As a result of

these changes, an enterprise in the financial services sector will need to retrain a large

proportion of its administrative and sales workforce in order to ensure continued

compliance with financial services regulation. At the balance sheet date, no retraining of

staff has taken place.

Present obligation as a result of past obligating event – There is no obligation because no

obligating event (retraining) has taken place.

Conclusion – No provision is recognized.

Example 7: A Single Guarantee

During 2004-05, Enterprise A gives a guarantee of certain borrowing of Enterprise B,

whose financial condition at that time is sound. During 2005-06, the financial condition of

Enterprise B deteriorates and at 30 September 2005 Enterprise B goes into liquidation..

(a) At 31 March 2005

Present obligation as a result of a past obligating event – The obligating event is the giving

of the guarantee, which gives rise to an obligation.

An outflow of resources embodying economics benefits in settlement – No outflow of

benefits is probable at 31 March 2005.

Conclusion – No provision is recognized. The guarantee is disclosed as a contingent

liability unless the probability of any is regarded as remote At 31 March 2006

Present obligation as a result of a past obligating event – The obligating event is the giving

of the guarantee, which gives to a legal obligation.

Conclusion – A provision is recognized for the best estimate of the obligation.

Note: This example deals with a single guarantee. If an enterprise has a portfolio of similar

guarantee, it will assess that portfolio as a whole in determining whether an outflow of

resources embodying economic benefit is probable. Where an enterprise gives guarantees

in exchange for a fee, revenue is recognized under AS 9, Revenue recognition.

Example 8: A Court Case

After a wedding in 2004-05, ten people died, possibly as a result of food poisoning from

products sold by the enterprise. Legal proceedings are started seeking damages from the

enterprise but it disputes liability. Up to the date of approval of the financial statements for

the year 31 March 2005, the enterprise‟s lawyers advice that it is probable that the

enterprise will not be found liable. However, when the enterprise prepares the financial

statements for the year 31 March 2006, its lawyer‟s advice that, owing to developments in

the case, it is probable that the enterprise will be found liable.

(a) At 31 March 2005

Present obligation as a result of a past obligating event – On the basis of the evidence

available when the financial statements were approved, there is no present obligation as a

result of past events.

Conclusion – No provision is recognized. The matter is disclosed as a contingent liability

unless the probability of any outflow is regarded as remote

(b) At 31 March 2006

Present obligation as a result of a past obligating event – On the basis of the evidence

available, there is a present obligation.

An outflow of resources embodying economic benefits in settlement – Probable.

Conclusion – A provision is recognized for the best estimate of the amount to settle the

obligation.

Example 9A: Refurbishment Costs – No Legislative Requirement

A furnace has a lining that needs to be replaced every five years for technical reasons. At

the balance sheet date, the lining has been in use for three years.

Present obligation as a result of a past obligating event- There is no present obligation.

Conclusion – No provision is recognized.

The cost of replacing the lining is not recognized because, at the balance sheet date, no

obligation to replace the lining exits independently of the company‟s future actions – even

the intention to incur the expenditure depends on the company deciding to continue

operating the furnace or to replace the lining.

Example 9B: Refurbishment Costs – Legislative Requirement

An Airline is required by law to overhaul its aircraft once every three years.

Present obligation as a result of a past obligating event – There is no present obligation.

Conclusion – No provision is recognized.

The costs of overhauling aircraft are not recognized as a provision for the same reason as

the cost of replacing the lining is not recognized as a provision in example 9A. Even a legal

requirement to overhaul does not make the cost of the overhaul a liability, because no

obligation exits to overhaul the aircraft independently of the enterprise‟s future actions –

the enterprise could avoid the future expenditure by its future actions, for example by

selling the aircraft.

Accounting Standard 1: Disclosure of Accounting Policies

Significant Accounting Policies followed in preparation and

presentation of financial statements should form part thereof and be

disclosed at one place in the financial statements.

Any change in the accounting policies having a material effect in the

current period or future periods should be disclosed. The amount by which

any item in financial statements is affected by such change should be

disclosed to the extent ascertainable. If the amount is not ascertainable

the fact should be indicated.

If fundamental assumptions (going concern, consistency and accrual)

are not followed, fact to be disclosed.

Major considerations governing selection and application of accounting

policies are i) Prudence, ii) Substance over form and iii) Materiality.

The ICAI has made an announcement that till the issuance of

Accounting Standards on (i) Financial Instruments : Presentation, (ii)

Financial Instruments : Disclosures and (iii) Financial Instruments :

Recognition and Measurement, an enterprise should provide information

regarding the extent of risks to which an enterprise is exposed and as a

minimum, make following disclosures in its financial statements:

a. category-wise quantitative data about derivative instruments that

are outstanding at the balance sheet date,

b. the purpose, viz. hedging or speculation, for which such derivative

instruments have been acquired, and

c. the foreign currency exposures that are not hedged by a derivative

instrument or otherwise.

This announcement is applicable in respect of financial statements for the

accounting period(s) ending on or after March 31, 2006.

Accounting Standard 2: Valuation of Inventories

This standard should be applied in accounting for inventories other than

WIP arising under construction contracts, WIP of service providers, shares,

debentures and financial instruments held as stock in trade, producers’

inventories of livestock, agricultural and forest products and mineral oils,

ores and gases to the extent measured at net realisable value in

accordance with well established practices in those industries.

Inventories are assets held for sale in ordinary course of business, in

the process of production of such sale, or in form of materials to be

consumed in production process or rendering of services.

Inventories do not include machinery spares which can be used with an

item of fixed asset and whose use is irregular.

Net realisable value is the estimated selling price less the estimated

costs of completion and estimated costs necessary to make the sale.

fb.com//SUPERWHIZZ4U CACWACS.WORDPRESS.COM

Cost of inventories should comprise all costs incurred for bringing the

inventories to their present location and condition.

Inventories should be valued at lower of cost and net realisable value.

Generally, weighted average cost or FIFO method is used in cases where

goods are ordinarily interchangeable.

Specific Identification Method to be used when goods are not ordinarily

interchangeable or have been segregated for specific projects.

Disclose the accounting policies adopted including the cost formula

used, total carrying amount of inventories and its classification.

Also refer ASI 2 – deals with accounting of machinery spares

Accounting Standard 3: Cash Flow Statements

Prepare and present a cash flow statement for each period for which

financial statements are prepared.

A cash flow statement should report cash flows during the period

classified by operating, investing and financial activities.

Operating activities are the principal revenue producing activities of the

enterprise other than investing or financing activities.

Investing activities are the acquisition and disposal of long term assets

and other investments not included in cash equivalents.

Financing activities are activities that result in changes in the size and

composition of the owner’s capital and borrowings of the enterprise.

A cash flow statement for operating activities should be prepared by

using either the direct method or the indirect method. For investing and

financing activities cash flows should be prepared using the direct method.

Cash flows arising from transactions in a foreign currency should be

recorded in enterprise’s reporting currency by applying the exchange rate

at the date of the cash flow.

Investing and financing transactions that do not require the use of cash

and cash equivalent balances should be excluded.

An enterprise should disclose the components of cash and cash

equivalents together with reconciliation of amounts as disclosed to

amounts reported in the balance sheet.

An enterprise should disclose together with a commentary by the

management the amount of significant cash and cash equivalent balances

held by it that are not available for use.

Accounting Standard 4: Contingencies and Events Occurring after the Balance

Sheet Date

A contingency is a condition or situation the ultimate outcome of which

will be known or determined only on the occurrence or non-occurrence of

uncertain future event/s.

fb.com//SUPERWHIZZ4U CACWACS.WORDPRESS.COM

Events occurring after the balance sheet date are those significant

events both favourable and unfavourable that occur between the balance

sheet date and the date on which the financial statements are approved.

Amount of a contingent loss should be provided for by a charge in P & L

A/c if it is probable that future events will confirm that an asset has been

impaired or a liability has been incurred as at the balance sheet date and a

reasonable estimate of the amount of the loss can be made.

Existence of contingent loss should be disclosed if above conditions are

not met, unless the possibility of loss is remote.

Contingent Gains if any, not to be recognised in the financial

statements.

Material change in the position due to subsequent events be accounted

or disclosed.

Proposed or declared dividend for the period should be adjusted.

Material event occurring after balance sheet date affecting the going

concern assumption and financial position be appropriately dealt with in

the accounts.

Contingencies or events occurring after the balance sheet date and the

estimate of the financial effect of the same should be disclosed.

Note: The underlined paras/words have been withdrawn on issuance of AS

29 effective for accounting periods commencing on or after 1-4-2004.

Accounting Standard 5: Net Profit/Loss for the Period, Prior Period Items and

Changes in Accounting Policies

All items of income and expense, which are recognised in a period,

should be included in determination of net profit or loss for the period

unless an accounting standard requires or permits otherwise.

Prior period, extraordinary items be separately disclosed in a manner

that their impact on current profit or loss can be perceived. Nature and

amount of significant items be provided. Extraordinary items should be

disclosed as a part of profit or loss for the period.

Effect of a change in the accounting estimate should be included in the

determination of net profit or loss in the period of change and also future

periods if it is expected to affect future periods.

Change in accounting policy, which has a material effect, should be

disclosed. Impact and the adjustment arising out of material change

should be disclosed in the period in which change is made. If the change

does not have a material impact in the current period but is expected to

have a material effect in future periods then the fact should be disclosed.

fb.com//SUPERWHIZZ4U CACWACS.WORDPRESS.COM

Accounting policy may be changed only if required by the statute or for

compliance with an accounting standard or if the change would result in

appropriate presentation of the financial statements.

A change in accounting policy on the adoption of an accounting

standard should be accounted for in accordance with the specific

transitional provisions, if any, contained in that accounting standard.

Accounting Standard 6: Depreciation Accounting

Standard does not apply to depreciation in respect of forests,

plantations and similar regenerative natural resources, wasting assets

including expenditure on exploration and extraction of minerals, oils,

natural gas and similar non-regenerative resources, expenditure on

research and development, goodwill and livestock. Special considerations

apply to these assets.

Allocate depreciable amount of a depreciable asset on systematic basis

to each accounting year over useful life of asset.

Useful life may be reviewed periodically after taking into consideration

the expected physical wear and tear, obsolescence and legal or other

limits on the use of the asset.

Basis for providing depreciation must be consistently followed and

disclosed. Any change to be quantified and disclosed.

A change in method of depreciation be made only if required by statute,

for compliance with an accounting standard or for appropriate

presentation of the financial statements. Revision in method of

depreciation be made from date of use. Change in method of charging

depreciation is a change in accounting policy and be quantified and

disclosed.

In cases of addition or extension which becomes integral part of the

existing asset depreciation to be provided on adjusted figure prospectively

over the residual useful life of the asset or at the rate applicable to the

asset.

Where the historical cost undergoes a change due to fluctuation in

exchange rate, price adjustment etc. depreciation on the revised

unamortised amount should be provided over the balance useful life of the

asset.

On revaluation of asset depreciation should be based on revalued

amount over balance useful life. Material impact on depreciation should be

disclosed.

Deficiency or surplus in case of disposal, destruction, demolition etc. be

disclosed separately, if material.

Historical cost, amount substituted for historical cost, depreciation for

the year and accumulated depreciation should be disclosed.

fb.com//SUPERWHIZZ4U CACWACS.WORDPRESS.COM

Depreciation method used should be disclosed. If rates applied are

different from the rates specified in the governing statute then the rates

and the useful life be also disclosed.

Accounting Standard 7 : Accounting for Construction Contracts (Revised 2002)

Applicable to accounting for construction contract.

Construction contract may be for construction of a single/combination

of interrelated or interdependent assets.

A fixed price contract is a contract where contract price is fixed or per

unit rate is fixed and in some cases subject to escalation clause.

A cost plus contract is a contract in which contractor is reimbursed for

allowable or defined cost plus percentage of these cost or a fixed fee.

In a contract covering a number of assets, each asset is treated as a

separate construction contract when there are:

separate proposal;

subject to separate negotiations and the contractor and

customer is able to accept/reject that part of the contract;

identifiable cost and revenues of each asset

A group of contracts to be treated as a single construction contract

when

they are negotiated as a single package;

contracts are closely interrelated with an overall profit margin;

and

contracts are performed concurrently or in a continuous

sequence.

Additional asset construction to be treated as separate construction

contract when

assets differs significantly in design/technology/function from

original contract assets.

a price negotiated without regard to original contract price

Contract revenue comprises of

initial amount and

variations in contract work, claims and incentive payments that

will probably result in revenue and are capable of being reliably

measured.

Contract cost comprises of

costs directly relating to specific contract

costs attributable and allocable to contract activity

fb.com//SUPERWHIZZ4U CACWACS.WORDPRESS.COM

other costs specifically chargeable to customer under the terms

of contracts.

Contract Revenue and Expenses to be recognised, when outcome can

be estimated reliably up to stage of completion on reporting date.

In Fixed Price Contract outcome can be estimated reliably when

total contract revenue can be measured reliably.

it is probable that economic benefits will flow to the enterprise;

contract cost and stage of completion can be measured reliably

at reporting date; and

contract costs are clearly identified and measured reliably for

comparing actual costs with prior estimates.

In cost plus contract outcome is estimated reliably when

it is probable that economic benefits will flow to the enterprise;

and

contract cost whether reimbursable or not can be clearly

identified and measured reliably.

When outcome of a contract cannot be estimated reliably

revenue to the extent of which recovery of contract cost is

probable should be recognised;

contract cost should be recognised as an expense in the period

in which they are incurred; and

An expected loss should be recognised as expense.

When uncertainties no longer exist revenue and expenses to be

recognised as mentioned above when outcomes can be estimated reliably.

When it is probable that contract costs will exceed total contract

revenue, the expected loss should be recognised as an expense

immediately.

Change in estimate to be accounted for as per AS 5.

An enterprise to disclose

contract revenue recognised in the period.

method used to determine recognised contract revenue.

methods used to determine the stage of completion of contracts

in progress.

For contracts in progress an enterprise should disclose

the aggregate amount of costs incurred and recognised profits

(less recognised losses) up to the reporting date.

amount of advances received and

fb.com//SUPERWHIZZ4U CACWACS.WORDPRESS.COM

amount of retention.

An enterprise should present

gross amount due from customers for contract work as an asset

and

the gross amount due to customers for contract work as a

liability.

Accounting Standard 8: Accounting for Research and Development

Note: In view of operation of AS 26, this Standard stands withdrawn.

Accounting Standard 9: Revenue Recognition

Standard does not deal with revenue recognition aspects of revenue

arising from construction contracts, hire-purchase and lease agreements,

government grants and other similar subsidies and revenue of insurance

companies from insurance contracts. Special considerations apply to these

cases.

Revenue from sales and services should be recognised at the time of sale

of goods or rendering of services if collection is reasonably certain; i.e.,

when risks and rewards of ownership are transferred to the buyer and when

effective control of the seller as the owner is lost.

In case of rendering of services, revenue must be recognised either on

completed service method or proportionate completion method by relating

the revenue with work accomplished and certainty of consideration

receivable.

Interest is recognised on time basis, royalties on accrual and dividend

when owner’s right to receive payment is established.

Disclose circumstances in which revenue recognition has been postponed

pending significant uncertainties.

Also refer ASI 14 (withdrawing GC 3/2002) deals with the manner of disclosure of

excise duty in presentation of revenue from sales transactions (turnover).

Accounting Standard 10: Accounting for Fixed Assets

Fixed asset is an asset held for producing or providing goods and/or

services and is not held for sale in the normal course of the business.

Cost to include purchase price and attributable costs of bringing asset to

its working condition for the intended use. It includes financing cost for

period up to the date of readiness for use.

Self-constructed assets are to be capitalised at costs that are specifically

related to the asset and those which are allocable to the specific asset.

Fixed asset acquired in exchange or part exchange should be recorded at

fair market value or net book value of asset given up adjusted for balancing

payment, cash receipt etc. Fair market value is determined with reference

to asset given up or asset acquired.

fb.com//SUPERWHIZZ4U CACWACS.WORDPRESS.COM

Revaluation, if any, should be of class of assets and not an individual asset.

Basis of revaluation should be disclosed.

Increase in value on revaluation be credited to Revaluation Reserve while

the decrease should be charged to P & L A/c.

Goodwill should be accounted only when paid for.

Assets acquired on hire purchase be recorded at cash value to be shown

with appropriate note about ownership of the same. (Not applicable for

assets acquired after 1st April, 2001 in view of AS 19 – Leases becoming

effective).

Gross and net book values at beginning and end of year showing additions,

deletions and other movements, expenditure incurred in course of

construction and revalued amount if any be disclosed.

Assets should be eliminated from books on disposal/when of no utility

value.

Profit/Loss on disposal be recognised on disposal to P & L statement.

Also refer ASI 2 which deals with accounting for machinery spares.

Accounting Standard 11: The Effects of Changes in Foreign Exchange Rates

(Revised 2003)

The Statement is applied in accounting for transactions in foreign currency

and translating financial statements of foreign operations. It also deals

with accounting of forward exchange contract.

Initial recognition of a foreign currency transaction shall be by applying the

foreign currency exchange rate as on the date of transaction. In case of

voluminous transactions a weekly or a monthly average rate is permitted, if

fluctuation during the period is not significant.

At each Balance Sheet date foreign currency monetary items such as cash,

receivables, payables shall be reported at the closing exchange rates

unless there are restrictions on remittances or it is not possible to effect

an exchange of currency at that rate. In the latter case it should be

accounted at realisable rate in reporting currency. Non monetary items

such as fixed assets, investment in equity shares which are carried at

historical cost shall be reported at the exchange rate on the date of

transaction. Non monetary items which are carried at fair value shall be

reported at the exchange rate that existed when the value was determined.

Note: Schedule VI to the Companies Act, 1956, provides that any increase

or reduction in liability on account of an asset acquired from outside India

in consequence of a change in the rate of exchange, the amount of such

increase or decrease, should added to, or, as the case may be, deducted

from the cost of the fixed asset.

fb.com//SUPERWHIZZ4U CACWACS.WORDPRESS.COM

Therefore, for fixed assets, the treatment described in Schedule VI will be

in compliance with this standard, instead of stating it at historical cost.

Exchange differences arising on the settlement of monetary items or on

restatement of monetary items on each balance sheet date shall be

recognised as expense or income in the period in which they arise.

Exchange differences arising on monetary item which in substance, is net

investment in a non integral foreign operation (long term loans) shall be

credited to foreign currency translation reserve and shall be recognised as

income or expense at the time of disposal of net investment.

The financial statements of an integral foreign operation shall be

translated as if the transactions of the foreign operation had been those of

the reporting enterprise; i.e., it is initially to be accounted at the exchange

rate prevailing on the date of transaction.

For incorporation of non integral foreign operation, both monetary and non

monetary assets and liabilities should be translated at the closing rate as

on the balance sheet date. The income and expenses should be translated

at the exchange rates at the date of transactions. The resulting exchange

differences should be accumulated in the foreign currency translation

reserve until the disposal of net investment. Any goodwill or capital reserve

on acquisition on non-integral financial operation is translated at the

closing rate.

In Consolidated Financial Statement (CFS) of the reporting enterprise,

exchange difference arising on intra group monetary items continues to be

recognised as income or expense, unless the same is in substance an

enterprise’s net investment in non integral foreign operation.

When the financial statements of non integral foreign operations of a

different date are used for CFS of the reporting enterprise, the assets and

liabilities are translated at the exchange rate prevailing on the balance

sheet date of the non integral foreign operations. Further adjustments are

to be made for significant movements in exchange rates upto the balance

sheet date of the reporting currency.

When there is a change in the classification of a foreign operation from

integral to non integral or vice versa the translation procedures applicable

to the revised classification should be applied from the date of

reclassification.

Exchange differences arising on translation shall be considered for

deferred tax in accordance with AS 22.

Forward Exchange Contract may be entered to establish the amount of the

reporting currency required or available at the settlement date of the

transaction or intended for trading or speculation. Where the contracts are

not intended for trading or speculation purposes the premium or discount

arising at the time of inception of the forward contract should be amortized

as expense or income over the life of the contract. Further, exchange

fb.com//SUPERWHIZZ4U CACWACS.WORDPRESS.COM

differences on such contracts should be recognised in the P & L A/c in the

reporting period in which there is change in the exchange rates. Exchange

difference on forward exchange contract is the difference between

exchange rate at the reporting date and exchange difference at the date of

inception of the contract for the underlying currency.

Profit or loss arising on the renewal or cancellation of the forward contract

should be recognised as income or expense for the period. A gain or loss on

forward exchange contract intended for trading or speculation should be

recognised in the profit and loss statement for the period. Such gain or loss

should be computed with reference to the difference between forward rate

on the reporting date for the remaining maturity period of the contract and

the contracted forward rate. This means that the forward contract is

marked to market. For such contract, premium or discount is not

recognised separately.

Disclosure to be made for:

o Amount of exchange difference included in Profit and Loss

statement.

o Net exchange difference accumulated in Foreign Currency

Translation Reserve.

o In case of reclassification of significant foreign operation, the

nature of the change, the reasons for the same and its impact on

the shareholders fund and the impact on the Net Profit and Loss for

each period presented.

Non mandatory Disclosures can be made for foreign currency risk

management policy.

Accounting Standard 12: Accounting for Government Grants

Grants can be in cash or in kind and may carry certain conditions to be

complied.

Grants should not be recognised unless reasonably assured to be realized

and the enterprise complies with the conditions attached to the grant.

Grants towards specific assets should be deducted from its gross value.

Alternatively, it can be treated as deferred income in P & L A/c on rational

basis over the useful life of the depreciable asset. Grants related to non-

depreciable asset should be generally credited to Capital Reserves unless

it stipulates fulfilment of certain obligations. In the latter case the grant

should be credited to the P & L A/c over a reasonable period. The deferred

income balance to be shown separately in the financial statements.

Grants of revenue nature to be recognised in the P & L A/c over the period

to match with the related cost, which are intended to be compensated.

Such grants can be treated as other income or can be reduced from related

expense.

fb.com//SUPERWHIZZ4U CACWACS.WORDPRESS.COM

Grants by way of promoter’s contribution is to be credited to Capital

Reserves and considered as part of shareholder’s funds.

Grants in the form of non-monetary assets, given at concessional rate,

shall be accounted at their acquisition cost. Asset given free of cost be

recorded at nominal value.

Grants receivable as compensation for losses/expenses incurred should be

recognised and disclosed in P & L A/c in the year it is receivable and

shown as extraordinary item, if material in amount.

Grants when become refundable, be shown as extraordinary item.

Revenue grants when refundable should be first adjusted against

unamortised deferred credit balance of the grant and the balance should

be charged to the P & L A/c.

Grants against specific assets on becoming refundable are recorded by

increasing the value of the respective asset or by reducing Capital Reserve

/ Deferred income balance of the grant, as applicable. Any such increase in

the value of the asset shall be depreciated prospectively over the residual

useful life of the asset.

Accounting policy adopted for grants including method of presentation,

extent of recognition in financial statements, accounting of non-monetary

assets given at concession/ free of cost be disclosed.

Accounting Standard 13: Accounting for Investments

Current investments and long term investments be disclosed distinctly with

further sub-classification into government or trust securities, shares,

debentures or bonds, investment properties, others unless it is required to

be classified in other manner as per the statute governing the enterprise.

Cost of investment to include acquisition charges including brokerage,

fees and duties.

Investment properties should be accounted as long term investments.

Current investments be carried at lower of cost and fair value either on

individual investment basis or by category of investment but not on global

basis.

Long term investments be carried at cost. Provision for decline (other than

temporary) to be made for each investment individually.

If an investment is acquired by issue of shares/securities or in exchange of

an asset, the cost of the investment is the fair value of the securities

issued or the assets given up. Acquisition cost may be determined

considering the fair value of the investments acquired.

Changes in the carrying amount and the difference between the carrying

amount and the net proceeds on disposal be charged or credited to the P &

L A/c.

fb.com//SUPERWHIZZ4U CACWACS.WORDPRESS.COM

Disclosure is required for the accounting policy adopted, classification of

investments; profit / loss on disposal and changes in carrying amount of

such investment.

Significant restrictions on right of ownership, realisability of investments

and remittance of income and proceeds of disposal thereof be disclosed.

Disclosure should be made of aggregate amount of quoted and unquoted

investments together with aggregate value of quoted investments.

Accounting Standard 14: Accounting for Amalgamations

Amalgamation in nature of merger be accounted for under Pooling of

Interest Method and in nature of purchase be accounted for under

Purchase Method.

Under the Pooling of the Interest Method, assets, liabilities and reserves of

the transferor company be recorded at existing carrying amount and in the

same form as it was appearing in the books of the transferor.

In case of conflicting accounting policies, a uniform policy be adopted on

amalgamation. Effect on financial statement of such change in policy be

reported as per AS5.

Difference between the amount recorded as share capital issued and the

amount of capital of the transferor company should be adjusted in

reserves.

Under Purchase Method, all assets and liabilities of the transferor

company be recorded at existing carrying amount or consideration be

allocated to individual identifiable assets and liabilities on basis of fair

values at date of amalgamation. The reserves of the transferor company

shall lose its identity. The excess or shortfall of consideration over value of

net assets be recognised as goodwill or capital reserve.

Any non-cash item included in the consideration on amalgamation should

be accounted at fair value.

In case the scheme of amalgamation sanctioned under the statute

prescribes a treatment to be given to the transferor company reserves on

amalgamation, same should be followed. However a description of

accounting treatment given to reserves and the reasons for following a

treatment different from that prescribed in the AS is to be given. Also

deviations between the two accounting treatments given to the reserves

and the financial effect, if any, arising due to such deviation is to be

disclosed. (Limited Revision to AS 14 w.e.f 1-4-2004)

Disclosures to include effective date of amalgamation for accounting, the

method of accounting followed, particulars of the scheme sanctioned.

In case of amalgamation under the Pooling of Interest Method the

treatment given to the difference between the consideration and the value

of the net identified assets acquired is to be disclosed. In case of

amalgamation under the Purchase Method the consideration and the

fb.com//SUPERWHIZZ4U CACWACS.WORDPRESS.COM

treatment given to the difference compared to the value of the net

identifiable assets acquired including period of amortization of goodwill

arising on amalgamation is to be disclosed.

Accounting Standard 15: Accounting for Retirement Benefits in the Financial

Statement of Employers

For retirement benefits of provident fund and other defined contribution

schemes, contribution payable by employer and any shortfall on collection

from employees if any for a year be charged to P & L A/c. Excess payment

be treated as pre-payment.

For gratuity and other defined benefit schemes, accounting treatment will

depend on the type of arrangements, which the employer has entered into.

If payment for retirement benefits out of employers funds, appropriate

charge to P & L to be made through a provision for accruing liability,

calculated according to actuarial valuation.

If liability for retirement benefit funded through creation of trust, cost

incurred be determined actuarially. Excess/ shortfall of contribution paid

against amount required to meet accrued liability as certified by actuary be

treated as pre-payment or charged to P & L account

If liability for retirement benefit is funded through a scheme administered

by an insurer, an actuarial certificate or confirmation from insurer to be

obtained. The excess/ shortfall of the contribution paid against the amount

required to meet accrued liability as certified by actuary or confirmed by

insurer should be treated as pre-payment or charged to P & L account.

Any alteration in the retirement benefit cost should be charged or credited

to P & L A/c and change in actuarial method should be disclosed as per AS

5.

Financial statements to disclose method by which retirement benefit cost

have been determined.

Accounting Standard – 15 - Employee Benefits – Effective from accounting period

commencing on or after 1 April, 2006.

Applicable to Level II & III enterprises (subject to certain relaxation

provided), if number of persons employed is 50 or more.

For Enterprises employing less than 50 persons, any method of accrual

for accounting long-term employee benefits liability is allowed.

Employee benefits are all forms of consideration given in exchange of

services rendered by employees. Employee benefits include those provided

under formal plan or as per informal practices which give rise to an

obligation or required as per legislative requirements. These include

performance bonus (payable within 12 months) and non-monetary benefits

such as housing, car or subsidized goods or services to current employees,

post-employment benefits, deferred compensation and termination

fb.com//SUPERWHIZZ4U CACWACS.WORDPRESS.COM

benefits. Benefits provided to employees’ spouses, children, dependents,

nominees are also covered.

Short-term employee benefits should be recognised as an expense

without discounting, unless permitted by other AS to be included as a cost

of an asset.

Cost of accumulating compensated absences is accounted on accrual

basis and cost of non-accumulating compensated absences is accounted

when the absences occur.

Cost of profit sharing and bonus plans are accounted as an expense

when the enterprise has a present obligation to make such payments as a

result of past events and a reliable estimate of the obligation can be made.

While estimating, probability of payment at a future date is also

considered.

Post employment benefits can either be defined contribution plans,

under which enterprise’s obligation is limited to contribution agreed to be

made and investment returns arising from such contribution, or defined

benefit plans under which the enterprise’s obligation is to provide the

agreed benefits. Under the later plans if actuarial or investment experience

are worse then expected, obligation of the enterprise may get increased at

subsequent dates.

In case of a multi-employer plans, an enterprise should recognise its

proportionate share of the obligation. If defined benefit cost can not be

reliably estimated it should recognise cost as if it were a defined

contribution plan, with certain disclosures (in para 30)

State Plans and Insured Benefits are generally Defined Contribution

Plan.

Cost of Defined contribution plan should be accounted as an expense

on accrual basis. In case contribution does not fall due within 12 months

from the balance sheet date, expense should be recognised for discounted

liabilities.

The obligation that arises from the enterprise’s informal practices

should also be accounted with its obligation under the formal defined

benefit plan.

For balance sheet purpose, the amount to be recognised as a defined

benefit liability is the present value of the defined benefit obligation

reduced by (a) past service cost not recognised and (b) the fair value of the

plan asset. An enterprise should determine the present value of defined

benefit obligations (through actuarial valuation at intervals not exceeding

three years) and the fair value of plan assets (on each balance sheet date)

so that amount recognised in the financial statements do not differ

materially from the liability required. In case of fair value of plan asset is

higher than liability required, the present value of excess should be treated

as an asset.

fb.com//SUPERWHIZZ4U CACWACS.WORDPRESS.COM

For determining Cost to be recognised in the profit and loss account for

the Defined benefit plan, following should be considered :

Current service cost

Interest cost

Expected return of any plan assets

Actuarial gains and losses

Past service cost

Effect of any curtailment or settlement

Surplus arising out of present value of plan asset being higher

than obligation under the plan.

Actuarial Assumptions comprise of following :

Mortality during and after employment

Employee Turnover

Plan members eligible for benefits

Claim rate under medical plans

The discount rate, based on market yields on Government bonds

of relevant maturity.

Future salary and benefits levels

In case of medical benefits, future medical costs (including

administration cost, if material)

Rate of return expectation on plan assets.

Actuarial gains / losses should be recognised in profit and loss account as

income / expenses.

o Past Service Cost arises due to introduction or changes in the defined

benefit plan. It should be recognised in the profit and loss account over the

period of vesting. Similarly, surplus on curtailment is recognised over the

vesting period. However, for other long – term employee benefits, past

service cost is recognised immediately.

o The expected return on plan assets is a component of current service cost.

The difference between expected return and the actual return on plan

assets is treated as an actuarial gain / loss, which is also recognised in the

profit and loss account.

o An enterprise should disclose information by which users can evaluate the

nature of its defined benefit plans and the financial effects of changes in

those plans during the period. For disclosures requirement refer to para

120 to 125 of the standard.

o Termination benefits are accounted as a liability and expense only when

the enterprise has a present obligation as a result of a past event, outflow

fb.com//SUPERWHIZZ4U CACWACS.WORDPRESS.COM

of resources will be required to settle the obligation and a reliable estimate

of it can be made. Where termination benefits fall due beyond 12 months

period, the present value of liability needs to be worked out using the

discount rate. If termination benefit amount is material, it should be

disclosed separately as per AS – 5 requirements. As per the transitional

provisions expenses on termination benefits incurred up to 31 March,

2009 can be deferred over the pay-back period, not beyond 1 April, 2010.

o Transitional Provisions

When enterprise adopts the revised standard for the first time, additional

charge on account of change in a liability, compared to pre-revised AS –

15, should be adjusted against revenue reserves and surplus.

Accounting Standard 16: Borrowing Costs

Statement to be applied in accounting for borrowing costs.

Statement does not deal with the actual or imputed cost of owner’s

equity/preference capital.

Borrowing costs that are directly attributable to the acquisition,

construction or production of any qualifying asset (assets that takes a

substantial period of time to get ready for its intended use or sale. should

be capitalized.) Generally, a period of 12 months is considered as a

substantial period of time (ASI-1).

Income on the temporary investment of the borrowed funds be deducted

from borrowing costs.

In case of funds obtained generally and used for obtaining a qualifying

asset, the borrowing cost to be capitalized is determined by applying

weighted average of borrowing cost on outstanding borrowings, other than

borrowings for obtaining qualifying asset.

Capitalization of borrowing costs should be suspended during extended

periods in which development is interrupted. When the expected cost of

the qualifying asset exceeds its recoverable amount or Net Realizable

Value, the carrying amount is written down.

Capitalization should cease when activity is completed substantially or if

completed in parts, in respect of that part, all the activities for its intended

use or sale are complete.

Financial statements to disclose accounting policy adopted for borrowing

cost and also the amount of borrowing costs capitalized during the period.

In case exchange difference on foreign currency borrowings represent

saving in interest, compared to interest rate for the local currency

borrowings, it should be treated as part of interest cost for AS 16 (ASI-10).

Accounting Standard 17: Segment Reporting

Requires reporting of financial information about different types of

products and services an enterprise provides and different geographical

areas in which it operates.

fb.com//SUPERWHIZZ4U CACWACS.WORDPRESS.COM

A business segment is a distinguishable component of an enterprise

providing a product or service or group of products or services that is

subject to risks and returns that are different from other business

segments.

A geographical segment is distinguishable component of an enterprise

providing products or services in a particular economic environment that is

subject to risks and returns that are different from components operating

in other economic environments.

Internal organizational management structure, internal financial reporting

system is normally the basis for identifying the segments.

The dominant source and nature of risk and returns of an enterprise should

govern whether its primary reporting format will be business segments or

geographical segments.

A business segment or geographical segment is a reportable segment if (a)

revenue from sales to external customers and from transactions with other

segments exceeds 10% of total revenues (external and internal) of all

segments; or (b) segment result, whether profit or loss, is 10% or more of (i)

combined result of all segments in profit or (ii) combined result of all

segments in loss whichever is greater in absolute amount; or (c) segment

assets are 10% or more of all the assets of all the segments. If there is

reportable segment in the preceding period (as per criteria), same shall be

considered as reportable segment in the current year.

If total external revenue attributable to reportable segment constitutes

less than 75% of total revenues then additional segments should be

identified, for reporting.

Under primary reporting format for each reportable segment the enterprise

should disclose external and internal segment revenue, segment result,

amount of segment assets and liabilities, cost of fixed assets acquired,

depreciation, amortization of assets and other non cash expenses.

Interest expense (on operating liabilities) identified to a particular segment

(not of a financial nature) will not be included as part of segment expense.

However, interest included in the cost of inventories (as per AS 16) is to be

considered as a segment expense (ASI-22).

Reconciliation between information about reportable segments and

information in financial statements of the enterprise is also to be provided.

Secondary segment information is also required to be disclosed. This

includes information about revenues, assets and cost of fixed assets

acquired.

When primary format is based on geographical segments, certain further

disclosures are required.

Disclosures are also required relating to intra-segment transfers and

composition of the segment.

AS disclosure is not required, if more than one business or geographical

segment is not identified (ASI-20).

Accounting Standard 18: Related Party Disclosures

fb.com//SUPERWHIZZ4U CACWACS.WORDPRESS.COM

Applicability of AS 18 has been restricted to enterprises whose debt or

equity securities are listed in any stock exchange in India or are in the

process of listing and all commercial enterprises whose turnover for the

accounting period exceeds Rs 50 crores.

The statement deals with following related party relationships: (i)

Enterprises that directly or indirectly control (through subsidiaries) or are

controlled by or are under common control with the reporting enterprise;

(ii) Associates, Joint Ventures of the reporting entity; Investing party or

venturer in respect of which reporting enterprise is an associate or a joint

venture; (iii) Individuals owning voting power giving control or significant

influence; (iv) Key management personnel and their relatives; and (v)

Enterprises over which any of the persons in (iii) or (iv) are able to exercise

significant influence. Remuneration paid to key management personnel

falls under the definition of a related party transaction (ASI-23).

Parties are considered related if one party has ability to control or exercise

significant influence over the other party in making financial and/or

operating decisions.

Following are not considered related parties: (i) Two companies merely

because of common director, (ii) Customer, supplier, franchiser, distributor

or general agent merely by virtue of economic dependence; and (iii)

Financiers, trade unions, public utilities, government departments and

bodies merely by virtue of their normal dealings with the enterprise.

Disclosure under the standard is not required in the following cases (i) If

such disclosure conflicts with duty of confidentially under statute, duty

cast by a regulator or a component authority; (ii) In consolidated financial

statements in respect of intra-group transactions; and (iii) In case of state-

controlled enterprises regarding related party relationships and

transactions with other state-controlled enterprises.

Relative (of an individual) means spouse, son, daughter, brother, sister,

father and mother who may be expected to influence, or be influenced by,

that individual in dealings with the reporting entity.

Standard also defines inter alia control, significant influence, associate,

joint venture, and key management personnel.

Where there are transactions between the related parties following

information is to be disclosed: name of the related party, nature of

relationship, nature of transaction and its volume (as an amount or

proportion), other elements of transaction if necessary for understanding,

amount or appropriate proportion outstanding pertaining to related parties,

provision for doubtful debts from related parties, amounts written off or

written back in respect of debts due from or to related parties.

Names of the related party and nature of related party relationship to be

disclosed even where there are no transactions but the control exists.

Items of similar nature may be aggregated by type of the related party. The

type of related party for the purpose of aggregation of items of a similar

nature implies related party relationships. Material transactions; i.e., more

than 10% of related party transactions are not to be clubbed in an

aggregated disclosure. The related party transactions which are not

fb.com//SUPERWHIZZ4U CACWACS.WORDPRESS.COM

entered in the normal course of the business would ordinarily be

considered material (ASI-13).

A non-executive director is not a key management person for the purpose

of this standard. Unless,

o he is in a position to exercise significant influence

by virtue of owning an interest in the voting power or,

o he is responsible and has the authority for directing and controlling

the activities of the reporting enterprise. Mere participation in the

policy decision making process will not attract AS 18. (ASI-21).

Accounting Standard 19: Leases

Applies in accounting for all leases other than leases to explore for or use

natural resources, licensing agreements for items such as motion pictures

films, video recordings plays etc. and lease for use of lands.

A lease is classified as a finance lease or an operating lease.

A finance lease is one where risks and rewards incident to the ownership

are transferred substantially; otherwise it is an operating lease.

Treatment in case of finance lease in the books of lessee:

At the inception, lease should be recognised as an asset and a liability at lower of

fair value of leased asset and the present value of minimum lease payments

(calculated on the basis of interest rate implicit in the lease or if not

determinable, at lessee’s incremental borrowing rate).

Lease payments should be appropriated between finance charge and the

reduction of outstanding liability so as to produce a constant periodic rate of

interest on the balance of the liability.

Depreciation policy for leased asset should be consistent with that for other

owned depreciable assets and to be calculated as per AS 6.

Disclosure should be made of assets acquired under finance lease, net carrying

amount at the balance sheet date, total minimum lease payments at the balance

sheet date and their present values for specified periods, reconciliation between

total minimum lease payments at balance sheet date and their present value,

contingent rent recognised as income, total of future minimum sub lease

payments expected to be received and general description of significant leasing

arrangements.

Treatment in case of finance lease in the books of lessor:

The lessor should recognize the asset as a receivable equal to net investment in

lease.

Finance income should be based on pattern reflecting a constant periodic return

on net investment in lease.

fb.com//SUPERWHIZZ4U CACWACS.WORDPRESS.COM

Manufacturer/dealer lessor should recognize sales as outright sales. If artificially

low interest rates quoted, profit should be calculated as if commercial rates of

interest were charged. Initial direct costs should be expensed.

Disclosure should be made of total gross investment in lease and the present

value of the minimum lease payments at specified periods, reconciliation between

total gross investment in lease and the present value of minimum lease

payments, unearned finance income, unguaranteed residual value accruing to the

lessor, accumulated provision for uncollectible minimum lease payments

receivable, contingent rent recognised, accounting policy adopted in respect of

initial direct costs, general description of significant leasing arrangements.

Treatment in case of operating lease in the books of the

lessee :

Lease payments should be recognised as an expense on straightline basis or

other systematic basis, if appropriate.

Disclosure should be made of total future minimum lease payments for the

specified periods, total future minimum sub lease payments expected to be

received, lease payments recognised in the P & L statement with separate

amount of minimum lease payments and contingent rents, sub lease payments

recognised in the P & L statement, general description of significant leasing

arrangements.

Treatment in case of operating lease in the books of the lessor:

Lessors should present an asset given on lease under fixed assets and lease

income should be recognised on a straight-line basis or other systematic basis, if

appropriate.

Costs including depreciation should be recognised as an expense.

Initial direct costs are either deferred over lease term or recognised as expenses.

Disclosure should be made of carrying amount of the leased assets, accumulated

depreciation and impairment loss, depreciation and impairment loss recognised

or reversed for the period, future minimum lease payments in aggregate and for

the specified periods, general description of the leasing arrangement and policy

for initial costs.

Sale and leaseback transactions

If the transaction of sale and lease back results in a finance lease, any excess or

deficiency of sale proceeds over the carrying amount should be amortized over

the lease term in proportion to depreciation of the leased assets.

fb.com//SUPERWHIZZ4U CACWACS.WORDPRESS.COM

If the transaction results in an operating lease and is at fair value, profit or loss

should be recognised immediately. But if the sale price is below the fair value any

profit or loss should be recognised immediately, however, the loss which is

compensated by future lease payments should be amortized in proportion to the

lease payments over the period for which asset is expected to be used. If the sales

price is above the fair value the excess over the fair value should be amortised.

In a transaction resulting in an operating lease, if the fair value is less than the

carrying amount of the asset, the difference (loss) should be recognised

immediately.

Note : Leases applies to all assets leased out after 1st April, 2001 and is

mandatory.

Accounting Standard 20: Earnings Per Share

Focus is on denominator to be adopted for earnings per share (EPS)

calculation.

In case of enterprises presenting consolidated financial statements EPS to

be calculated on the basis of consolidated information, as well as

individual financial statements.

Requirement is to present basic and diluted EPS on the face of Profit and

Loss statement with equal prominence to all periods presented.

EPS required being presented even when negative.

Basic EPS is calculated by dividing net profit or loss for the period

attributable to equity shareholders by weighted average of equity shares

outstanding during the period. Basic & Diluted EPS to be computed on the

basis of earnings excluding extraordinary items (net of tax expense).

(Limited Revision w.e.f 1-4-2004)

Earnings attributable to equity shareholders are after

the preference dividend for the period and the attributable tax.

The weighted average number of shares for all the periods presented is

adjusted for bonus issue, share split and consolidation of shares. In case of

rights issue at price lower than fair value, there is an embedded bonus

element for which adjustment is made.

For calculating diluted EPS, net profit or loss attributable to equity

shareholders and the weighted average number of shares are adjusted for

the effects of dilutive potential equity shares (i.e., assuming conversion

into equity of all dilutive potential equity).

Potential equity shares are treated as dilutive when their conversion into

equity would result in a reduction in profit per share from continuing

operations.

Effect of anti-dilutive potential equity share is ignored in calculating

diluted EPS.

In calculating diluted EPS each issue of potential equity share is

considered separately and in sequence from the most dilutive to the least

dilutive.

fb.com//SUPERWHIZZ4U CACWACS.WORDPRESS.COM

This is determined on the basis of earnings per incremental potential

equity.

If the number of equity shares or potential equity shares outstanding

increases or decreases on account of bonus, splitting or consolidation

during the year or after the balance sheet date but before the approval of

financial statement, basic and diluted EPS are recalculated for all periods

presented. The fact is also disclosed.

Amounts of earnings used as numerator for computing basic and diluted

EPS and their reconciliation with Profit and Loss statement are disclosed.

Also, the weighted average number of equity shares used in calculating the

basic EPS and diluted EPS and the reconciliation between the two EPS is

to be disclosed.

Nominal value of shares is disclosed along with EPS.

It has been clarified that if an enterprise discloses EPS for complying with

requirements of any source or otherwise, should calculate and disclose

EPS as per AS 20. Disclosure under Part IV of Schedule VI to the

Companies Act, 1956 should be in accordance with AS 20 (ASI-12).

Note: Earnings Per Share apply to the enterprise whose equity shares and

potential equity shares are listed on a recognised stock exchange. If the

enterprise is not so covered but chooses to present EPS, then it should

calculate EPS in accordance with the standard.

Accounting Standard 21: Consolidated Financial Statements

To be applied in the preparation and presentation of consolidated financial

statements (CFS) for a group of enterprises under the control of a parent.

Consolidated Financial Statements is recommendatory. However, if

consolidated financial statements are presented, these should be prepared

in accordance with the standard. For listed companies mandatory as per

listing agreement.

Control means, the ownership directly or indirectly through subsidiaries, of

more than one-half of the voting power of an enterprise or control of the

composition of the board of directors or such other governing body, to

obtain economic benefit. Subsidiary is an enterprise that is controlled by

parent.

Control of composition implies power to appoint or remove all or a majority

of directors.

When an enterprise is controlled by two enterprises definitions of control,

both the enterprises are required to consolidate the financial statements of

the first mentioned enterprise (ASI-24).

Consolidated financial statements to be presented in addition to separate

financial statements.

All subsidiaries, domestic and foreign to be consolidated except where

control is intended to be temporary; i.e., intention at the time of investing

is to dispose the relevant investment in the ‘near future’ or the subsidiary

operates under severe long-term restrictions impairing transfer of funds to

the parent. ‘Near future’ generally means not more than twelve months

fb.com//SUPERWHIZZ4U CACWACS.WORDPRESS.COM

from the date of acquisition of relevant investments (ASI-8). Control is to be

regarded as temporary when an enterprise holds shares as ‘stock-in-trade’

and has acquired and held with an intention to dispose them in the near

future (ASI-25).

CFS normally includes consolidated balance sheet, consolidated P & L,

notes and other statements necessary for preparing a true and fair view.

Cash flow only in case parent presents cash flow statement.

Consolidation to be done on a line by line basis by adding like items of

assets, liabilities, income and expenses which involves:

Elimination of cost to the parent of the investment in the subsidiary and the

parent’s portion of equity of the subsidiary at the date of investment. The

difference to be treated as goodwill/capital reserve, as the case may be.

Minority interest in the net income to be adjusted against income of the group.

Minority interest in net assets to be shown separately as a liability.

Intra-group balances and intra-group transactions and resulting unrealised profits

should be eliminated in full. Unrealised losses should also be eliminated unless

cost cannot be recovered.

The tax expense (current tax and deferred tax) of the parent and its subsidiaries

to be aggregated and it is not required to recompute the tax expense in context of

consolidated information (ASI-26).

The parent’s share in the post-acquisition reserves of a subsidiary is not required

to be disclosed separately in the consolidated balance sheet. (ASI-28).

Where two or more investments are made in a subsidiary, equity of the

subsidiary to be generally determined on a step by step basis.

Financial statements used in consolidation should be drawn up to the

same reporting date. If reporting dates are different, adjustments for the

effects of significant transactions/events between the two dates to be

made.

Consolidation should be prepared using same accounting policies. If the

accounting policies followed are different, the fact should be disclosed

together with proportion of such items.

In the year in which parent subsidiary relationship ceases to exist,

consolidation of P & L account to be made up to date of cessation.

Disclosure is to be of all subsidiaries giving name, country of incorporation

or residence, proportion of ownership and voting power held if different.

Also nature of relationship between parent and subsidiary if parent does

not own more than one half of voting power, effect of the acquisition and

disposal of subsidiaries on the financial position, names of the subsidiaries

whose reporting dates are different than that of the parent.

fb.com//SUPERWHIZZ4U CACWACS.WORDPRESS.COM

When the consolidated statements are presented for the first time, figures

for the previous year need not be given.

Notes forming part of the separate financial statements of the parent

enterprise and its subsidiaries which are material to represent a true and

fair view are required to be included in the notes to the consolidated

financial statements

(ASI-15).

Accounting Standard 22: Accounting for Taxes on Income

Effective date when mandatory – (a) For listed companies and their

subsidiaries – 1-4-2001 (b) For other companies - 1-4-2002 (c) All other

enterprises - 1-4-2003.

The differences between taxable income and accounting income to be

classified into permanent differences and timing differences.

Permanent differences are those differences between taxable income and

accounting income, which originate in one period and do not get reverse

subsequently.

Timing differences are those differences between taxable income and

accounting income for a period that originate in one period and are

capable of reversal in one or more subsequent periods.

Deferred tax should be recognised for all the timing differences, subject to

the consideration of prudence in respect of deferred tax assets (DTA).

When enterprise has carry forward tax losses, DTA to be recognised only if there is

virtual certainty supported by convincing evidence of future taxable income.

Unrecognised DTA to be reassessed at each balance sheet date. Virtual certainty

refers to the fact that there is practically no doubt regarding the determination of

availability of the future taxable income. Also, convincing evidence is required to

support the judgment of virtual certainty (ASI-9).

In respect of loss under the head Capital Gains, DTA shall be recognised

only to the extent that there is a reasonable certainty of sufficient future

taxable capital gain (ASI - 4). DTA to be recognised on the amount, which is

allowed as per the provisions of the Act; i.e., loss after considering the cost

indexation as per the Income Tax Act.

Treatment of deferred tax in case of Amalgamation

(ASI-11)

in case of amalgamation in nature of purchase, where identifiable assets /

liabilities are accounted at the fair value and the carrying amount for tax

purposes continue to be the same as that for the transferor enter price, the

difference between the values shall be treated as a permanent difference

and hence it will not give rise to any deferred tax. The consequent

difference in depreciation charge of the subsequent years shall also be

treated as a permanent difference.

The transferee company can recognise a DTA in respect of carry forward

losses of the transferor enterprise, if conditions relating to prudence as per

fb.com//SUPERWHIZZ4U CACWACS.WORDPRESS.COM

AS 22 are satisfied, though transferor enterprise would not have

recognised such deferred tax assets on account of prudence. Accounting

treatment will depend upon nature of amalgamation, which shall be as

follows :

o In case of amalgamation is in the nature of purchase and assets and

liabilities are accounted at the fair value, DTA should be recognised at the

time of amalgamation (subject to prudence).

o In case of amalgamation is in the nature of purchase and assets and

liabilities are accounted at their existing carrying value, DTA shall not be

recognised at the time of amalgamation. However, if DTA gets recognised

in the first year of amalgamation, the effect shall be through adjustment to

goodwill/ capital reserve.

o In case of amalgamation is in the nature of merger, the deferred tax assets

shall not be recognised at the time of amalgamation. However, if DTA gets

recognised in the first year of amalgamation, the effect shall be given

through revenue reserves.

o In all the above if the DTA cannot be recognised by the first annual balance

sheet following amalgamation, the corresponding effect of this recognition

to be given in the statement of profit and loss.

Tax expenses for the period, comprises of current tax and deferred tax.

Current tax [includes payment u/s 115JB of the Act

(ASI-6)] should be measured at the amount expected to be paid to

(recovered from) the taxation authorities, using the applicable tax rates.

Deferred tax assets and liabilities should be measured using the tax rates

and tax laws that have been enacted or substantively enacted by the

balance sheet date and should not be discounted to their present value.

Deferred Tax to be measured using the regular tax rates for companies that

pay tax u/s 115JB of the Act (ASI-6).

DTA should be disclosed separately after the head ‘Investments’ and

deferred tax liability (DTL) should be disclosed separately after the head

‘Unsecured Loans’

(ASI-7) in the balance sheet of the enterprise. Assets and liabilities to be

netted off only when the enterprise has a legally enforceable right to set

off.

The break-up of deferred tax assets and deferred tax liabilities into major

components of the respective balances should be disclosed in the notes to

accounts.

The nature of the evidence supporting the recognition of deferred tax

assets should be disclosed, if an enterprise has unabsorbed depreciation or

carry forward of losses under tax laws.

The deferred tax assets and liabilities in respect of timing differences

which originate during the tax holiday period and reverse during the tax

holiday period, should not be recognised to the extent deduction from the

total income of an enterprise is allowed during the tax holiday period.

However, if timing differences reverse after the tax holiday period, DTA and

fb.com//SUPERWHIZZ4U CACWACS.WORDPRESS.COM

DTL should be recognised in the year in which the timing differences

originate. Timing differences, which originate first, should be considered

for reversal first (ASI-3) and (ASI-5).

On the first occasion of applicability of this AS the enterprise should

recognise, the deferred tax balance that has accumulated prior to the

adoption of this Statement as deferred tax asset / liability with a

corresponding credit / charge to the revenue reserves.

Accounting Standard 23: Accounting for Investments in Associates in

Consolidated Financial Statements

Consolidation is applicable to all associates including foreign associates.

The statement deals with accounting of associates in the preparation and

presentation of CFS.

Associates is an enterprise in which the investor has significant influence

and which is neither a subsidiary nor a joint venture of the investor.

Significant influence (ordinarily having 20% or more of the voting power) is

termed as power to participate in the financial/operating policy decisions

but does not have control over such policies. The potential equity shares

held by the investee should not be taken into account for determining the

voting power of the investor. (ASI-18).

Investment in associates is accounted in CFS as per equity method. The

equity method is not applicable where the investment is acquired for

temporary period (AS 18), i.e. intention at the time of investing is to

dispose the relevant investment in the ‘near future’ or where associates

operate under severe long-term restrictions. In these circumstances, the

investment should be recognised as per AS 13. The use of equity method to

be discontinued from the date when investor ceases to have significant

influence in an associate.

Provision for proposed dividend made by the associate in its financial

statements, should not be considered for the computation of the investor’s

share of the results of operations of the associate (ASI-16).

Goodwill / Capital Reserve on the acquisition of an associate should be

separately disclosed under carrying amount of investments.

Under the equity method, unrealised profit/losses resulting from the

transaction between investor and associates should be eliminated to the

extent of investor’s interest in the associates. However unrealised losses

should not be eliminated if cost of the assets cannot be recovered.

If associate has outstanding preference shares held outside the group,

preference dividends whether declared or not to be adjusted in arriving at

the investors share of profit or loss.

If investor’s share of losses of an associate equals or exceeds the carrying

amount of the investment, the investor will discontinue its share of loss

and will show its investment at nil value.

Where an associate presents consolidated financial statement, the results

and net assets of the associate’s CFS should be taken into account.

fb.com//SUPERWHIZZ4U CACWACS.WORDPRESS.COM

The carrying amount of investment in associates, on an individual basis,

should be reduced to recognize permanent decline in the value of

investment.

Listing and description of associates including proportion of ownership

interest and proportion of voting power should be disclosed in CFS.

The investor’s share of profits or losses and any extra- ordinary or prior

period items should be disclosed separately in CFS Profit and Loss A/c.

If reporting dates or accounting policies of associates are different from

that of financial statement of investor then the difference should be

reported in the CFS.

On the first occasion when investment in an associate is accounted for in

CFS, the carrying amount of investment in the associate should be

adjusted by using equity method, from the date of acquisition, with the

corresponding adjustment to the retained earnings in CFS.

Accounting Standard 24: Discontinuing Operations

The standard requires an enterprise to segregate information about

discontinuing operations from continuing one and establishes principles

for reporting information about discontinuing operations.

A Discontinuing operation is a part of an enterprise – (a) which is being

disposed of or abandoned pursuant to a single co-ordinated plan; (b) it

represents separate line of business or geographical area of operations;

and (c) can be distinguished operationally and for financial reporting. All

these three conditions need to be satisfied simultaneously.

Initial Disclosure Event is the earliest occurrence of one of the following :–

a. Entering into binding sale agreement for substantially all of the assets

attributable to the Discontinuing Operation.

b. Enterprise’s Governing body has approved a detailed, formal plan for the

discontinuance and made an announcement of the plan.

The statement does not establish any recognition and measurement

principles. It requires enterprise to follow principles established in other

Accounting Standard for the purpose of changes in assets, liabilities,

revenue, expenses etc.

An enterprise should give these information in its financial statements

beginning with the financial period in which the ‘Initial Disclosure Event’

occurs: (a) Description of discontinuing operation, (b) Segment in which it

is reported as per AS 17, (c) Date and nature of Initial Disclosure Event, (d)

Time by which the discontinuation is expected to be completed, (e) The

carrying amounts of the assets to be disposed of, (f) Revenue, expenses,

pre-tax profit / loss, income-tax in relation to the ordinary activities of

identified discounting operations.

On disposal of Assets or settlement of liabilities, disclosure is required for

gain/loss recognised on disposal/settlement and income tax expenses

thereto.

fb.com//SUPERWHIZZ4U CACWACS.WORDPRESS.COM

On entering into binding contract for sale of assets, disclosure is required

for Net Selling price after deducting expected disposal cost, the expected

timing of cash flow and the carrying amount of assets on the balance sheet

date.

For period subsequent to initial disclosure event period, description of any

significant changes in amount or timing of cash flow is required to be

disclosed.

The disclosures to continue up to the period in which the discontinuance is

completed; i.e., discontinuance plan is substantially completed or

abandoned.

In case discontinuance plan is abandoned, the disclosure is required of this

fact, reason therefore and its effect on the financial statements.

All disclosures should be separately presented for each discontinuing

operation.

Disclosure of pre-tax profit/loss from ordinary activities of the

discontinuing operation, income tax expenses related thereto, pre-tax

gain/loss recognised on the disposal / settlement to be made on the face

of profit and loss account.

Comparative information for prior periods to be re-stated to segregate

discontinuing operations.

In the Interim financial report, disclosure is required for any significant

activities or event and any significant changes in the amount or timing of

cash flows relating to disposal / settlement.

Accounting Standard 25: Interim Financial Reporting

Interim financial reports are financial statements (complete or condensed)

for on interim period that is shorter than a full financial year.

Interim financial report should include at a minimum a condensed balance

sheet, condensed profit and loss statement, cash flow and selected

explanatory notes.

They should include at least each of the heading and sub headings that

were included in the most recent annual financial statements.

Earnings per share if disclosed is to be calculated and presented as per AS

20.

Notes to include at least

o a statement on uniform accounting policies or any change therein.

o explanatory comments about the seasonality of interim operations.

o any unusual items (as per AS 5)

o changes in estimates of amounts reported in prior interim

periods/year, if material.

o issuances, buy-backs repayments and restructuring of debt, equity

and potential equity shares.

o dividends.

o segment reporting if required.

o any changes in composition of the enterprise.

o material changes in contingent liabilities.

fb.com//SUPERWHIZZ4U CACWACS.WORDPRESS.COM

Interim reports to include

o Balance sheet as of the end of current interim period and a

comparative balance sheet as of the end of the preceding financial

year.

o Statements of Profit & Loss for current interim period and

cumulative for current financial year to date and comparative

statements of the previous year (current and year to date)

o Cash flow statement cumulatively for the current financial year to

date with a comparative statement of previous year (year to date)

Interim measurements may rely on estimates.

For final interim period separate report not necessary as annual

statements are presented.

Uniform accounting policies to be applied in interim and annual financial

statements.

Seasonal/occasional revenues and uneven costs to be anticipated or

deferred only if appropriate to do so at the end of the financial year.

Estimates to be measured in such a way that resulting information is

reliable and all material information disclosed.

In case of change of accounting policies, other than one for which

transition is specified by an accounting standard, figures of prior interim

periods of current financial year to be restated.

Note: The presentation and disclosure requirements contained in AS 25 are not

required to be applied in respect of 'Interim financial results' – example, the one

presented under Clause 41 of the Listing Agreement, since they do not meet the

definition of 'interim financial report'. However, the recognition and measurement

principles as per AS 25 should be applied.

(ASI-27)

Accounting Standard 26: Intangible Assets

Not applicable to intangibles covered by other AS, financial assets, mineral

rights/expenditure on exploration, etc. arising in insurance enterprises

from contracts with policy holders and also to expenditure in respect of

termination benefits.

An intangible asset is an identifiable non-monetary asset, without physical

substance, held for use in the production or supply of goods or services, for

rental to others, or for administrative purposes. An asset is a resource:

o controlled by an enterprise as a result of past events; and

o from which future economic benefits are expected to flow to the

enterprise.

Useful life is period of time over which an asset is expected to be used or

the number of production units expected to be obtained from the asset.

Impairment loss is the amount by which the carrying amount exceeds its

recoverable amount.

An intangible asset to be recognised only if future economic benefits will

flow and the cost of the asset can be measured reliably.

fb.com//SUPERWHIZZ4U CACWACS.WORDPRESS.COM

Probability of future economic benefits to be assessed using reasonable

and supportable assumptions.

An intangible asset should be measured initially at cost.

Internally generated goodwill, brands, mastheads, publishing titles etc.

should not be recognised as an asset.

No intangible asset arising from research to be recognised and expenditure

on research should be recognised as an expense, when incurred.

An intangible asset arising from development to be recognised, if an

enterprise can demonstrate its feasibility to complete, intention and ability

to use or sell, generation of future economic benefits, and availability of

resources for completion and ability to measure the expenditure.

Expenditure on an intangible item that cannot be treated as an asset,

should be recognised as an expense and treated as goodwill (capital

reserve), in case of an amalgamation (AS 14).

Treatment of expenditure (other than expenditure on VRS) incurred on

intangible items, which do not meet the criteria of an 'intangible asset':

o If incurred after the date of AS 26 becoming mandatory – to be

expensed out when incurred;

The balances of expenditure incurred before the date of AS

26 becoming mandatory and appearing in the balance sheet,

should continue to be expensed out over a number of years

as originally contemplated;

If such balances have been adjusted against the opening

balances of revenue reserves as on 1-4-2003, it should be

rectified and treated on the above lines.

Expenditure, on an intangible item recognised as an expense should not

form part of cost of an intangible asset at a later date.

Subsequent expenditure to be added to cost only if is probable that the

expenditure will generate future benefits in excess of the original

estimates.

An intangible asset should be carried at its cost less any accumulated

amortisation and any accumulated impairment loses.

An intangible asset should be amortised over its useful life on a systematic

basis, to reflect the pattern in which the economic benefits are consumed

or if the pattern cannot be determined reliably, on the straightline method.

There is a rebuttable presumption for useful life of an intangible asset –

not exceeding ten years from the date it is available for use. In case of

intangible assets in form of legal rights, the useful life is not to exceed the

period of the legal rights, unless renewable, which is virtually certain.

Residual value to be taken as zero unless a commitment to purchase the

asset or an active market exists.

The amortisation period and method to be reviewed at each financial year

end and any change to be accounted for as per

AS 5.

Any impairment losses to be recognised.

fb.com//SUPERWHIZZ4U CACWACS.WORDPRESS.COM

The recoverable amount of each intangible asset to be estimated at each

year end in case of an intangible asset which is not yet available for use

and one which is amortised over a period exceeding ten years.

An intangible asset to be derecognised on disposal or when no future

economic benefits are expected from its use and gain or loss recognised.

Disclosure for each class of intangibles, their useful lives, amortisation

rate, amount and method, carrying amount (gross and net), any additions,

retirements, impairment losses recognised or reversed and any other

change.

In case of useful life of an intangible asset exceeding ten years, proper

disclosure of the reasons for the same should be given.

Research and Development expenditure recognised as expense to be

disclosed.

On standard being applicable, adjustment to any intangible asset as

required to be made with a corresponding adjustment to the opening

revenue reserves.

Accounting Standard 27: Financial Reporting of Interests in Joint Ventures

A joint venture is a contractual arrangement whereby two or more parties

undertake an economic activity, which is subject to joint control.

In cases, wherein an enterprise by a contractual arrangement establishes

joint control over an entity which is a subsidiary (as per AS 21) the entity is

to be consolidated under AS 21 and is not to be treated as a joint venture

as per this Statement. The other venturer(s) may treat the same as a joint

venture. (Limited Revision to AS 27 w.e.f 1-4-2004)

Joint control is the contractually agreed sharing of control over an

economic activity.

For evaluating joint control, one need to consider whether the contractual

arrangement provides protective rights or participating rights to the

enterprise. The existence of participating rights would be evidence of joint

control. With effect from 1-4-2004 this explanations is removed by Limited

Revision to the Standard.

Control is the power to govern the financial and operating policies of an

economic activity so as to obtain benefits from it.

A venturer is a party to a joint venture and has joint control over that joint

venture.

An investor in a joint venture is a party to a joint venture and does not have

joint control over that joint venture.

Proportionate consolidation is a method of accounting and reporting

whereby a venturer’s share of each of the assets, liabilities, income and

expenses of a jointly controlled entity is reported as separate line items in

the venturer’s financial statements. The venturer's share in the post

acquisition reserves of the jointly controlled entity should be shown

separately under the relevant reserves in the consolidated financial

statements (ASI 28).

fb.com//SUPERWHIZZ4U CACWACS.WORDPRESS.COM

Venturer to recognise in individual and consolidated financial statements

its share of assets, liabilities, incomes and expenses in the jointly

controlled operations and also in jointly controlled assets.

In venturer’s separate financial statements any interest in a jointly

controlled entity to be accounted as an investment and AS 13 to be

followed.

In a venturer’s consolidated financial statements interest in jointly

controlled entity to be reported using proportionate consolidation except

o when interest is acquired and held with a view of disposal in near

future to be considered as not more than 12 months from

acquisition of relevant investments unless a longer period can be

justified on the basis of facts and circumstances (ASI 8)

o when severe long-term restrictions that impair the ability to transfer

funds to the venturer exists.

In such cases interest to be accounted as investments as per AS 13.

The venturer’s share in the post acquisition reserves of the jointly controlled entity

should be shown separately under the relevant reserves in the consolidated

financial statements (ASI-28).

A venturer to discontinue use of proportionate consolidation from the date

o it ceases to have joint control (may retain interest)

o use of proportionate consolidation is no longer appropriate.

In such cases AS 21 to be followed if venturer becomes parent and in other

cases AS 13 and/or AS 23 to be followed.

Cost in such cases is the venturers’ share in net assets on date of

discontinuance of proportionate consolidation as adjusted by any

goodwill/capital reserve recognised at the time of acquisition.

In case of sale of assets by a venturer to the joint venture the venturer

should recognise only that portion of gain or loss as attributable to the

interests of the other venturers. Full loss to be booked in case of evidence

of reduction in the net realisable value of current assets or on impairment

loss.

In case of purchase of assets by a venturer from a joint venture, the

venturer should recognise its share of profit only on a resale of the asset to

an independent party. Loss to be booked in case of reduction in net

realisable value of current asset or impairment loss.

In case of transactions between venturer and joint venture the above

principles to be followed only in consolidated financial statements.

Investor to follow AS 13, AS 21 and AS 23 as appropriate, for investments

in joint ventures.

Operators/Managers of joint ventures to account for fees as per AS 9.

A venturer to disclose separately, in respect of the joint venture,

contingent liabilities and capital commitments.

fb.com//SUPERWHIZZ4U CACWACS.WORDPRESS.COM

A venturer to disclose list of joint ventures and interests in significant joint

ventures.

A venturer to disclose aggregate amounts of each of the assets, liabilities,

income and expenses related to its interests in the jointly controlled

entities.

Accounting Standard 28 : Impairment of Assets

Applied in accounting for the impairment of all assets, other than:

o inventories (AS 2);

o assets arising from construction contracts (AS 7);

o financial assets, including investments (AS 13); and

o deferred tax assets (AS 22).

Recoverable amount is the higher of an asset’s net selling price and its

value in use.

Value in use is the present value of estimated future cash flows expected

to arise from the continuing use of an asset and from its disposal at the

end of its useful life.

An impairment loss is the amount by which the carrying amount of an asset

exceeds its recoverable amount.

Useful life is either:

o the period of time over which an asset is expected to be used ; or

o the number of production or similar units expected to be obtained

from the asset.

A cash generating unit is the smallest identifiable group of assets that

generates cash inflows largely independent of the cash inflows from other

assets.

Corporate assets are assets other than goodwill that contribute to the

future cash flows of both the cash generating unit under review and other

cash generating units.

An active market is a market where:

o the items traded are homogeneous;

o willing buyers and sellers can normally be found at any time; and

o prices are available to the public.

o To assess at each balance sheet date any indication, external or

internal as given in AS, that an asset may be impaired and estimate

the recoverable amount of the asset.

In measuring value in use:

o cash flow projections should be based on assumptions that

represent management’s best estimate of the set of economic

conditions that will exist over the remaining useful life of the asset.

Greater weight should be given to external evidence;

o cash flow projections should be based on the most recent financial

budgets/forecasts (maximum 5 years, unless longer period justified)

that have been approved by management.

o cash flow projections beyond the period covered by the most recent

budgets/forecasts should be estimated by extrapolating the

fb.com//SUPERWHIZZ4U CACWACS.WORDPRESS.COM

projections based on the budgets/forecasts using a steady or

declining growth rate for subsequent years, unless an increasing

rate can be justified. This growth rate should not exceed the long-

term average growth rate for the products, industries, or country or

countries in which the enterprise operates, or for the market in

which the asset is used, unless a higher rate can be justified.

Estimates of future cash flows should include:

o projections of cash inflows from the continuing use of the asset;

o projections of cash outflows that are necessarily incurred to

generate the cash inflows from continuing use of the asset

(including cash outflows to prepare the asset for use) and that can

be directly attributed, or allocated on a reasonable and consistent

basis, to the asset; and

o net cash flows, if any, to be received (or paid) for the disposal of the

asset at the end of its useful life.

Future cash flows should be estimated for the asset in its current

condition. They should not include estimated future cash inflows or

outflows that are expected to arise from:

o a future restructuring to which an enterprise is not yet committed;

or

o future capital expenditure that will improve or enhance the asset in

excess of its originally assessed standard of performance.

Estimates of future cash flows should not include:

o cash inflows or outflows from financing activities; or

o income tax receipts or payments.

The estimate of net cash flows to be received (or paid) for the disposal of

an asset at the end of its useful life should be the amount that is expected

to be obtained from the disposal of the asset in an arm’s length transaction

between knowledgeable, willing parties, after deducting the estimated

costs of disposal.

The discount rate should be a pre tax rate that reflect current market

assessments of the time value of money and the risks specific to the asset

and should not reflect risks for which future cash flow estimates have been

adjusted.

Impairment loss is the reduction in carrying amount of the assets to its

recoverable amount.

An impairment loss should be recognised as an expense in the profit and

loss account immediately. Impairment loss of a revalued asset should be

treated as a revaluation decrease as per AS 10.

If the estimated impairment loss is greater than the carrying amount of the

asset, recognise a liability if, and only if, required by another AS.

The depreciation/amortisation charge for the asset should be adjusted in

future periods to allocate the asset’s revised carrying amount, less its

residual value on a systematic basis over its remaining useful life.

In case of any indication of impairment, the recoverable amount should be

estimated for the individual asset. If it is not possible, determine the

fb.com//SUPERWHIZZ4U CACWACS.WORDPRESS.COM

recoverable amount of the cash-generating unit to which the asset

belongs.

If an active market exists for the output produced by an asset or a group of

assets, the same should be identified as a separate cash-generating unit,

even if some or all of the output is used internally. In such case

management’s

best estimate for future market price of output should be used:

o in determining the value in use of this cash-generating unit, when

estimating the future cash inflows that relate to the internal use of

the output; and

o in determining the value in use of other cash-generating units of the

reporting enterprise, when estimating the future cash outflows that

relate to the internal use of the output.

Cash-generating units should be identified consistently from period to

period for the same asset or types of assets, unless a change is justified.

The carrying amount of a cash-generating unit should be determined

consistently with the way the recoverable amount of the cash-generating

unit is determined

In testing a cash-generating unit for impairment, identify whether goodwill

that relates to this unit is recognised in the financial statements. If this is

the case, an enterprise should:

o perform a ‘bottom-up’ test.

o if, in the ‘bottom-up’ test, the carrying amount of goodwill could not

be allocated on a reasonable and consistent basis to the cash-

generating unit under review, the enterprise should also perform a

‘top-down’ test.

In testing a cash-generating unit for impairment, identify all the corporate

assets that relate to the cash-generating unit under review. For each

identified corporate asset, apply ‘bottom-up’ test or ‘bottom-up’ and ‘top-

down’ test both as required.

Impairment loss should be recognised for a cash-generating unit if, and

only if, its recoverable amount is less than its carrying amount. The

impairment loss should be allocated to reduce the carrying amount of the

assets of the unit in the following order:

o first, to goodwill allocated to the cash-generating unit (if any); and

o then, to the other assets of the unit on a pro rata basis based on the

carrying amount of each asset in the unit.

These reductions in carrying amounts should be treated as impairment

losses on individual assets and recognised either in P & L account or as

revaluation decrease as applicable.

In allocating an impairment loss, the carrying amount of an asset should

not be reduced below the highest of:

o its net selling price (if determinable);

o its value in use (if determinable); and

o zero.

fb.com//SUPERWHIZZ4U CACWACS.WORDPRESS.COM

The amount of the impairment loss that would otherwise have been

allocated to the asset should be allocated to the other assets of the unit on

a pro rata basis.

A liability should be recognised for any remaining amount of an impairment

loss for a cash-generating unit if, required by another AS.

At each balance sheet date, if there are indications internal or external,

that an impairment loss recognised for an asset in prior accounting

periods, no longer exists/has decreased, then the recoverable amount of

that asset to be estimated. For the same consider the following as

minimum indications:

An impairment loss recognised for an asset in prior accounting periods

should be reversed if there is a change in the estimates of cash inflows,

cash outflows or discount rates used to determine the asset’s recoverable

amount since the last impairment loss was recognised. The carrying

amount of the asset should be increased to its recoverable amount.

The increased carrying amount of an asset due to a reversal of an

impairment loss should not exceed the carrying amount that would have

been determined (net of amortisation or depreciation) had no impairment

loss been recognised for the asset in prior accounting periods.

A reversal of an impairment loss for an asset should be recognised as

income immediately in profit and loss account. In case of revalued assets,

the same should be treated as a revaluation increase as per AS 10.

After a reversal of an impairment loss, the depreciation (amortisation)

charge for the asset should be adjusted in future periods to allocate the

asset’s revised carrying amount, less its residual value (if any), on a

systematic basis over its remaining useful life.

A reversal of an impairment loss for a cash-generating unit should be

allocated to increase the carrying amount of the assets of the unit in the

following order:

o first, assets other than goodwill on a pro rata basis based on the

carrying amount of each asset in the unit; and

o then, to goodwill allocated to the cash-generating unit, if the

requirements of reversal of impairment loss of goodwill are met.

These increases in carrying amounts should be treated as reversals of

impairment losses for individual assets and recognised accordingly.

In allocating a reversal of an impairment loss for a cash-generating unit,

the carrying amount of an asset should not be increased above the lower

of:

o its recoverable amount (if determinable); and

o the carrying amount that would have been determined (net of

amortisation or depreciation) had no impairment loss been

recognised for the asset in prior accounting periods.

The amount of the reversal of the impairment loss that would otherwise

have been allocated to the asset should be allocated to the other assets of

the unit on a pro-rata basis.

An impairment loss recognised for goodwill should not be reversed in a

subsequent period unless:

fb.com//SUPERWHIZZ4U CACWACS.WORDPRESS.COM

o the impairment loss was caused by a specific external event of an

exceptional nature that is not expected to recur; and

o subsequent external events have occurred that reverse the effect of

that event.

For each class of assets, the financial statements should disclose:

o the amount of impairment losses recognised in the statement of

profit and loss during the period and the line item(s) of the

statement of profit and loss in which those impairment losses arae

included;

o the amount of reversals of impairment losses recognised in the

statement of profit and loss during the period and the line item(s) of

the statement of

profit and loss in which those impairment losses are reversed;

o the amount of impairment losses recognised

directly against revaluation surplus during the period; and

o the amount of reversals of impairment losses recognised directly in

revaluation surplus during the period.

An enterprise that applies AS 17, should disclose the following for each

reportable segment based on an enterprise’s primary format (as defined in

AS 17):

o the amount of impairment losses recognised in the statement of

profit and loss and directly against revaluation surplus during the

period; and

o the amount of reversals of impairment losses recognised in the

statement of profit and loss and directly in revaluation surplus

during the period.

o If an impairment loss for an individual asset or a cash-generating

unit is recognised or reversed during the period and is material to

the financial statements of the reporting enterprise as a whole, an

enterprise should disclose;

o the events and circumstances that led to the recognition or reversal

of the impairment loss;

the amount of the impairment loss recognised or reversed;

o for an individual asset:

the nature of the asset; and

the reportable segment to which the asset belongs, based on the

enterprise’s primary format (as per AS 17);

o for a cash-generating unit:

a description of the cash-generating unit;

the amount of the impairment loss recognised or reversed by class

of assets and by reportable segment based on the enterprise’s

primary format (as defined in AS 17); and

if the aggregation of assets for identifying the cash-generating unit

has changed since the previous estimate of the cash-generating

fb.com//SUPERWHIZZ4U CACWACS.WORDPRESS.COM

unit’s recoverable amount (if any), the enterprise should describe

the current and former way of aggregating assets and the reasons

for changing the way the cash-generating unit is identified;

whether the recoverable amount of the asset

(cash-generating unit) is its net selling price or its value in use;

o if recoverable amount is net selling price, the basis used to

determine net selling price; and

o if recoverable amount is value in use, the discount rate used in the

current estimate and previous estimate (if any) of value in use.

If impairment losses recognised (reversed) during the period are material in

aggregate to the financial statements of the reporting enterprise as a

whole, an enterprise should disclose a brief description of the following:

o the main classes of assets affected by impairment losses (reversals

of impairment losses) for which no information is disclosed; and

o the main events and circumstances that led to the recognition

(reversal) of these impairment losses for which no information is

disclosed.

As a transitional provision any impairment loss determined before this

standard becomes mandatory should be adjusted against the opening

balance of revenue reserve. Impairment losses on revalued assets to be

adjusted against balance in revaluation reserve and excess, if any against

the opening balance of revenue reserve.

Accounting Standard 29 : Provisions, Contingent Liabilities and Contingent Assets

This statement should be applied in accounting for provisions and

contingent liabilities and in dealing with contingent assets, other than

those resulting from financial instruments that are carried at fair value,

those resulting from executory contracts, those arising in insurance

enterprises from contracts with policy – holders and those covered by

another Accounting Standard.

Provision is a liability, which can be measured only by using a substantial

degree of estimation.

Liability is a present obligation arising from past events, the settlement of

which is expected to result in an outflow of resources embodying economic

benefits.

Contingent Liability is -

o a possible obligation that arises from past events and the existence

of which will be confirmed only by the occurrence or non-occurrence

of one or more uncertain future events not wholly within the control

of the enterprise; or

o a present obligation, but is not recognised because it is not

probable that outflow of resources embodying economic benefits

will be required (or is remote) for its settlement or a reliable

estimate of the amount of the obligation cannot be made.

fb.com//SUPERWHIZZ4U CACWACS.WORDPRESS.COM

Contingent asset is a possible asset that arises from past events, the

existence of which will be confirmed only by the occurrences or non-

occurrence of one or more uncertain future events not wholly within the

control of the enterprise.

A provision should be recognised when –

o an enterprises has a present obligation as a result of a past event;

o it is probable (more likely than not) that an outflow of resources will

be required to settle the obligation; and

o a reliable estimate can be made of the amount of the obligation.

A contingent liability is not recognised in financial statements but is

disclosed.

A contingent asset is not recognised in financial statements.

The amount of provision should be measure before tax at the best estimate

of the expenditure required to settle the present obligation and should not

be discounted to its present value.

The risks and uncertainties that inevitably surround many events and

circumstances should be taken into account in arriving at the best

estimate of provision to avoid its under or over statement.

Expected future events, which are likely to affect the amount required to

settle an obligation, may be important in measuring provisions.

Gains on the expected disposal of assets should not be taken into account

in measuring a provision, even if the expected disposal is closely linked

with the item requiring provision.

Whenever all or part of the expenditure relevant to a provision is expected

to be reimbursed by another party, the reimbursement should be

recognised only on virtual certainty of its receipt. The reimbursement

should be treated as a separate asset and should not exceed the amount of

the provision. In the statement of profit and loss, the expense relating to a

provision may be presented net of the amount recognised for a

reimbursement.

Provisions should be reviewed at each balance sheet date and adjusted to

reflect the current best estimate. The provision should be reversed, if it is

no longer probable to result in a liability.

A provision should be used only for expenditures for which the provision

was originally recognised and not against a provision recognised for

another purpose, so as not to conceal the impact of two different events.

Provision should not be recognised for future operating losses, since it is

not a liability nor meet the crieteria for provisions.

A restructuring provision should include only the direct expenditures,

necessarily entailed by the restructuring and not associated with the

ongoing activities of the enterprise.

Disclosure

o For each class of provision - the carrying amount at the beginning

and end of the period; additional provisions made, amounts used

and unused amounts reversed during the period.

o Also for each class of provision – description of the nature of the

obligation, the expected timing of any resulting outflows of

fb.com//SUPERWHIZZ4U CACWACS.WORDPRESS.COM

economic benefits, the uncertainties about those outflows and the

amount of any expected reimbursement (also stating the amount of

any asset recognised therefor)

o For each class of contingent liability – a brief description of its

nature and where practicable, an estimate of its financial effect, the

uncertainties relating to any outflow and the possibility of any

reimbursement. If the information is not disclosed, being not

practicable, the fact thereof is to be disclosed.

o In extremely rare cases, disclosure of any information can be

expected to prejudice seriously the position of the enterprise in a

dispute with other parties; in such cases the information need not

be disclosed but,

the fact and reason for such non–disclosure alongwith the general

nature of dispute should be disclosed.

fb.com//SUPERWHIZZ4U CACWACS.WORDPRESS.COM

sipoy satish kumar

8.38

UNIT 6: PROBLEMS BASED ON ACCOUNTING STANDARDS AND GUIDANCE NOTES

Question 1

Events Occurring after the Balance Sheet Date and their disclosure requirements.

(5 marks) (Intermediate–Nov. 1994, May 97 and May 1998)

Answer

Events occurring after the balance sheet date are those significant events, both favourable and unfavourable, that occur between the balance sheet date and the date on which the financial statements are approved by the Board of Directors in the case of a company and in the case of any other entity by the corresponding approving authority.

Assets and liabilities should be adjusted for events occurring after the balance sheet date that provide additional evidence to assist the estimation of amounts relating to conditions existing at the balance sheet date or that indicate that the fundamental accounting assumption of going concern (i.e., the continuance of existence or substratum of the enterprise) is not appropriate. However, assets and liabilities should not be adjusted for but disclosure should be made in the report of the approving authority of events occurring after the balance sheet date that represent material changes and commitments affecting the financial position of the enterprise.

(ii) Disclosure regarding events occurring after the balance sheet date :

(a) The nature of the event;

(b) An estimate of the financial effect, or a statement that such an estimate cannot be made.

Question 2

Prior-Period items. (2 marks) (Intermediate–Nov. 1994, May 1996 and May 1998)

Answer

When income or expenses arise in the current period as a result of errors or omissions in the preparation of the financial statements of one or more prior periods, the said incomes or expenses have to be classified as prior period items. The errors may occur as a result of mathematical mistakes, mistakes in applying accounting policies, misinterpretation of facts or oversight.

Question 3

Pre–incorporation expenses. (5 marks) (Intermediate–May 1996)

Answer

Pre–incorporation expenses denote expenses incurred by the promoters for the purposes of the company before its incorporation.

Broadly, these include expenses in connection with:

(a) preliminary analysis of the conceived idea,

(b) detailed investigation in terms of technical feasibility and commercial viability to establish the soundness of the proposition,

(c) preparation of ‘project report’ or ‘feasibility report’ and its verification through independent appraisal authority (before giving final approval to the proposition) and

(d) organisation of funds, property and managerial ability and assembling of other business elements.

These expenses should be properly capitalised and shown in the balance sheet under the heading “Miscellaneous Expenditure”. There is no legal requirement to write–off these expenses to profit and loss

8.39

account within any specified period of time nor is there any rigid accounting convention in regard to this matter. However, good corporate practice recognises the need to write off these expenses to profit and loss account whtin a period of 3 to 5 years.

Question 4

Provisions contained in the Accounting Standard in respect of Revaluation of fixed assets.

(10 marks) (Intermediate–Nov. 1996)

Answer

(i) Revaluation of fixed Assts

According to Accounting Standard 10 on “Accounting for Fixed Assets”

(a) When fixed assets are revalued in financial statements, the basis of selection should be an entire class of assets or the selection should be done on a systematic basis. The basis of selection should be disclosed.

(b) The revaluation of any class of assets should not result in the net book value of that class being greater than the recoverable amount of that class of assets.

(c) The accumulated depreciation should not be credited to profit and loss account.

(d) The net increase in book value should be credited to a revaluation reserve account.

(e) On disposal of a previously revalued item of fixed asset, the difference between net disposal proceeds and the net book value should be charged or credited to the profit and loss account except that to the extent to which such a loss is related to an increase and which has not been subsequently reversed or utilised may be charged directly to that account.

Questiion 5

The difference between actual expense or income and the estimated expense or income as accounted for in earlier years’ accounts, does not necessarily constitue the item to be a prior period item comment. (2 marks) (Intermediate–May 1998)

Answer

The statement given in the question is correct and is in accordance with the Accounting Standard (AS) 5 (Revised) “Net Profit or Loss for the Period. Prior Period Items and Changes in Accounting Policies’’.

The use of reasonable estimates is an essential part of the preparation of financial statements and does not undermine their reliability. An estimate may have to be revised if changes occur regarding the circumstances on which the estimate was based, or as a result of new information or subsequent developments. The revision of the estimate, by its nature, does not bring the adjustments within the definition of an extraordinary item or a prior period item.

Question 6

When can revenue be recognised in the case of transaction of sale of goods?

(2 marks) (Intermediate–May 1998)

Answer

As per AS 9 Revenue Recognition, revenue from sales transactions should be recognised when the following requirements as to performance are satisfied, provided that at the time of performance it is not unreasonable to expect ultimate collection :

(i) The seller of goods has transferred to the buyer the property in the goods for a price or all significant risks and rewards of ownership have been transferred to the buyer and the seller retains no effective control of the goods transferred to a degree usually associated with ownership; and

8.40

(ii) No significant uncertainty exists regarding the amount of the consideration that will be derived from the sale of goods.

Question 7

Valuation of fixed assets in special cases. (3 marks) (Intermediate–Nov. 1998)

Answer

Para 15 of Accounting Standard 10 on “Accounting for Fixed Assets” states the following provisions regarding valuation of fixed assets in special cases :

1. In the case of fixed assets acquired on hire purchase terms, although legal ownership does not vest in the enterprise, such assets are recorded at their cash value, which if not readily available, is calculated by assuming an appropriate rate of interest. They are shown in the balance sheet with an appropriate arration to indicate that the enterprise does not have full ownership thereof.

2. Where an enterprise owns fixed assets jointly with others (otherwise than as a partner in a firm), the extent of its share in such assets, and the proportion in the original cost, accumulated depreciaiton and written down value are stated in the balance sheet. Alternatively, the pro rata cost of such jointly owned assets is grouped together with similar fully owned assets. Details of such jointly owned assets are indicated separately in the fixed assets register.

3. Where several assets are purchased for a consolidated price, the consideration is apportioned to the various assets on a fair basis as determined by competent valuers.

Question 8

What are the main features of the Cash Flow Statement? Explain with special reference to AS 3? (5 marks) (Intermediate–Nov. 1999)

Answer

According to AS 3 (Revised) on “Cash Flow Statements”, cash flow statement deals with the provision of information about the historical changes in cash and cash equivalents of an enterprise during the given period from operating, investing and financing activities. Cash flows from operating activities can be reported using either

(a) the direct method, whereby major classes of gross cash receipts and gross cash payments are disclosed; or

(b) the indirect method, whereby net profit or loss is adjusted for the effects of transactions of non–cash nature, any deferrals or accruals of past or future operating cash receipts or payments, and items of income or expense associated with investing or financing cash flows.

As per para 42 of AS 3 (Revised), an enterprise should disclose the components of cash and cash equivalents and should present a reconciliation of the amounts in its cash flow statement with the equivalent items reported in the balance sheet.

A cash flow statement when used in conjunction with the other financial statements, provides information that enables users to evaluate the changes in net assets of an enterprise, its financial structure (including its liquidity and solvency), and its ability to affect the amount and timing of cash flows in order to adapt to changing circumstances and opportunities. This statement also enhances the comparability of the reporting of operating performance by different enterprises because it eliminates the effects of using different accounting treatments for the same transactions and events.

AS 3 (revised) is recommendatory at present but for companies listed on stock exchanges, its compliance is mandatory due to the listing agreement which provides for the listed companies to furnish cash flow statement in their Annual Reports.

Question 9

Extraordinary Items to be disclosed as per the Accounting Standard.

(3 marks) (Intermediate–Nov. 1994)

8.41

Answer

Extraordinary items are gains or losses which arise from events or transactions that are distinct from the ordinary activities of the business and which are both material and expected not to recur in future frequently. These would also include material adjustments necessitated by circumstances, which though related to previous periods are determined in the current period. Some examples of extraordinary items may be the sale of a signficant part of the business, the sale of an investment not acquired with the intention of resale etc. The nature and amount of each extraordinary item are separately disclosed so that users of financial statements can evaluate the relative significance of such items and their effect on the current operating results. It may be noted that income or expenses arising from the ordinary activities of the enterprise, though abnormal in amount or infrequent in occurrence, do not qualify as extraordinary.

Question 10

(i) A major fire has damaged the assets in a factory of a limited company on 2nd April-two days after the year end closure of account. The loss is estimated at Rs. 20 crores out of which Rs. 12 crores will be recoverable from the insurers. Explain briefly how the loss should be treated in the final accounts for the previous year.

(ii) There is a sales tax demand of Rs. 2.50 crores against a company relating to prior years against which the company has gone on appeal to the appellate authority in the department. The grounds of appeal deal with points covering Rs. 2 crores of the demand. State how the matter will have to be dealt with in the final accounts for the year.

(8 marks) (Intermediate–May 1995)

Answer

(i) The loss due to break out of fire is an example of event occurring after the balance sheet date that does not relate to conditiont existing at the balance sheet date. It has not affected the financial position as on the date of the balance sheet and therefore requires no specific adjustments in the financial statements. However, paragraph 8.6 of AS-4 states that disclosure is generally made of events in subsequent periods that represent unusual changes affecting the existence or substratum of the enterprise at the balance sheet date. In the given case, the loss of assets in a factory is considered to be an event affecting the substratum of the enterprise after the balance sheet date. Hence, as recommended in paragraph 15 of AS-4, disclosure of the event should be made in the report of the approving authority that represent material changes and commitments affecting the financial position of the enterprise.

(ii) The undisputed part of sales tax liability of Rs. 0.50 crore should be considered as actual liability and adequately provided for. The Institute of Chartered Accountants of India has issued Accounting standard 29 on “Provisions Contingent Liabilities and Contingent Assets’’ (comes into effect in respect of accounting periods commencing on or after 1.4.2004). According to the standard, an enterprise should not recognise a contingent liability but should disclose it, as required by paragraph 68, unless the possibility of an outflow of resources embodying economic benefits is remote. Accordingly the company should disclose the disputed part of sales tax liability of Rs. 2 crore as contingent liability in their financial statements of the year. However, the above disclosed contingent liability should be reviewed continuosly and if it becomes probable that an outflow of future economic benefit will be required , then recognise the contingent liability as a provision.

Question 11

Jagannath Ltd. had made a rights issue of shares in 1996. In the offer document to its members, it had projected a surplus of Rs. 40 crores during the accounting year to end on 31st March, 1998. The draft results for the year, prepared on the hitherto followed accounting policies and presented for perusal of the board of directors showed a deficit of Rs. 10 crores. The board in consultation with the managing director, decided on the following :

8.42

(i) Value year-end inventory at works cost (Rs. 50 crores) instead of the hitherto method of valuation of inventory at prime cost (Rs. 30 crores).

(ii) Provide depreciation for the year on straight line basis on account of substantial additions in gross block during the year, instead of on the reducing balance method, which was hitherto adopted. As a consequence, the charge for depreciation at Rs. 27 crores is lower than the amount of Rs. 45 crores which would have been provided had the old method been followed, by Rs. 18 cores.

(iii) Not to provide for “after sales expenses” during the warranty period. Till the last year, provision at 2% of sales used to be made under the concept of “matching of costs against revenue” and actual expenses used to be charged against the provision. The board now decided to account for expenses as and when actually incurred. Sales during the year total to Rs. 600 crores.

(iv) Provide for permanent fall in the value of investments - which fall had taken place over the past five years - the provision being Rs. 10 crores.

As chief accountant of the company, you are asked by the managing director to draft the notes on accounts for inclusion in the annual report for 1997-1998 (6 Marks) (Intermediate–May 1998)

Answer

As per AS 1 “Any change in the accounting policies which has a material effect in the current period or which is reasonably expected to have a material effect in later periods should be disclosed. In the case of a change in accounting policies which has a material effect in the current period, the amount by which any item in the financial statements is affected by such change should also be disclosed to the extent ascertainable. Where such amount is not ascertainable, wholly or in part, the fact should be indicated. Accordingly, the notes on accounts should properly disclose the change and its effect.

Notes on Accounts :

(i) During the year inventory has been valued at factory cost, against the practice of valuing it at prime cost as was the practice till last year. This has been done to take cognisance of the more capital intensive method of production on account of heavy capital expenditure during the year. As a result of this change, the year-end inventory has been valued at Rs. 50 crores and the profit for the year is increased by Rs. 20 crores.

(ii) In view of the heavy capital intensive method of production introduced during the year, the company has decided to change the method of providing depreciation from reducing balance method to straight line method. As a result of this change, depreciation has been provided at Rs. 27 crores which is lower than the charge which would have been made had the old method and the old rates been applied, by Rs. 18 crores. To that extent, the profit for the year is increased.

(iii) So far, the company has been providing 2% of sales for meeting “after sales expenses during the warranty period. With the improved method of production, the probability of defects occurring in the products has reduced considerably. Hence, the company has decided not to make provision for such expenses but to account for the same as and when expenses are incurred. Due to this change, the profit for the year is increased by Rs. 12 crores than would have been the case if the old policy were to continue.

(iv) The company has decided to provide Rs. 10 crores for the permanent fall in the value of investments which has taken place over the period of past five years. the provision so made has reduced the profit disclosed in the accounts by Rs. 10 crores.

Question 12

Media Advertisers obtained advertisement rights for One Day World Cup Cricket Tournament to be held in May/June, 1999 for Rs. 250 lakhs.

By 31st March, 1999 they have paid Rs. 150 lakhs to secure these advertisement rights. The balance Rs. 100 lakhs was paid in April, 1999.

8.43

By 31st March, 1999 they procured advertisement for 70% of the available time for Rs. 350 lakhs. The advertisers paid 60% of the amount by that date. The balance 40% was received in April, 1999.

Advertisements for the balance 30% time were procured in April, 1999 for Rs. 150 lakhs. The advertisers paid the full amount while booking the advertisement.

25% of the advertisement time is expected to be available in May, 1999 and the balance 75% in June, 1999.

You are asked to :

(i) Pass journal entries in relation to the above.

(ii) Show in columnar form as to how the items will appear in the monthly financial statements for March, April, May and June 1999.

Give reasons for your treatment. (12 marks) (Intermediate–May 1999)

8.44

Answer

In the books of Media Advertisers

Journal Entries

Dr. Cr.

Rs. in lakhs Rs. in lakhs

1999

March Advance for advertisement rights (purchase) A/c Dr. 150.00

To Bank A/c 150.00

(Being advance paid for obtaining advertisement

rights)

Bank A/c Dr. 210.00

To Advance for advertisement time (sale) A/c 210.00

(Being advance received from advertisers

amounting to 60% of Rs. 350 lakhs for booking

70% advertisement time)

April Advance for advertisement rights (purchase) A/c Dr. 100.00

To Bank A/c 100.00

(Being balance advance i.e., Rs. 250 lakhs less

Rs. 150 lakhs paid)

Bank A/c Dr. 140.00

To Advance for advertisement time (sale) A/c 140.00

(Being balance advance i.e., Rs. 350 lakhs less

Rs. 210 lakhs received from advertisers)

Bank A/c Dr. 150.00

To Advance for advertisement time (sale) A/c 150.00

(Being advance received from advertisers

in respect of booking of balance 30% time)

May Advertisement rights (purchase) A/c Dr. 62.50

To Advance for advertisement rights (purchase) A/c 62.50

(Being cost of advertisement rights used in May

i.e., 25% of Rs. 250 lakhs, adjusted against advance

paid)

Advance for advertisement time (sale) A/c Dr. 125.00

To Advertisement time (sale) A/c 125.00

(Being sale price of advertisement time in May i.e.,

25% of Rs. 500 lakhs adjusted, against advance

received from advertisers)

Profit and Loss A/c Dr. 62.50

To Advertisement rights (purchase) A/c 62.50

8.45

(Being cost of advertisement rights debited to Profit

and Loss Account in May)

Advertisement time (sale) A/c Dr. 125.00

To Profit and Loss A/c 125.00

(Being revenue recognised in Profit and Loss

Account in May)

June Advertisement rights (purchase) A/c Dr. 187.50

To Advance for advertisement rights (purchase) 187.50

A/c

(Being cost of advertisement rights used in June, i.e.,

75% of Rs. 250 lakhs, adjusted against

advance paid)

Advance for advertisement time (sale) A/c Dr. 375.00

To Advertisement time (sale) A/c 375.00

(Being sale price of advertisement time availed in

June i.e., 75% of Rs. 500 lakhs, adjusted against

advance received from advertisers)

June Profit and Loss A/c Dr. 187.50

To Advertisement rights (purchase) A/c 187.50

(Being cost of advertisement rights used in June,

debited to Profit and Loss Account in June)

Advertisement time (sale) A/c Dr. 375.00

To Profit and Loss Account 375.00

(Being revenue recognised in June)

(ii) Monthly financial statements

(1) Revenue statement (Rs. in lakhs)

March April May June

Rs. Rs. Rs. Rs.

Sale of advertisement time – – 125.00 375.00

Less: Purchase of advertisement rights – – 62.50 187.50

Netprofit – – 62.50 187.50

(2) Balance sheet as at 31.3.99 30.4.99 31.5.99 30.6.99

Sources of funds:

Net profit – – 62.50 250.00

Application of funds:

Current assets, loans and advances:

Advance for advertisement rights 150.00 250.00 187.50 –

8.46

Bank Balance 60.00 250.00 250.00 250.00

210.00 500.00 437.50 250.00

Less: Current liabilities

Advance for advertisement time

(received from advertisers) 210.00 500.00 375.00 –

Net current assets – – 62.50 250.00

As per para 7.1 of AS 9 on Revenue Recognition, under proportionate completion method, revenue from service transactions is recognised proportionately by reference to the performance of each act where performance consists of the execution of more than one act. Therefore, income from advertisement is recognised in May, 1999 (25%) and June, 1999 (75%) in the proportion of availability of the advertisement time.

Question 13

(a) Describe the factors for determination of “Reportable Segments” as per AS-17.

(b) Briefly describe the disclosure requirements for related party transactions as per Accounting Standard 18.

(c) State the different types of Leases contemplated in Accounting Standard 19 and discuss briefly. (12 marks) (Intermediate–May 2002)

Answer

(a) Paragraphs 27 to 29 of AS 17 on Segment Reporting deals with reportable segments.

Paragraph 27 requires that a business segment or geographical segment should be identified as a reportable segment if :

(i) its revenue from sales to external customers and from transactions with other segments is 10 percent or more of the total revenue, external and internal, of all segments; or

(ii) its segment result, whether profit or loss, is 10 percent or more of-

(a) the combined result of all segments in profit, or

(b) the combined result of all segments in loss, whichever is greater in absolute amount; or

(iii) its segment assets are 10 percent or more of the total assets of all segments.

A business segment or a geographical segment which is not a reportable segment as per paragraph 27, may be designated as a reportable segment despite its size at the discretion of the management of the enterprise. If that segment is not designated as a reportable segment, it should be included as an unallocated reconciling item.

If total external revenue attributable to reportable segments constitutes less than 75% of the total enterprise revenue, additional segments should be identified as reportable segments, even if they do not meet the 10 percent thresholds specified in paragraph 27 of the standard, until at least 75 percent of the total enterprise revenue is included in reportable segments.

(b) Paragraph 23 of AS 18 on Related Party Disclosures requires that if there have been transactions between related parties, during the existence of the a related party relationship, the reporting enterprise should disclose the following :

(i) the name of the transacting related party;

(ii) a description of the relationship between the parties;

(iii) a description of the nature of transactions;

(iv) volume of the transactions either as an amount or as an appropriate proportion;

8.47

(v) any other elements of the related party transactions necessary for an understanding of the financial statements;

(vi) the amounts or appropriate proportions of outstanding items pertaining to related parties at the balance sheet date and provisions for doubtful debts due from such parties at that date;

(vii) amounts written off or written back in the period in respect of debts due from or to related parties.

Point (v) requires disclosure of ‘any other elements of the related party transactions necessary for an understanding of the financial statements. An example of such a disclosure would be an indication that the transfer of a major asset had taken place at an amount materially different from that obtainable on normal commercial terms.

(c) Accounting Standard 19 has divided the lease into two types viz. (i) Finance Lease and (ii) Operating Lease.

Finance Lease : A lease is classified as a finance lease if it transfers substantially all the risks and rewards incident to ownership. title may or may not eventually be transferred. At the inception of a finance lease, the lessee should recognise the lease as an asset and a liability. Such recognition should be at an amount equal to the fair value of the leased asset at the inception of the lease. However, if the fair value of the leased asset exceeds the present value of the minimum lease payments from the standpoint of the lessee, the amount recorded as an asset and liability should be the present value of the minimum lease payments from the standpoint of the lessee.

Operating Lease : A lease is classified as an operating lease if it does not transfer substantially all the risks and rewards incident to ownership. Lease payments under an operating lease should be recognised as an expense in the statement of profit and loss on a straight line basis over the lease term unless another systematic basis is more representative of the time pattern of the user’s benefit.

Question 14

(a) When Capitalisation of borrowing cost should cease as per Accounting Standard 16?

(b) Define a "Business Segment" and a "Geographical Segment" as per Accounting Standard 17.

(c) Briefly describe, how do you calculate "Diluted Earnings per Share" as per Accounting Standard 20.

(d) Briefly describe the disclosure requirements for "Deferred Tax Assets" and "Deferred Tax Liabilities" as per Accounting Standard 22.

(e) Write short note on Sale and Lease Back Transactions as per Accounting Standard 19.

( 20 marks) (PE-II – Nov. 2002)

Answer

(a) Capitalisation of borrowing costs should cease when substantially all the activities necessary to prepare the qualifying asset for its intended use or sale are complete.

An asset is normally ready for its intended use or sale when its physical construction or production is complete even though routine administrative work might still continue. If minor modifications such as the decoration of a property to the user’s specification, are all that are outstanding, this indicates that substantially all the activities are complete.

When the construction of a qualifying asset is completed in parts and a completed part is capable of being used while construction continues for the other parts, capitalisation of borrowing costs in relation to a part should cease when substantially all the activities necessary to prepare that part for its intended use or sale are complete.

(b) A Business Segment: A business segment is a distinguishable component of an enterprise that is engaged in providing an individual product or service or a group of related products or services and that is subject to risks and returns that are different from those of other business segments. Factors that should be considered in determining whether products or services are related include:

8.48

(a) the nature of the products or services;

(b) the nature of the production processes;

(c) the type or class of customers for the products or services;

(d) the methods used to distribute the products or provide the services and

(e) if applicable, the nature of the regulatory environment, for example, banking, insurance or public utilities.

A geographical segment: A geographical segment is a distinguishable component of an enterprise that is engaged in providing product or services within a particular economic environment and that is subject to risks and returns that are different from those of components operating in other economic environments. Factors that should be considered in identifying geographical segments include:

(a) similarity of economic and political conditions;

(b) relationships between operations in different geographical areas;

(c) proximity of operations;

(d) special risks associated with operations in a particular area;

(e) exchange control regulations; and

(f) the underlying currency risks.

(c) For the purpose of calculating diluted earnings per share, the net profit or loss for the period attributable to equity shareholders and the weighted average number of shares outstanding during the period should be adjusted for the effects of all dilutive potential equity shares.

The amount of net profit or loss for the period attributable to equity shareholders should be adjusted, after taking into account any attributable change in tax expense for the period.

The number of equity shares should be the aggregate of the weighted average number of equity shares (as per paragraphs 15 and 22 of AS 20) and the weighted average number of equity shares which would be issued on the conversion of all the dilutive potential equity shares into equity shares. Dilutive potential equity shares should be deemed to have been converted into equity shares at the beginning of the period or, if issued later, the date of the issue of the potential equity shares.

An enterprise should assume the exercise of dilutive options and other dilutive potential equity shares of the enterprise. The assumed proceeds from these issues should be considered to have been received from the issue of shares at fair value. The difference between the number of shares issuable and the number of shares that would have been issued at fair value should be treated as an issue of equity shares for no consideration.

(d) (i) An enterprise should offset deferred tax assets and deferred tax liabilities if:

(a) the enterprise has a legally enforceable right to set off assets against liabilities representing current tax, and

(b) the deferred tax assets and the deferred tax liabilities relate to taxes on income levied by the same governing taxation laws.

(ii) Deferred tax assets and liabilities should be distinguished from assets and liabilities representing current tax for the period. Deferred tax assets and liabilities should be disclosed under a separate heading in the balance sheet of the enterprise, separately from current assets and current liabilities.

(iii) The break-up of deferred tax assets and deferred tax liabilities into major components of the respective balances should be disclosed in the notes to accounts.

8.49

(iv) The nature of the evidence supporting the recognition of deferred tax assets should be disclosed, if an enterprise has unabsorbed depreciation or carry forward of losses under tax laws.

(e) Sale and leaseback transactions: As per AS 19 on ‘Leases’, a sale and leaseback transaction involves the sale of an asset by the vendor and the leasing of the asset back to the vendor. The lease payments and the sale price are usually interdependent, as they are negotiated as a package. The accounting treatment of a sale and lease back transaction depends upon the type of lease involved.

If a sale and leaseback transaction results in a finance lease, any excess or deficiency of sale proceeds over the carrying amount should be deferred and amortised over the lease term in proportion to the depreciation of the leased asset.

If sale and leaseback transaction results in a operating lease, and it is clear that the transaction is established at fair value, any profit or loss should be recognised immediately. If the sale price is below fair value any profit or loss should be recognised immediately except that, if the loss is compensated by future lease payments at below market price, it should be deferred and amortised in proportion to the lease payments over the period for which the asset is expected to be used. If the sale price is above fair value, the excess over fair value should be deferred and amortised over the period for which the asset is expected to be used.

Question 15

(a) X Co. Ltd. charged depreciation on its asset on SLM basis. For the year ended 31.3.2003 it changed to WDV basis. The impact of the change when computed from the date of the asset coming to use amounts to Rs. 20 lakhs being additional charge.

Decide how it must be disclosed in Profit and loss account. Also, discuss, when such changes in method of depreciation can be adopted by an enterprise as per AS–6.

(b) Decide when research and development cost of a project can be deferred to future periods as per AS 26.

(c) You are an accountant preparing accounts of A Ltd. as on 31.3.2003. After year end the following events have taken place in April, 2003:

(i) A fire broke out in the premises damaging, uninsured stock worth Rs. 10 lakhs (Salvage value Rs. 2 lakhs).

(ii) A suit against the company’s advertisement was filed by a party claiming damage of Rs. 20 lakhs.

(iii) Dividend proposed @ 20% on share capital of Rs. 100 lakhs.

Describe, how above will be dealt with in the account of the company for the year ended on 31.3.2003.

(d) How the government grants related to specific fixed assets should be presented in the Balance Sheet as per AS–12?

(e) Briefly describe the disclosure requirements for amalgamation including additional disclosure, if any, for different methods of amalgamation as per AS–14.

(f) Mention the prescribed accounting treatment in respect of gratuity benefits payable to employees as per AS–15. (24 marks) (PE-II – May 2003)

Answer

(a) The company should disclose the change in method of depreciation adopted for the accounting year. The impact on depreciation charge due to change in method must be quantified and reported by the enterprise.

Following aspects may be noted in this regard as per AS 6 on Depreciation Accounting.

(a) The depreciation method selected should be applied consistently from period to period.

8.50

(b) A change from one method of providing depreciation to another should be made only if the adoption of the new method is required by statute or for compliance with an accounting standard if it is considered that the change would result in a more appropriate preparation or presentation of the financial statements of the enterprise.

(c) When such a change in the method of depreciation is made, depreciation should be recalculated in accordance with the new method from the date of the asset coming into use. The deficiency or surplus arising from retrospective recomputation of depreciation in accordance with the new method should be adjusted in the accounts in the year in which the method of depreciation is changed.

(d) In case the change in the method results in deficiency in depreciation in respect of past years, the deficiency should be charged in the statement of profit and loss.

(e) In case the change in the method results in surplus, the surplus should be credited to the statement of profit and loss. Such a change should be treated as a change in accounting policy and its effect should be quantified and disclosed.

(b) As per para 41 of AS 26 ‘Intangible Assets’, no intangible asset arising from research should be recognized. The expenditure incurred on development phase can be deferred to the subsequent years if the company can demonstrate all of the following conditions (as specified in para 44 of AS 26 ‘Intangible Assets’):

(a) the technical feasibility of completing the intangible asset so that it will be available for use or sale;

(b) its intention to complete the intangible asset and use or sell it;

(c) its ability to use or sell the intangible asset;

(d) how the intangible asset will generate probable future economic benefits. Among other things, the enterprise should demonstrate the existence of a market for the output of the intangible asset or the intangible asset itself or, if it is to be used internally, the usefulness of the intangible asset;

(e) the availability of adequate technical, financial and other resources to complete the development and to use or sell the intangible asset; and

(f) its ability to measure the expenditure attributable to the intangible asset during its development reliably.

(c) Events occurring after the Balance Sheet date that represent material changes and commitments affecting the financial position of the enterprise must be disclosed according to para 15 of AS 4 on Contingencies and Events occurring after the Balance Sheet date. Hence, fire accident and loss thereof must be disclosed.

Suit filed against the company being a contingent liability must be disclosed with the nature of contingency, an estimate of the financial effect and uncertainties which may affect the future outcome must be disclosed as per para 16 of AS 4.

There are events which, although take place after the balance sheet date, are sometimes reflected in the financial statements because of statutory requirements or because of their special nature. Such items include the amount of dividend proposed or declared by the enterprise after the balance sheet date in respect of the period covered by the financial statements. Thus, dividends which are proposed or declared by the enterprise after the balance sheet date but before approval of the financial statements, should be adjusted as per para 14 of AS 4.

(d) Paragraphs 8 and 14 of AS 12 on Accounting for Government Grants deal with presentation of government grants related to specific fixed assets.

Government grants related to specific fixed assets should be presented in the balance sheet by showing the grant as a deduction from the gross value of the assets concerned in arriving at their book

8.51

value. Where the grant related to a specific fixed asset equals the whole, or virtually the whole, of the cost of the asset, the asset should be shown in the balance sheet at a nominal value. Alternatively, government grants related to depreciable fixed assets may be treated as deferred income which should be recognised in the profit and loss statement on a systematic and rational basis over the useful life of the asset, i.e., such grants should be allocated to income over the periods and in proportion in which depreciation on those assets is charged. Grants related to non-depreciable assets should be credited to capital reserve under this method. However, if a grant related to a non-depreciable asset requires the fulfillment of certain obligations, the grant should be credited to income over the same period over which the cost of meeting such obligations is charged to income. The deferred income balance should be separately disclosed in the financial statements.

(e) The disclosure requirements for amalgamations have been prescribed in paragraphs 43 to 46 of AS 14 on Accounting for Amalgamation.

For all amalgamations, the following disclosures should be made in the first financial statements following the amalgamation:

(a) names and general nature of business of the amalgamating companies;

(b) the effective date of amalgamation for accounting purpose;

(c) the method of accounting used to reflect the amalgamation; and

(d) particulars of the scheme sanctioned under a statute.

For amalgamations accounted under the pooling of interests method, the following additional disclosures should be made in the first financial statements following the amalgamation:

(a) description and number of shares issued, together with the percentage of each company’s equity shares exchanged to effect the amalgamation; and

(b) the amount of any difference between the consideration and the value of net identifiable assets acquired, and the treatment thereof.

For amalgamations, accounted under the purchase method, the following additional disclosures should be made in the first financial statements following the amalgamation;

(a) consideration for the amalgamation and a description of the consideration paid or contingently payable; and

(b) the amount of any difference between the consideration and the value of net identifiable assets acquired, and the treatment thereof including the period of amortisation of any goodwill arising on amalgamation.

(f) Accounting treatment in respect of gratuity benefits payable to employees has been prescribed under paragraph 28 of AS 15 on Accounting for Retirement Benefits in the Financial Statements of Employers.

Accounting treatment in respect of gratuity benefit and other defined benefit schemes will depend on the type of arrangement, which the employer has chosen to make.

(i) If the employer has chosen to make payment for retirement benefits out of his own funds, an appropriate charge to the statement of profit and loss for the year should be made through a provision for the accruing liability. The accruing liability should be calculated according to actuarial valuation. However, those enterprises which employ only a few persons may calculate the accrued liability by reference to any other rational method e.g., a method based on the assumption that such benefits are payable to all employees at the end of the accounting year.

(ii) In case the liability for retirement benefits is funded through creation of a trust, the cost incurred for the year should be determined actuarially. Such actuarial valuation should normally be conducted at least once in every three years. However, where actuarial valuation are not conducted annually, the actuary’s report should specify the contributions to be made by the employer on annual basis during the inter-valuation period. This annual contribution (which is in

8.52

addition to the contribution that may be required to finance unfunded past service cost) reflects proper accrual of retirement benefit cost for each of the years during the inter-valuation period and should be charged to the statement of profit or loss each year. Where the contribution paid during a year is lower than the amount required to be contributed during the year to meet the accrued liability as certified by the actuary, the shortfall should be charged to the statement of profit or loss for the year. Where the contribution paid during a year is in excess of the amount required to be contributed during the year to meet the accrued liability as certified by the actuary, the excess should be treated as a pre-payment.

(ii) In case the liability for retirement benefits is funded through a scheme administered by an insurer, an actuarial certificate or a confirmation from the insurer should be obtained that the contribution payable to the insurer is the appropriate accrual of the liability for the year. Where the contribution paid during a year is lower than the amount required to be contributed during the year to meet the accrued liability as certified by the actuary or confirmed by the insurer, as the case may be, the shortfall should be charged to the statement of profit or loss for the year. Where the contribution paid during a year is in excess of the amount required to be contributed during the year to meet the accrued liability as certified by the actuary or confirmed by the insurer, as the case may be, the excess should be treated as a pre-payment.

Question 16

(a) How is software acquired for internal use accounted for under AS-26?

(b) What are the principles for recognition of deferred taxes under AS-22?

(c) Define related party transaction under AS-18.

(d) A Limited company charged depreciation on its assets on the basis of W.D.V. method from the date of assets coming to use till date amounts to Rs. 32.23 lakhs. Now the company decides to switch over to Straight Line method of providing for depreciation. The amount of depreciation computed on the basis of S.L.M. from the date of assets coming to use till the date of change of method amounts to Rs. 20 lakhs.

Discuss as per AS-6, when such changes in method of can be adopted by the company and what would be the accounting treatment and disclosure requirement.

(e) X Limited has recognized Rs. 10 lakhs on accrual basis income from dividend on units of mutual funds of the face value of Rs. 50 lakhs held by it as at the end of the financial year 31st March, 2003. The dividends on mutual funds were declared at the rate of 20% on 15th June, 2003. The dividend was proposed on 10th April, 2003 by the declaring company. Whether the treatment is as per the relevant Accounting Standard? You are asked to answer with reference to provisions of Accounting Standard.

(20 marks) (PE-II – Nov. 2003)

Answer

(a) Paragraphs 10 and 11 of Appendix A to the Accounting Standard 26 on Intangible Assets, lays down the following procedure for accounting of software acquired for internal use:-

The cost of a software acquired for internal use should be recognised as an asset if it meets the recognition criteria prescribed in paragraphs 20 and 21 of this statement.

The cost of a software purchased for internal use comprises its purchase price, including any import duties and other taxes (other than those subsequently recoverable by the enterprise from the taxing authorities) and any directly attributable expenditure on making the software ready for its use.

Any trade discounts and rebates are deducted in arriving at the cost. In the determination of cost, matters stated in paragraphs 24 to 34 of the Statement which deal with the method of accounting for ‘Separate Acquisitions’, ‘Acquisitions as a part of Amalgamations’, Acquisitions by way of Government Grant’, and ‘Exchanges of Assets’, need to be considered, as appropriate.

8.53

Recognition criteria as per paragraphs 20 and 21 of the standard are stated below:-

An intangible asset should be recognised if, and only if:

(a) it is probable that the future economic benefits that are attributable to the asset will flow to the enterprise; and

(b) the cost of the asset can be measured reliably.

An enterprise should assess the probability of future economic benefits using reasonable and supportable assumptions that represent best estimate of the set of economic conditions that will exist over the useful life of the asset.

(b) Taxable income is calculated in accordance with tax laws. In some circumstances the requirements of these laws to compute taxable income differ from the accounting policies applied to determine accounting income. This results in a difference between the taxable and the accounting income. Such differences are classified into Permanent and Timing differences. The tax effect of the timing differences is known as Deferred Tax and is included as tax expense in the statement of profit and loss and as deferred tax assets or as deferred tax liabilities, in the balance sheet.

Prudence would dictate that deferred tax liabilities are provided for without exception, even in situations where an enterprise is incurring losses. Deferred tax assets should be recognized and carried forward only to the extent that there is reasonable certainty that sufficient future taxable income will be available against which such deferred tax asset can be realized. Reasonable certainty can be demonstrated by providing robust and realistic estimates of profits for the future. A company with a track record of losses with no immediate visibility of a turnaround should not recognise a deferred tax asset as a matter of prudence. In the case of an unabsorbed depreciation and carry forward losses under the tax laws, the recognition principles are more stricter, i.e. deferred tax asset should be recognized only to the extent that there is virtual certainty supported by convincing evidence that sufficient future taxable income will be available against which such deferred tax asset can be realized. The existence of unabsorbed depreciation or carry forward of losses under tax laws is strong evidence that future taxable income may not be available.

In that situation there has to be convincing evidence that sufficient future taxable income will be available against which such deferred tax asset can be realized. This is a matter of judgement and the conclusion would depend on facts and circumstances of each case.

(c) Accounting Standard 18 on Related Party Disclosures defines a related party transaction as transfer of resources or obligations between related parties, regardless of whether or not a price is charged.

Related parties have been defined by the standard in the following words. “Parties are considered to be related if at any time during the reporting period one party has the ability to control the other party or exercise significant influence over the other party in making financial and/or operating decisions.”

Further, paragraph 24 of the Standard gives certain examples of related party transactions in respect of which disclosures may be made by a reporting enterprise. Those examples are listed below:-

(a) purchases or sales of goods (finished or unfinished);

(b) purchases or sales of fixed assets;

(c) rendering or receiving of services;

(d) agency arrangements;

(e) leasing or hire purchase arrangements;

(f) transfer of research and development;

(g) license agreements;

(h) finance (including loans and equity contributions in cash or in kind);

(i) guarantees and collaterals; and

8.54

(j) management contracts including for deputation of employees.

(d) Paragraph 21 of Accounting Standard 6 on Depreciation Accounting says, "The depreciation method selected should be applied consistently from period to period. A change from one method of providing depreciation to another should be made only if the adoption of the new method is required by statute or for compliance with an accounting standard or if it is considered that the change would result in a more appropriate preparation or presentation of the financial statements of the enterprise."

The paragraph also mentions the procedure to be followed when such a change in the method of depreciation is made by an enterprise. As per the said paragraph, depreciation should be recalculated in accordance with the new method from the date of the asset coming to use. The difference in the amount, being deficiency or surplus from retrospective recomputation should be adjusted in the profit and loss account in the year such change is effected. Since such a change amounts to a change in the accounting policy, it should be properly quantified and disclosed. In the question given, the surplus arising out of retrospective recomputation of depreciation as per the straight line method is Rs. 12.23 lakhs (Rs. 32.23 lakhs – Rs. 20 lakhs). This should be written back to Profit and Loss Account and should be disclosed accordingly.

(e) Paragraph 8.4 and 13 of Accounting Standard 9 on Revenue Recognition states that dividends from investments in shares are not recognised in the statement of profit and loss until a right to receive payment is established.

In the given case, the dividend is proposed on 10th April, 2003, while it is declared on 15th June, 2003. Hence, the right to receive payment is established on 15th June, 2003. As per the above mentioned paragraphs, income from dividend on units of mutual funds should be recognised by X Ltd. in the financial year ended 31st March, 2004.

The recognition of Rs. 10 lakhs on accrual basis in the financial year 2002-2003 is not as per AS 9 'Revenue Recognition'.

(i) Acting as a banker in respect of funds of local bodies, Zilla Parishads, Panchayat Institutions etc. who keep their funds with the treasuries.

(ii) Custody of opium and other valuables because of the strong room facility provided at the treasury.

(iii) Custody of cash balances of the State Government and conducting cash business of Government at non-banking treasuries.

Question 17

(a) X Ltd. received a grant of Rs. 2 crores from the Central Government for the purpose of a special Machinery during 1998-99. The cost of Machinery was Rs. 20 crores and had a useful life of 9 years. During 2002-03, the grant has become refundable due to non-fulfillment of certain conditions attached to it. Assuming the entire grant was deducted from the cost of Machinery in the year of acquisition. State with reasons, the accounting treatment to be followed in the year 2002-03.

(b) The company deals in three products, A, B and C, which are neither similar nor interchangeable. At the time of closing of its account for the year 2002-03. The Historical Cost and Net Realizable Value of the items of closing stock are determined as follows:

Items Historical Cost (Rs. in lakhs)

Net Realisable Value (Rs. in lakhs)

A 40 28

B 32 32

C 16 24

What will be the value of Closing Stock?

8.55

(c) During the current year 20022003, X Limited made the following expenditure relating to its plant building:

Rs. in lakhs

Routine Repairs 4

Repairing 1

Partial replacement of roof tiles 0.5

Substantial improvements to the electrical wiring system which will increase efficiency

10

What amount should be capitalized?

(d) A plant was depreciated under two different methods as under:

Year SLM

(Rs. in lakhs)

W.D.V.

(Rs. in lakhs)

1 7.80 21.38

2 7.80 15.80

3 7.80 11.68

4 7.80 8.64

31.20 57.50

5 7.80 6.38

What should be the amount of resultant surplus/deficiency, if the company decides to switch over from W.D.V. method to SLM method for first four years? Also state, how will you treat the same in Accounts.

(e) Briefly explain the methods of accounting for amalgamation as per Accounting Standard-14. (20 marks) (PE-II – May 2004)

Answer

(a) As per para 11.3 of AS 12 on Accounting for Government Grants, the amount refundable in respect of a government grant related to a specific fixed asset is recorded by increasing the book value of the asset. Depreciation on the revised book value is provided prospectively over the residual useful life of the asset. In the given case, book value of machinery will be increased by Rs. 2 crores in the year 2002-2003. The computations for the depreciation on machinery can be given as:

Cost of machinery Rs. 20 crores

Less: Grant received Rs. 2 crores

Cost of machinery Rs. 18 crores

Useful life of machinery 9 years

Depreciation per year as per straight line method Rs. 18 crores/9

(assuming residual value to be zero) = Rs. 2 crores

Total depreciation for 4 years (1998-99 to 2001-2002) Rs. 8 crores

Book value (in year 2002-2003) Rs. 10 crores

Add: Grant refunded Rs. 2 crores

Revised book value Rs. 12 crores

Remaining useful life 5 years

Revised annual depreciation Rs. 12 crores/5

8.56

= 2.4 crores

Thus, book value of machinery will be Rs. 12 crores in the year 2002-2003 and the depreciation amounting Rs. 2.4 crores will be charged on machinery. Annual depreciation of Rs. 2.4 crores will be charged in the next four years.

(b) As per para 5 of AS 2 on Valuation of Inventories, inventories should be valued at the lower of cost and net realizable value. Inventories should be written down to net realizable value on an item-by-item basis in the given case.

Items Historical Cost

(Rs. in lakhs)

Net Realisable Value (Rs. in lakhs)

Valuation of closing stock (Rs. in lakhs)

A 40 28 28

B 32 32 32

C 16 24 16

88 84 76

Hence, closing stock will be valued at Rs. 76 lakhs.

(c) As per para 12.1 of AS 10 on Accounting for Fixed Assets, expenditure that increases the future benefits from the existing asset beyond its previously assessed standard of performance is included in the gross book value, e.g., an increase in capacity. Hence, in the given case, Repairs amounting Rs. 5 lakhs and Partial replacement of roof tiles should be charged to profit and loss statement. Rs. 10 lakhs incurred for substantial improvement to the electrical writing system which will increase efficiency should be capitalized.

(d) As per para 21 of AS 6 on Depreciation Accounting, when a change in the method of depreciation is made, depreciation should be recalculated in accordance with the new method from the date of the asset coming into use. The deficiency or surplus arising from retrospective recomputation of depreciation in accordance with the new method should be adjusted in the accounts in the year in which the method of depreciation is changed. In the given case, there is a surplus of Rs. 26.30 lakhs on account of change in method of depreciation, which will be credited to Profit and Loss Account. Such a change should be treated as a change in accounting policy and its effect should be quantified and disclosed.

(e) As per AS 14 on ‘Accounting for Amalgamations’, there are two main methods of accounting for amalgamations:

(i) The Pooling of Interest Method

Under this method, the assets, liabilities and reserves of the transferor company are recorded by the transferee company at their existing carrying amounts (after making the necessary adjustments).

If at the time of amalgamation, the transferor and the transferee companies have conflicting accounting policies, a uniform set of accounting policies is adopted following the amalgamation. The effects on the financial statements of any changes in accounting policies are reported in accordance with AS 5 on ‘Net Profit or Loss for the Period, Prior Period Items and Changes in Accounting Policies’.

(ii) The Purchase Method

Under the purchase method, the transferee company accounts for the amalgamation either by incorporating the assets and liabilities at their existing carrying amounts or by allocating the consideration to individual identifiable assets and liabilities of the transferor company on the basis of their fair values at the date of amalgamation. The identifiable assets and liabilities may include assets and liabilities not recorded in the financial statements of the transferor company.

Where assets and liabilities are restated on the basis of their fair values, the determination of fair values may be influenced by the intentions of the transferee company.

8.57

Question 18

(a) On 20.4.2003 JLC Ltd. obtained a loan from the Bank for Rs. 50 lakhs to be utilised as under:

Rs.

Construction of a shed 20 lakhs

Purchase of machinery 15 lakhs

Working capital 10 lakhs

Advance for purchase of truck 5 lakhs

In March, 2004 construction of shed was completed and machinery installed. Delivery of truck was not received. Total interest charged by the bank for the year ending 31.3.2004 was Rs. 9 lakhs. Show the treatment of interest under AS 16.

(b) A limited company created a provision for bad and doubtful debts at 2.5% on debtors in preparing the financial statements for the year 2003-2004.

Subsequently on a review of the credit period allowed and financial capacity of the customers, the company decided to increase the provision to 8% on debtors as on 31.3.2004. The accounts were not approved by the Board of Directors till the date of decision. While applying the relevant accounting standard can this revision be considered as an extraordinary item or prior period item?

(c) Explain the treatment of cost arising from alteration in retirement benefit cost as per AS 15. (12 marks) (PE-II – Nov. 2004)

Answer

(a) As per AS 16, borrowing costs that are directly attributable to the acquisition, construction or production of a qualifying asset should be capitalized. A qualifying asset is an asset that necessarily takes a substantial period of time (usually 12 months or more) to get ready for its intended use or sale. If an asset is ready for its intended use or sale at the time of its acquisition then it is not treated as a qualifying asst for the purposes of AS 16.

Treatment of interest as per AS 16

Particulars Nature Interest to be capitalized Interest to be charged to profit and loss account

(1) Construction of a shed

Qualifying asset

lakhs 50 Rs.lakhs 20 Rs.

lakhs 9 Rs.

= Rs. 3.60 lakhs

(2) Purchase of machinery

Not a qualifying asset

lakhs 50 Rs.lakhs 15 Rs.

lakhs 9 Rs. =

Rs. 2.70 lakhs.

(3) Working capital

Not qualifying asset

lakhs 50 Rs.lakhs 10 Rs.

lakhs 9 Rs. =

Rs. 1.80 lakhs

(4) Advance for purchase of truck

Not a qualifying asset

lakhs 50 Rs.lakhs 5 Rs.

lakhs 9 Rs. =

Rs. 0.90 lakhs

Total Rs.3.60 lakhs Rs.5.40 lakhs

On the basis that machinery is ready for its intended use at the time of its acquisition/purchase.

8.58

(b) The preparation of financial statements involve making estimates which are based on the circumstances existing at the time when the financial statements are prepared. It may be necessary to revise an estimate in a subsequent period if there is a change in the circumstances on which the estimate was based. Revision of an estimate, by its nature, does not bring the adjustment within the definitions of a prior period item or an extraordinary item [para 21 of AS 5 (Revised) on Net Profit or Loss for the Period, Prior Period Items and Changes in Accounting Policies].

In the given case, a limited company created 2.5% provision for doubtful debts for the year 2003-2004. Subsequently in 2004 they revised the estimates based on the changed circumstances and wants to create 8% provision. As per AS-5 (Revised), this change in estimate is neither a prior period item nor an extraordinary item.

However, as per para 27 of AS 5 (Revised), a change in accounting estimate which has material effect in the current period, should be disclosed and quantified. Any change in the accounting estimate which is expected to have a material effect in later periods should also be disclosed.

(c) Alteration in the retirement benefit cost may arise from introduction of a retirement benefit scheme for existing employees or because of making of improvements to an existing scheme. As per AS 15 any alternation in retirement benefit cost arising from changes in the actuarial method used or assumptions adopted should be charged or credited to the statement of profit or loss as they arise in accordance with AS 5 “Net Profit or Loss for the Period, Prior Period Items and Changes in Accounting Policies”. Additionally, a change in the actuarial method should be treated as a change in accounting policy and disclosed in accordance with AS 5. The cost of additional benefits provided to retired employees due to amendments in the retirement benefit scheme should also be treated in the same manner (i.e. charged to profit and loss statement of the year).

Question 19

(a) A major fire has damaged assets in a factory of X Co. Ltd. on 8.4.2004, 8 days after the year end closing of accounts. The loss is estimated to be Rs. 16 crores (after estimating the recoverable amount of Rs. 24 crores from the Insurance Company).

If the company had no insurance cover, the loss due to fire would be Rs. 40 crores.

Explain, how the loss should be treated in the Final accounts of the year ended 31.3.2004.

(b) A Company had deferred research and development cost of Rs. 150 lakhs. Sales expected in the subsequent years are as under:

Years Sales (Rs. in lakhs)

I 400

II 300

III 200

IV 100

You are asked to suggest how should Research and Development cost be charged to Profit and Loss account.

If at the end of the III year, it is felt that no further benefit will accrue in the IV year, how the unamortised expenditure would be dealt with in the accounts of the Company?

(c) In April, 2004 a Limited Company issued 1,20,000 equity shares of Rs. 100 each. Rs. 50 per share was called up on that date which was paid by all shareholders. The remaining Rs. 50 was called up on 1.9.2004. All shareholders paid the sum in September, 2004, except one shareholder having 24,000 shares. The net profit for the year ended 31.3.2005 is Rs. 2,64,000 after dividend on preference shares and dividend distribution tax of Rs. 64,000.

Compute basic EPS for the year ended 31.3.2005 as per Accounting Standard 20.

8.59

(d) (i) Mr. Raj a relative of key Management personnel received remuneration of Rs. 2,50,000 for his services in the company for the period from 1.4.2004 to 30.6.2004. On 1.7.2004 he left the service.

Should the relative be identified as at the closing date i.e. on 31.3.2005 for the purposes of AS 18?

(ii) X Ltd. sold goods to its associate Company for the 1st quarter ending 30.6.2004. After that, the related party relationship ceased to exist. However, goods were supplied as was supplied to any other ordinary customer. Decide whether transactions of the entire year has to be disclosed as related party transaction.

(e) On 1.4.2001 ABC Ltd. received Government grant of Rs. 300 lakhs for acquisition of a machinery costing Rs. 1,500 lakhs. The grant was credited to the cost of the asset. The life of the machinery is 5 years. The machinery is depreciated at 20% on WDV basis. The Company had to refund the grant in May 2004 due to non-fulfillment of certain conditions.

How you would deal with the refund of grant in the books of ABC Ltd.?

(4 marks each) (PE-II – May 2005 Answer

(a) The present event does not relate to conditions existing at the balance sheet date. Hence, no specific adjustment is required in the financial statements for the year ending on 31.3.2004. But if the event occurring after balance sheet date gives an indication that the enterprise may cease to be a going concern, then the assets and liabilities are required to be adjusted for the financial year ended 31st March, 2004. AS 4 (Revised) requires disclosure in respect of events occurring after the balance sheet date representing unusual changes affecting the existence or substratum of the enterprise after the date of the Balance Sheet. In the present event, the loss of assets in a factory can be considered to be an event affecting the substratum of the enterprise. Hence, an appropriate disclosure should be made in the report of the approving authority.

(b) (i) Based on sales, research and development cost to be allocated as follows:

Year Research and Development cost allocation (Rs. in lakhs)

I 60 150

1,000400

II 45 150

1,000300

III 30 150

1,000200

IV 15 150

1,000100

(ii) If at the end of the III year, the circumstances do not justify that further benefit will accrue in IV year, then the company has to charge the unamortised amount i.e. remaining Rs. 45 lakhs [150 – (60 + 45)] as an expense immediately.

Note: As per para 41 of AS 26 on Intangible Assets, expenditure on research (or on the research phase of an internal project) should be recognized as an expense when it is incurred. It has been assumed in the above solution that the entire cost of Rs. 150 lakhs is development cost. Therefore, the expenditure has been deferred to the subsequent years on the basis of presumption that the company can demonstrate all the conditions specified in para 44 of AS 26. An intangible asset should be derecognised when no future economic benefits are expected from its use according to para 87 of

8.60

the standard. Hence the remaining unamortised amount of Rs. 45,00,000 has been written off as an expense at the end of third year.

(c) Basic earnings per share (EPS) = year the during goutstandin sharesequity of number average Weighted

rsshareholdeequity to leattributabprofit Net

= 3 Rs. note) workingin calculated (as shares 88,000

2,64,000 Rs.

Working Note:

Calculation of weighted average number of equity shares

Number of shares Nominal value of shares Amount paid

1st April, 2004 1,20,000 100 50

1st September, 2004 96,000 100 100

24,000 100 50

As per para 19 of AS 20 on Earnings per share, Partly paid equity shares are treated as a fraction of equity share to the extent that they were entitled to participate in dividends relative to a fully paid equity share during the reporting period. Assuming that the partly paid shares are entitled to participate in the dividends to the extent of amount paid, weighted average number of shares will be calculated as:

Shares

125

21

1,20,000 = 25,000

127

96,000 = 56,000

127

21

24,000 = 7,000

88,000 shares

(d) (i) According to para 10 of AS 18 on Related Party Disclosures, parties are considered to be related if at any time during the reporting period one party has the ability to control the other party or exercise significant influence over the other party in making financial and/or operating decisions. Hence, Mr. Raj, a relative of key management personnel should be identified as relative as at the closing date i.e. on 31.3.2005.

(ii) As per para 23 of AS 18, transactions of X Ltd. with its associate company for the first quarter ending 30.06.2004 only are required to be disclosed as related party transactions. The transactions for the period in which related party relationship did not exist need not be reported.

(e) According to para 21 of AS 12 on Accounting for Government Grants, the amount refundable in respect of a grant related to a specific fixed asset should be recorded by increasing the book value of the asset or by reducing the capital reserve or deferred income balance, as appropriate, by the amount refundable. In the first alternative, i.e., where the book value is increased, depreciation on the revised book value should be provided prospectively over the residual useful life of the asset. The accounting treatment in both the alternatives can be given as follows:

Alternative 1:

Rs. (in lakhs) 1st April, 2001 Acquisition cost of machinery (Rs. 1,500 – 300) 1,200.00 31st March, 2002 Less: Depreciation @ 20% 240.00 Book value 960.00

8.61

31st March, 2003 Less: Depreciation @ 20% 192.00 Book value 768.00 31st March, 2004 Less: Depreciation @ 20% 153.60 1st April, 2004 Book value 614.40 May, 2004 Add: Refund of grant 300.00 Revised book value 914.40

Depreciation @ 20% on the revised book value amounting Rs. 914.40 lakhs is to be provided prospectively over the residual useful life of the asset i.e. years ended 31st March, 2005 and 31st March, 2006.

Alternative 2:

ABC Ltd. can also debit the refund amount of Rs. 300 lakhs in capital reserve of the company.

Question 20

(a) ABC Ltd. could not recover Rs. 10 lakhs from a debtor. The company is aware that the debtor is in great financial difficulty. The accounts of the company were finalized for the year ended 31.3.2005 by making a provision @ 20% of the amount due from the said debtor.

The debtor became bankrupt in April, 2005 and nothing is recoverable from him.

Do you advise the company to provide for the entire loss of Rs. 10 lakhs in the books of account for the year ended 31st March, 2005?

(b) X Co. Ltd. signed an agreement with its employees union for revision of wages in June, 2004. The wage revision is with retrospective effect from 1.4.2000. The arrear wages upto 31.3.2004 amounts to Rs. 80 lakhs. Arrear wages for the period from 1.4.2004 to 30.06.2004 (being the date of agreement) amounts to Rs. 7 lakhs.

Decide whether a separate disclosure of arrear wages is required.

(c) An intangible asset appears in Balance Sheet of A Co. Ltd. at Rs. 16 lakhs as on 31.3.2004. The asset was acquired for Rs. 40 lakhs in April, 1991. The Company has been amortising the asset value on straight line basis. The policy is to amortise for 20 years.

Do you advise the Company to amortise the entire asset value in the books of the company as on 31.3.2004?

(d) Ram Co. (P) Ltd. furnishes you the following information for the year ended 31.3.2005:

Depreciation for the year ended 31.3.2005 (under straight line method)

Rs. 100 lakhs

Depreciation for the year ended 31.3.2005 (under written down value method)

Rs. 200 lakhs

Excess of depreciation for the earlier years calculated under written down value method over straight line method

Rs. 500 lakhs

The Company wants to change its method of claiming depreciation from straight line method to written down value method.

Decide, how the depreciation should be disclosed in the Financial Statement for the year ended 31.3.2005.

(e) How refund of revenue grant received from the Government is disclosed in the Financial Statements? (4 Marks each) (PE-II – Nov. 2005)

Answer

8.62

(a) As per AS 4 ‘Contingencies and Events occurring after the Balance Sheet Date’, adjustments to assets and liabilities are required for events occurring after the balance sheet date that provide additional information materially affecting the determination of the amounts relating to conditions existing at the Balance Sheet date.

In the given case, bankruptcy of the debtor in April, 2005 and consequent non-recovery of debt is an event occurring after the balance sheet date which materially affects the determination of profits for the year ended 31.3.2005. Therefore, the company should be advised to provide for the entire amount of Rs. 10 lakhs according to para 8 of AS 4.

(b) It is given that revision of wages took place in June, 2004 with retrospective effect from 1.4.2000. The arrear wages payable for the period from 1.4.2000 to 30.6.2004 cannot be taken as an error or omission in the preparation of financial statements and hence this expenditure cannot be taken as a prior period item.

Additional wages liability of Rs. 87 lakhs (from 1.4.2000 to 30.6.2004) should be included in current year’s wages.

It may be mentioned that additional wages is an expense arising from the ordinary activities of the company. Although abnormal in amount, such an expense does not qualify as an extraordinary item. However, as per Para 12 of AS 5 (Revised),’ Net Profit or loss for the Period, Prior Period Items and Changes in the Accounting Policies’, when items of income and expense within profit or loss from ordinary activities are of such size, nature or incidence that their disclosure is relevant to explain the performance of the enterprise for the period, the nature and amount of such items should be disclosed separately.

However, wages payable for the current year (from 1.4.2004 to 30.6.2004) amounting Rs. 7 lakhs is not a prior period item, hence need not be disclosed separately. This may be shown as current year wages.

(c) AS 26 ‘Intangible Assets’, came into effect for accounting periods commencing on or after 1.4.2003 and is mandatory in nature. Para 67 of the standard provides that if there is persuasive evidence that the life of the intangible asset is 20 years, then no adjustment is required at 1.4.2003. However, para 63 of the standard states that if it cannot be demonstrated that the life of the intangible asset is greater than 10 years, then AS 26 would require the asset to be amortised over not more than 10 years. Since, in the given case, the amortisation period determined by applying para 63 has already expired as on 1.4.2003, the carrying amount of Rs. 16 lakhs would be required to be eliminated with a corresponding adjustment to the opening balance of revenue reserves as on 1.4.2003.

(d) As per para 21 of AS 26 ‘Intangible Assets’, when a change in the method of depreciation is made, depreciation should be calculated in accordance with the new method from the date of the asset coming into use. The deficiency or surplus arising from retrospective recomputation should be adjusted in the accounts in the year in which the method of depreciation is changed. The deficiency should be charged to profit and loss account. Similarly, any surplus should be credited in the statement of profit and loss. Such change is a change in the accounting policy, and its effect should be quantified and disclosed.

In the given case, the deficiency of Rs. 500 lakhs would be charged to the profit and loss account of 31.3.2005. In the notes to account, the fact of change in method of depreciation should be elaborated along with the effect of Rs. 500 lakhs. The current depreciation charge of 200 lakhs determined in accordance with the written down value method should be debited to the profit and loss account.

(e) The amount refundable in respect of a grant related to revenue should be applied first against any unamortised deferred credit remaining in respect of the grant. To the extent that the amount

8.63

refundable exceeds any such deferred credit, or where no deferred credit exists, the amount should be charged to profit and loss statement. The amount refundable in respect of a grant related to a specific fixed asset should be recorded by increasing the book value of the asset or by reducing the capital reserve or the deferred income balance, as appropriate, by the amount refundable. In the first alternative, i.e., where the book value of the asset is increased, depreciation on the revised book value should be provided prospectively over the residual useful life of the asset.

Question 21

(a) X Co. Limited purchased goods at the cost of Rs.40 lakhs in October, 2005. Till March, 2006, 75% of the stocks were sold. The company wants to disclose closing stock at Rs.10 lakhs. The expected sale value is Rs.11 lakhs and a commission at 10% on sale is payable to the agent. Advise, what is the correct closing stock to be disclosed as at 31.3.2006.

(b) Explain the ‘Accounting of Revaluation of Assets’ with reference to AS 10.

(c) Arjun Ltd. sold farm equipments through its dealers. One of the conditions at the time of sale is, payment of consideration in 14 days and in the event of delay interest is chargeable @ 15% per annum. The Company has not realized interest from the dealers in the past. However, for the year ended 31.3.2006, it wants to recognise interest due on the balances due from dealers. The amount is ascertained at Rs.9 lakhs. Decide whether the income by way of interest from dealers is eligible for recognition as per AS 9.

(d) AB Ltd. launched a project for producing product X in October, 2004. The Company incurred Rs.20 lakhs towards Research and Development expenses upto 31st March, 2006. Due to prevailing market conditions, the Management came to conclusion that the product cannot be manufactured and sold in the market for the next 10 years. The Management hence wants to defer the expenditure write off to future years.

Advise the Company as per the applicable Accounting Standard.

(4 Marks each) (PE-II May 2006)

Answer

(a) As per Para 5 of AS 2 “Valuation of Inventories”, the inventories are to be valued at lower of cost and net realizable value.

In this case, the cost of inventory is Rs.10 lakhs. The net realizable value is 11,00,000 90% = Rs.9,90,000. So, the stock should be valued at Rs.9,90,000.

(b) As per Para 30 of AS 10 “Accounting for Fixed Assets”, an increase in net book value arising on revaluation of fixed assets should be credited to owner’s interests under the head of ‘revaluation reserve, except that, to the extent that such increase is related to and not greater than a decrease arising on revaluation previously recorded as a charge to the profit and loss statement, it may be credited to the profit and loss statement. A decrease in net book value arising on revaluation of fixed assets is charged directly to profit and loss statement except that to the extent such a decrease is related to an increase which was previously recorded as a credit to revaluation reserve and which has not been subsequently reversed or utilized , it may be charged directly to that account.

(c) As per AS 9 “Revenue Recognition”, where the ability to assess the ultimate collection with reasonable certainty is lacking at the time of raising any claim, the revenue recognition is postponed to the extent of uncertainty inverted. In such cases, the revenue is recognized only when it is reasonably certain that the ultimate collection will be made.

In this case, the company never realized interest for the delayed payments make by the dealers. Hence, it has to recognize the interest only if the ultimate collection is certain. The interest income hence is not to be recognized.

(d) As per Para 41 of AS 26 “Intangible Assets”, expenditure on research should be recognized as an expense when it is incurred. An intangible asset arising from development (or from the development phase of an internal project) should be recognized if, and only if, an enterprise can demonstrate all of the conditions specified in para 44 of the standard. An intangible asset (arising from development) should be derecognised when no future economic benefits are expected from its use according to para 87 of the

8.64

standard. Therefore, the manager cannot defer the expenditure write off to future years. Hence, the expenses amounting Rs. 20 lakhs incurred on the research and development project has to be written off in the current year ending 31st March, 2006.

8.65

Question 22

(a) What are the costs that are to be included in Research and Development costs as per AS 8.

(b) The Company reviewed an actuarial valuation for the first time for its Pension Scheme, which revalued a surplus of Rs.12 lacs. It wants to spread the same over the next 2 years by reducing the annual contribution to Rs.4 lacs instead of Rs.10 lacs. The average remaining life of the employees, if estimated to be 6 years, you are required to advise the Company considering the accounting standards 5 and 15.

(c) X Ltd. entered into an agreement to sell its immovable property included in the Balance Sheet at Rs.10 lacs to another company for Rs.15 lacs. The agreement to sell was concluded on 28th February, 2006 and the sale deed was registered on 1st May, 2006. Comment with reference to AS 4.

(d) Define related party transaction under AS 18. (4 Marks each) (PE-II- Nov. 2006)

Answer

(a) According to paras 41 and 43 of AS 26, “No intangible asset arising from research (or from the research phase of an internal project) should be recognized in the research phase. Expenditure on research (or on the research phase of an internal project) should be recognized as an expense when it is incurred.

Examples of research costs are:

Costs of activities aimed at obtaining new knowledge;

Costs of the search for, evaluation and final selection of, applications of research findings or other knowledge;

Costs of the search for alternatives for materials, devices, products, processes, systems or services; and

Costs of the activities involved in formulation, design, evaluation and final selection of possible alternatives for new or improved materials, devices, products, processes systems or services.”

According to paras 45 and 46 of AS 26, “In the development phase of a project, an enterprise can, in some instances, identify an intangible asset and demonstrate that future economic benefits from the asset are probable. This is because the development phase of a project is further advanced than the research phase.

Examples of development activities/costs are:

Costs of the design, construction and testing of pre-production or pre-use prototypes and models;

Costs of the design of tools, jigs, moulds and dies involving new technology;

Costs of the design, construction ad operation of a pilot plant that is not of a scale economically feasible for commercial production; and

Costs of the design, construction and testing of a chosen alternative for new or improved materials, devices, products, processes, systems or services.”

(b) According to para 92 of AS 15 (Revised 2005) on “Employee Benefits”, any actuarial gains and losses should be recognized immediately in the statement of profit and loss account as income or expense.

AS 8 stands withdrawn w.e.f. 1st April, 2003 i.e. the date from which AS 26 ‘Intangible Assets’ becomes

mandatory. Therefore the above answer has been given as per AS 26.

8.66

In the given case, the amount of surplus from pension scheme of Rs. 12 lacs is an actuarial gain, which should be recognized as income in the profit and loss account of the current year and not to be adjusted from the amount of annual contribution.

The surplus arising due to review of actuarial valuation of pension scheme by a company should be treated as a change in accounting policy and disclosed in accordance with AS 5(Revised).

(c) According to para 13 of AS 4 “Contingences and Events occurring after the Balance Sheet Date”, assets and liabilities should be adjusted for events occurring after the balance sheet date that provide additional evidence to assist the estimation of amounts relating to conditions existing at the balance sheet date.

In this case the sale of immovable property was carried out before the closure of the books of Accounts. This is clearly an event occurring after the balance sheet date. Agreement to sell was effected before the balance sheet date and the registration was done after the balance sheet date. So the adjustment for the sale of immovable property is necessary in the books of account for the year ended 31st March, 2006.

(d) According to AS 18, “Parties are considered to be related if at any time during the reporting period one party has the ability to control the other party or exercise significant influence over the other party in making financial and/or operating decisions.”

A related party transaction involves a transfer of resources or obligations between related parties, regardless of whether or not a price is charged.

Following are the examples of the related party transactions in respect of which disclosures may be made by a reporting enterprise:

Purchases or sales of goods (finished or unfinished);

Purchases or sales of fixed assets;

Rendering or receiving of services;

Agency arrangements;

Leasing or hire purchase arrangements;

Transfer of research and development;

Licence agreements;

Finance (including loans and equity contributions in cash or in kind);

Guarantees and collateral etc.

Management contracts including for deputation of employees.

Question 23

(a) What are the disclosure requirements of AS-7 (Revised)?

(b) How would you treat the Government grant received relating to a depreciable asset under the following cases as per AS-12?

Case i: Gross value of asset Rs.2 crores and Grant received Rs.20 lakhs only.

Case ii: Gross value of asset Rs.2 crores and Grant received Rs.2 crores.

(c) Explain the concept of actuarial valuation.

(d) What are the information that are to be disclosed in the financial statements as per AS-10? (4x4= 16 Marks) (PE II- May, 2007)

8.67

Answer

(a) According to paragraphs 38, 39 and 41 of AS 7, an enterprise should disclose:

(a) the amount of contract revenue recognized as revenue in the period;

(b) the methods used to determine the contract revenue recognized in the period; and

(c) the methods used to determine the stage of completion of contracts in progress.

In case of contract still in progress the following disclosures are required at the reporting date:

(a) the aggregate amount of costs incurred and recognised profits (less recognised losses) upto the reporting date;

(b) the amount of advances received; and

(c) the amount of retentions.

An enterprise should also present:

(a) the gross amount due from customers for contract work as an asset; and

(b) the gross amount due to customers for contract work as a liability.

(b) In accordance with AS 12, government grants related to specific fixed assets should be presented in the balance sheet by showing the grant as a deduction from the gross value of the assets concerned in arriving at their book value. Where the grant related to a specific fixed asset equals the whole, or virtually the whole, of the cost of the asset, the asset should be shown in the balance sheet at a nominal value.

Alternatively, government grants related to depreciable fixed assets may be treated as deferred income which should be recognized in the profit and loss statement on a systematic and rational basis over the useful life of the asset, i.e., such grants should be allocated to income over the periods and in the proportions in which depreciation on those assets is charged.

Case i

Grant received amounting Rs.20 lakhs is required to be deducted from Rs.2 crores. The balance of Rs.1.80 crores to be shown as an assest in the Balance Sheet and depreciation should also be charged on Rs.1.80 crores.

Case ii

As the grant is received for the entire cost of the asset, the asset shall be recorded at a nominal value of Rs.100 in the Balance sheet so that the existence of the amount is reflected. No depreciation is to be charged in this case.

Note: Alternatively, in both the cases government grant may be treated as deferred income which should be recognized in the profit and loss statement on a systematic and rational basis over the useful life of the asset.

(c) Actuarial valuation is the process used by an actuary to estimate the present value of benefits to be paid under a retirement scheme and the present values of the scheme assets and, sometimes, of future contributions. In the case of defined benefit scheme the cost of retirement benefits, to be charged to Profit and Loss Account on year to year basis, is determined on actuarial basis. According to paragraph 65 of AS 15 (revised 2005), an enterprise should use the Projected Unit Creditmethod

Actuary is an expert person who can calculate the liability where the factors affecting the calculation of liability are uncertain and cannot be determined in ordinary course. Projected Unit Credit method (sometimes known as the accrued benefit method pro-rated on service or as the benefit/years of service method) considers each period of service as giving rise to an additional unit of benefit entitlement and measures each unit separately to build the final obligation.

8.68

to determine the present value of its defined benefit obligations and the related current service cost and, wherever applicable, past service cost.

8.69

(d) As per AS 10, the following information should be disclosed in the financial statements :

(i) gross and net book values of fixed assets at the beginning and end of an accounting period showing additions, disposals, acquisitions and other movements ;

(ii) expenditure incurred on account of fixed assets in the course of construction or acquisition ; and

(iii) revalued amount substituted for historical costs of fixed assets, the method adopted to compute the revalued amounts, the nature of indices used, the year of any appraisal made, and whether an external valuer was involved, in case where fixed assets are stated at revalued amounts.

Question 24

(a) Explain the treatment of Refund of Government Grants as per AS-12.

(b) The Company X Ltd., has to pay for delay in cotton clearing charges. The company up to 31.3.2006 has included such charges in the valuation of closing stock. This being in the nature of interest, X Ltd. decided to exclude such charges from closing stock for the year 2006-07. This would result in decrease in profit by Rs.5 lakhs. Comment.

(c) The Board of Directors of X Ltd. decided on 31.3.2007 to increase sale price of certain items of goods sold retrospectively from 1st January, 2007. As a result of this decision the company has to receive Rs.5 lakhs from its customers in respect of sales made from 1.1.2007 to 31.3.2007. But the Company’s Accountant was reluctant to make-up his mind. You are asked to offer your suggestion.

(d) Briefly explain disclosure requirements for Investments as per AS-13.

(4x4 = 16 Marks)(PE II-Nov. 2007)

Answer

(a) As per para 11 of AS 12 ‘Accounting for Government Grants’, government grant that becomes refundable is treated as an extraordinary item.

The amount refundable in respect of a government grant related to revenue is first applied against any unamortised deferred credit remaining in respect of the grant.

The amount refundable in respect of a government grant related to a specific fixed asset is recorded by increasing the book value of the asset or by reducing the capital reserve or the deferred income balance, as appropriate, by the amount refundable.

Where a grant which is in the nature of promoters’ contribution becomes refundable, in part or in full, to the government on non-fulfillment of some specified conditions, the relevant amount recoverable by the government is reduced from the capital reserve.

8.70

(b) As per para 12 of AS 2 (revised), interest and other borrowing costs are usually considered as not relating to bringing the inventories to their present location and condition and are therefore, usually not included in the cost of inventories. However, X Ltd. was in practice to charge the cost for delay in cotton clearing in the closing stock. As X Ltd. decided to change this valuation procedure of closing stock, this treatment will be considered as a change in accounting policy and such fact to be disclosed as per AS 1. Therefore, any change in amount mentioned in financial statement, which will affect the financial position of the company should be disclosed properly as per AS 1, AS 2 and AS 5.

Also a note should be given in the annual accounts that, had the company followed earlier system of valuation of closing stock, the profit before tax would have been higher by Rs. 5 lakhs.

(c) As per para 10 of AS 9 ‘Revenue Recognition’, the additional revenue on account of increase in sales price with retrospective effect, as decided by Board of Directors of X Ltd., of Rs.5 lakhs to be recognised as income for financial year 2006-07, only if the company is able to assess the ultimate collection with reasonable certainty. If at the time of raising of any claim it is unreasonable to expect ultimate collection, revenue recognition should be postponed.

(d) The disclosure requirements as per para 35 of AS 13 are as follows:

(i) Accounting policies followed for valuation of investments.

(ii) Classification of investment into current and long term in addition to classification as per Schedule VI of Companies Act in case of company.

(iii) The amount included in profit and loss statements for

(a) Interest, dividends and rentals for long term and current investments, disclosing therein gross income and tax deducted at source thereon;

(b) Profits and losses on disposal of current investment and changes in carrying amount of such investments;

(c) Profits and losses and disposal of long term investments and changes in carrying amount of investments.

(iv) Aggregate amount of quoted and unquoted investments, giving the aggregate market value of quoted investments;

(v) Any significant restrictions on investments like minimum holding period for sale/disposal, utilisation of sale proceeds or non-remittance of sale proceeds of investment held outside India.

(vi) Other disclosures required by the relevant statute governing the enterprises.

Question 25

Answer any four of the following:

(i) (a) X Ltd. purchased debentures of Rs.10 lacs of Y Ltd., which are traded in stock exchange. How will you show this item as per AS 3 while preparing cash flow statement for the year ended on 31st March, 2008?

(b) Mr. Raj a relative of key management personnel received remuneration of Rs.2,50,000 for his services in the company for the period from 1.4.2007 to 30.6.2007. On 1.7.2007, he left the service.

Should the relative be identified as a related party at the closing date i.e., on 31.3.2008 for the purpose of AS 18?

(ii) A manufacturing company purchased shares of another company from stock exchange on 1st May, 2007 at a cost of Rs.5,00,000. It also purchased Gold of Rs.2,00,000 and Silver of Rs.1,50,000 on 1st

8.71

April, 2005. How will you treat these investments as per the applicable AS in the books of the company for the year ended on 31st March, 2008, if the values of these investments are as follows:

Rs.

Shares 2,00,000

Gold 4,00,000

Silver 2,50,000

(iii) (a) Wye Ltd. received Rs.50 lacs from the Central Government as subsidy for setting up an industry in backward area. How will you treat it in accounts?

(b) How Government grant relating to Specific Fixed Assets is treated in the books as per AS 12?

(iv) A Ltd. had 6,00,000 equity shares on April 1, 2007. The company earned a profit of Rs.15,00,000 during the year 2007-08. The average fair value per share during 2007-08 was Rs.25. The company has given share option to its employees of 1,00,000 equity shares at option price of Rs.15. Calculate basic EPS and diluted EPS.

(v) In a production process, normal waste is 5% of input. 5,000 MT of input were put in process resulting in wastage of 300 MT. Cost per MT of input is Rs.1,000. The entire quantity of waste is on stock at the year end. State with reference to Accounting Standard, how will you value the inventories in this case?

(4 x 4= 16 Marks)(PEII-May, 2008)

8.72

Answer

(i) (a) As per AS 3 on ‘Cash flow Statement’, cash and cash equivalents consists of cash in hand, balance with banks and short-term, highly liquid investments1. If investment, of Rs.10 lacs, made in debentures is for short-term period then it is an item of ‘cash equivalents’.

However, if investment of Rs.10 lacs made in debentures is for long-term period then as per AS 3, it should be shown as cash flow from investing activities.

(b) According to para 10 of AS 18 on ‘Related Party Disclosures’, parties are considered to be related if at any time during the reporting period one party has the ability to control the other party or exercise significant influence over the other party in making financial and/or operating decisions.

Here, Mr. Raj, who received remuneration of Rs.2,50,000 from the company, is the relative of the key management personnel of that company. And as per para 3 clause (d) of the Standard, ‘key management personnel and relatives of such personnel’ are said to be in related party relationships. Hence, Mr. Raj, a relative of key management personnel ,should be identified as related party at the closing date i.e. on 31.3.2008.

(ii) As per para 32 of AS 13 on ‘Accounting for Investments’, any investment of long term period is shown at cost. Hence, the investment in Gold and Silver (purchased on 1st April 2005) shall continue to be shown at cost i.e., Rs.2,00,000 and Rs.1,50,000 respectively as their value have increased.

Also as per AS 13, for investment in shares - if the investment is for short-term period then the loss of Rs.3,00,000 is to be charged to profit & loss account for the year ended 31st March, 2008. If investment is of long term period then it will continue to be shown at cost in the Balance Sheet of the company. However, provision for diminution shall be made to recognize a decline, other than temporary, in the value of the investments, such reduction being determined and made for each investment individually.

(iii) (a) As per para 10 of AS 12 on ‘Accounting for Government Grants’, subsidy of Rs.50 lacs from the Central government, for setting up an industry in backward area is a government grant in the nature of promoter’s contribution. Such grants are treated as capital reserve which can be neither distributed as dividend nor considered as deferred income.

1 As per para 6 of AS 3, an investment normally qualifies as a cash equivalent only when it has a short maturity of, say three months or less from the date of acquisition.

8.73

(b) According to para 8 of AS 12 on ‘Accounting for Government Grants’, two methods of presentation, in financial statements, of grants related to specific fixed assets are regarded as acceptable alternatives.

Under one method, the grant is shown as a deduction from the gross value of the asset concerned in arriving at its book value.

Under the other method, grant related to depreciable asset is treated as deferred income which is recognized in the profit and loss statement on a systematic and rational basis over the useful life of the assets. Grants related to non-depreciable assets are credited to capital reserve under this method, as there is usually no charge to income in respect of such assets. However, if a grant related to a non-depreciable asset requires the fulfillment of certain obligations, the grant is credited to income over the same period over which the cost of meeting such obligations is charged to income. The deferred income is suitably disclosed in the balance sheet pending its apportionment to profit and loss account.

(iv) Computation of earnings per share Earnings Shares Earnings per

share Net profit for the year 2007-08 Rs.15,00,000 Weighted average number of shares outstanding during year 2007-08

6,00,000

Basic earnings per share Rs. 2.50 Number of shares under option 1,00,000 Number of shares that would have been issued at fair value: (100,000 x 15.00)/25.00

*

(60,000)

Diluted earnings per share Rs. 15,00,000 6,40,000 Rs. 2.34 (approx.)

*The earnings have not been increased as the total number of shares has been increased only by the number of shares (40,000) deemed for the purpose of the computation to have been issued for no consideration.

(v) As per para 13 of AS 2 (Revised), abnormal amounts of wasted materials, labour and other production costs are excluded from cost of inventories and such costs are recognized as expenses in the period in which they are incurred.

In this case, normal waste is 250 MT and abnormal waste is 50 MT.

The cost of 250 MT will be included in determining the cost of inventories (finished goods) at the year end. The cost of abnormal waste amounting to Rs.50,000 (50 MT × Rs.1,000) will be charged to the profit and loss statement.

Question 26

Following is the cash flow abstract of Alpha Ltd. for the year ended 31st March, 2008:

Cash Flow Abstract

Inflows Rs. Outflows Rs.

Opening balance: Payment to creditors 90,000

Cash 10,000 Salaries and wages 25,000

Bank 70,000 Payment of overheads 15,000

Share capital – shares issued 5,00,000 Fixed assets acquired 4,00,000

Collection from Debtors 3,50,000 Debentures redeemed 50,000

8.74

Sale of fixed assets 70,000 Bank loan repaid 2,50,000

Taxation 55,000

Dividends 1,00,000

Closing balance:

Cash 5,000

bank 10,000

10,00,000 10,00,000 Prepare Cash Flow Statement for the year ended 31st March, 2008 in accordance with Accounting

standard – 3.

(8 Marks) (PE II- Nov. 2008)

Answer

Cash Flow Statement

for the year ended 31.3.2008

Rs. Rs.

Cash flow from operating activities

Cash received from customers 3,50,000

Cash paid to suppliers (90,000)

Cash paid to employees (salaries and wages) (25,000)

Other cash payments (overheads) (15,000)

Cash generated from operations 2,20,000

Income tax paid (55,000)

Net cash from operating activities 1,65,000

Cash flow from investing activities

Payment for purchase of fixed assets (4,00,000)

Proceeds from sale of fixed assets 70,000

Net cash used in investment activities (3,30,000)

Cash flow from financing activities

Proceeds from issue of share capital 5,00,000

Bank loan repaid (2,50,000)

Debentures redeemed (50,000)

Dividends paid (1,00,000)

Net cash from financing activities 1,00,000

Net decrease in cash and cash equivalents (65,000)

Cash and cash equivalents at the beginning of the year 80,000

Cash and cash equivalents at the end of the year 15,000

Question 27

8.75

(a) B Ltd. undertook a construction contract for Rs. 50 crores in April, 2007. the cost of construction was initially estimated at Rs. 35 crores. The contract is to be completed in 3 years. While executing the contract, the company estimated the cost of completion of the contract at Rs. 53 crores.

Can the company provide for the expected loss in the book of account for the year ended 31st March, 2008?

(b) List any five related party transactions, which require disclosure as per AS 18.

(c) A Government grant of Rs. 25 lakhs received 3 years ago in respect of a machinery which costs Rs. 200 lakhs, became refundable in March, 2008.

(i) How the receipt of grant would have been recorded in the books of the recipient?

(ii) How the refund of grant would be reflected in the books, at the time of its refund?

(d) List the conditions to be fulfilled as per Accounting Standard 14 (AS 14) for an amalgamation to be in the nature of merger, in the case of companies.

(e) Discuss the treatment of exchange loss relating to fixed assets as per AS 11 vis – a – vis the Schedule VI disclosure under the Companies Act, 1956.

(4 x 5 = 20 Marks) (PE II- Nov. 2008)

8.76

Answer

(a) As per para 35 of AS 7 “Construction Contracts”, when it is probable that total contract costs will exceed total contract revenue, the expected loss should be recognised as an expense immediately. Therefore, The foreseeable loss of Rs.3 crores (Rs. 53 crores less Rs. 50 crores) should be recognised as an expense immediately in the year ended 31st march, 2008. The amount of loss is determined irrespective of

(i) Whether or not work has commenced on the contract;

(ii) Stage of completion of contract activity; or

(iii) The amount of profits expected to arise on other contracts which are not treated as a single construction contract in accordance with para 8 of AS 7.

(b) Five examples of related party transactions for which disclosure is required according to AS 18 are:

(i) Purchase and/or sales of goods (finished or unfinished)

(ii) Purchase or sale of fixed assets.

(iii) Rendering or receiving of services.

(iv) Agency arrangements.

(v) Leasing or hire purchase arrangements.

(c) The grant is shown as a deduction from the gross value of the asset. Depreciation on machinery would be charged on the reduced value of Rs.175 lakhs. Alternatively, the grant may be treated as deferred income which should be credited to profit and loss statement on a systematic and rational basis over the useful life of the asset.

As per para 21 of AS 12, the amount refundable in respect of a grant related to a specific fixed asset should be recorded by increasing the book value of the asset or by reducing the capital reserve or the deferred income balance, as appropriate, by the amount refundable. In the first alternative, i.e., where the book value of the asset is increased, depreciation on the revised book value should be provided prospectively over the residual useful life of the asset.

(d) An amalgamation should be considered to be an amalgamation in the nature of merger if the following conditions are satisfied:

(i) All the assets and liabilities of the transferor company become, after amalgamation, the assets and liabilities of the transferee company.

8.77

(ii) Shareholders holding not less than 90% of the face value of the equity shares of the transferor company (other than the equity shares already held therein, immediately before the amal-gamation, by the transferee company or its subsidiaries or their nominees) become equity shareholders of the transferee company by virtue of the amalgamation.

(iii) The consideration for the amalgamation receivable by those equity shareholders of the transferor company who agree to become equity shareholders of the transferee company is discharged by the transferee company wholly by the issue of equity shares in the transferee company, except that cash may be paid in respect of any fractional shares.

(iv) The business of the transferor company is intended to be carried on, after the amalgamation, by the transferee company.

(v) No adjustment is intended to be made to the book values of the assets and liabilities of the transferor company when they are incorporated in the financial statements of the transferee company except to ensure uniformity of accounting policies.

(e) Schedule VI to The Companies Act, 1956 provides that any increase or decrease in liability due to change in the rate of exchange relating to any fixed asset should be added to or deducted from the cost of the asset. The amount arrived at should be taken to be the cost of the fixed asset.

AS 11 (revised), however, does not require adjustment of exchange difference in the carrying amount of fixed assets. The exchange difference is required to be recognised in the statement of profit or loss since it is felt that this treatment is conceptually preferable to that required in Schedule VI and is in consonance with the international position in this regard.

The provisions of AS 11 will prevail over Schedule VI of the Companies Act. National Advisory Committee on Accounting Standards (NACAS) has notified AS 11 for preparation of financial statements of companies. ICAI has come up with the announcement in this regard, stating that after the notification of AS 11 by NACAS, AS 11 will overrule Schedule VI of the Companies Act.

13 FINANCIAL ANALYSIS

UNITS 1 & 2 : FUND FLOW STATEMENT AND CASH FLOW STATEMENT

(A) Write short notes on:

Question 1

Cash Flow Statement. (5 marks) (Intermediate–Nov. 1997)

Answer

Cash flow statement is a statement of inflows and outflows of cash and cash equivalents. It starts with the opening balance of cash and cash equivalents at the start of the accounting period. It then gives in a summary form, the inflows and outflows relating to the following three classifications of activities :

(i) Operating activities : They are the principal revenue producing activities of the enterprise.

(ii) Investing activities : They deal with the acquisition and disposal of long-term assets and long term investments.

(iii) Financing activities : They reflect changes in the size and composition of capital in the case of a company this would preference capital and borrowings of the enterprise.

The cash flows arising from extraordinary items are disclosed separately under each of the above three classifications.

Likewise where the amount of significant cash and cash equivalent balances held by an enterprise are not available for use by the enterprise, the same should be disclosed separately together with a commentary by the management.

Question 2

In the case of manufacturing company :

(i) List the items of ‘inflows’ of cash receipts from operating activities;

(ii) List the items of “outlflows” of investing activities. (4 marks) [Intermediate May 1998]

13.2

Answer

(i) Inflows of cash receipts from operating activities :

(a) Cash receipts from the sales of goods;

(b) Royalties, fees, commission and other revenue;

(c) Refunds of income-tax.

(ii) Outflows of investing activities :

(a) Cash payments for acquisition of fixed assets;

(b) Cash payments for acquisition of shares, warrants or debts instruments of other enterprises and interests in joint ventures (other than payments for instruments considered to cash equivalents and those for dealing or trading purposes);

(c) Cash advances and loans to third parties.

Question 3

Classification of activities (with two examples) as suggested in AS 3, to be used for preparing a cash flow statements. (5 marks) (Intermediate–May 2001)

Answer

AS 3 (Revised) on Cash Flow Statements requires that the cash flow statement should report cash flows by operating, investing and financing activities.

(i) Operating activities are the principal revenue-producing activities of the enterprise and other activities that are not investing or financing activities. Cash receipts from sale of goods and cash payments to suppliers of goods are two examples of operating activities.

(ii) Investing activities are acquisition and disposal of long-term assets and other investments not included in cash equivalents. Payment made to acquire machinery and cash received for sale of furniture are examples of investing activities.

(iii) Financial activities are those activities that result in changes in the size and composition of the owner’s capital (including preference share capital in the case of a company) and borrowings of the enterprise. Cash proceeds from issue of shares and cash paid to redeem debentures are two examples of financing activities.

Question 4

Explain the difference between direct and indirect methods of reporting cash flows from operating activities with reference to Accounting Standard 3, (AS 3) revised. (8 marks) (Final Nov. 2001)

Answer

As per para 18 of AS 3 (Revised) on Cash Flow Statements, an enterprise should report cash flows from operating activities using either :

13.3

(a) the direct method, whereby major classes of gross cash receipts and gross cash payments are disclosed; or

(b) the indirect method, whereby net profit or loss in adjusted for the effects of transactions of a non-cash nature, any deferrals or accruals of past or future operating cash receipts or payments, and items of income or expense associated with investing or financing cash flows.

The direct method provides information which may be useful in estimating future cash flows and which is not available under the indirect method and is, therefore, considered more appropriate than the indirect method. Under the direct method, information about major classes of gross cash receipts and gross cash payments may be obtained either :

(a) from the accounting records of the enterprise; or

(b) by adjusting sales, cost of sales (interest and similar income and interest expense and similar charges for a financial enterprise) and other items in the statment of profit and loss for :

(i) changes during the period in inventories and operating receivables and payables;

(ii) other non-cash items; and

(iii) other items for which the cash effects are investing or financing cash flows.

Under the indirect method, the net cash flow from operating activies is determined by adjusting net profit or loss for the effects of :

(a) changes during the period in inventories and operating receivables and payables;

(b non-cash items such as depreciation, provisions, deferred taxes and unrealised foreign exchange gains and losses; and

(c) all other items for which the cash effects are investing or financing cash flows.

Alternatively, the net cash flow from operating activities may be presented under the indirect method by showing the operating revenues and expenses, excluding non-cash items disclosed in the statement of profit and loss and the changes during the period in inventories and operating receivables and payables.

Question 5

What all are the differences between Cash Flow statement and Fund Flow statement?

(4 Marks) (PE-II – May 2006)

Answer

Differences between cash flow statement and fund flow statement (i) Cash flow statement deals with the change in cash position between two points of time. Fund flow

statement deals with the changes in working capital position.

(ii) Cash flow statement contains opening as well as closing balances of cash and cash equivalents. The fund flow statement does not contain any such opening and closing balance.

(iii) Cash flow statement records only inflow and outflow of cash. Fund flow statement records sources and application of funds.

(iv) Fund flow statement can be prepared from the cash flow statement under indirect method. However, a cash flow statement cannot be prepared from fund flow statement.

(v) A statement of changes in working capital is usually prepared alongwith fund flow statement. No such statement is prepared along with the cash flow statement.

(B) Practical Questions:

Question 1

Given below are the condensed Balance Sheets of Lambakadi Ltd. for two years and the statement of Profit and Loss for one year : (Figures Rs. in lakhs)

13.4

As at 31st March 1998 1997 Share Capital In equity shares of Rs. 100 each 150 110 10% redeemable preference shares of Rs. 100 each 10 40 Capital redemption reserve 10 — General reserve 15 10 Profit and loss account balance 30 20 8% debentures with convertible option 20 40 Other term loans 15 30

250 250

Fixed assets less depreciation 130 100 Long term investments 40 50 Working capital 80 100

250 250

Statement of Profit and Loss for the year ended 31st March, 1998 (Figures Rs. in lakhs) Sales 600 Less : Cost of sales 400 200 Establishment charges 30 Selling and distribution expenses 60 Interest expenses 5

Loss on sale of equipment (Book value Rs. 40 lakhs) 15 110 90 Interest income 4 Dividend income 2 Foreign exchange gain 10 Damages received for loss of reputation 14 30

120 Depreciation 50 70 Taxes 30 40 Dividends 15

Net profit carried to Balance Sheet 25

Your are informed by the accountant that ledgers relating to debtors, creditors and stock for both the years were seized by the income-tax authorities and it would take atleast two months to obtain copies of the same. However, he is able to furnish the following data : (Figures Rs. in lakhs) 1998 1997 Dividend receivable 2 4 Interest receivable 3 2 Cash on hand and with bank 7 10 Investments maturing within two months 3 2

15 18

Interest payable 4 5 Taxes payable 6 3

10 8

Current ratio 1.5 1.4 Acid test ratio 1.1 0.8

13.5

It is also gathered that debentureholders owning 50% of the debentures outstanding as on 31.3.97 exercised the option for conversion into equity shares during the financial year and the same was put through. You are required to prepare a direct method cash flow statement for the financial year, 1998 in accordance with para 18(a) of Accounting Standard (AS) 3 revised. (20 marks) (Final May 1998) Answer

Lambakadi Ltd.

Direct Method Cash Flow Statement

for the year ended 31st March, 1998

(Rs. in lakhs)

Cash flows from operating activities

Cash receipts from customers 621

Cash paid to suppliers and employees (496)

Cash generated from operations 125

Taxes paid (27)

Cash flows before extraordinary item 98

Damages received for loss of reputation 14

Net cash from operating activities 112

Cash flows from investing activities

Purchase of fixed assets (120)

Proceeds from sale of equipment 25

Proceeds from sale of long term investments 10

Interest received 3

Dividend received 4

Net cash used in investing activities (78)

Cash flows from financing activities

Proceeds from issuance of share capital 20

Redemption of preference share capital (30)

Repayments of term loans (15)

Interest paid (6)

Dividend paid (15)

Net cash used in financing activities (46)

Net increase in cash and cash equivalents (12)

Cash and cash equivalents at beginning of period 12

(See Note 1 to the Cash Flow Statement)

Cash and cash equivalents at end of the period

(See Note 1 to the Cash Flow Statement) NIL

Notes to the Cash Flow Statement

(Rs. in lakhs)

1. Cash and Cash Equivalents 31.3.1998 31.3.1997

Cash on hand and with bank 7 10

Short-term investments 3 2

13.6

10 12

Effect of exchange rate changes (10) –

Cash and cash equivalents Nil 12

2. Conversion of debentures into equity shares, a non-cash transaction, amounted to

Rs.20 lakhs.

Working Notes :

(Rs. in lakhs) 1. Calculation of debtors, creditors and stock 31.3.98 31.3.97 (a) Current Ratio 1.5:1 1.4:1 Working Capital to Current Liabilities Ratio 0.5:1 0.4:1 Working Capital (Rs.in lakhs) 80 100

Current Assets (Rs.in lakhs) 2405.0

5.180

3504.0

4.110

Current Liabilities (Rs.in lakhs) 240 – 80 = 160 350 – 100 = 250 (b) Current Ratio 1.5 1.4 Less : Acid Test Ratio 1.1 0.8 0.4 0.6 Stock : Current Liabilities 0.4:1 0.6:1 Stock (Rs.in lakhs) 160 × 0.4 = 64 250 × 0.6 = 150 (Rs. in lakhs) (c) Break-up of Current Assets Stock 64 150 Debtors (Balancing figures) 161 182 Other Current Assets 15 18 240 350 (d) Break-up of Current Liabilities Creditors (Balancing figures) 150 242 Others 10 8 160 250 2. Cash receipts from customers Sales 600 Add: Debtors at the beginning of the year 182 782 Less : Debtors at the end of the year 161 621 3. Cash paid to suppliers and employees Cost of sales 400 Establishment charges 30 Selling and distribution expenses 60 490 Add: Creditors at the beginning of the year 242 Stock at the end of the year 64 306 796 Less : Creditors at the end of year 150 Stock at the beginning of the year 150 300

496

13.7

4. Taxes paid Tax expense for the year 30 Add : Tax liability at the beginning of the year 3 33 Less : Tax liability at the end of year 6 27 5. Fixed assets acquisitions W.D.V. at 31.3.1998 130 Add back : Depreciation for the year 50 Disposals 40 220 Less : W.D.V. at 31.3.1997 100 Purchase of fixed assets 120 6. Interest received Interest income for the year 4 Add : Amount receivable at the beginning of the year 2 6 Less : Amount receivable at the end of the year 3 3 7. Dividend received Dividend income for the year 2 Add : Amount receivable at the beginning of the year 4 6 Less : Amount receivable at the end of the year 2 4 8. Issue of shares Equity share capital at the end of the year 150 Less : Equity share capital at the beginning of the year 110 40 Less : Conversion of debentures into equity shares during the year (non-cash transaction) 20 Cash flow from issue of equity shares 20 9. Interest paid Interest expense for the year 5 Add : Interest payable at the beginning of the year 5 10 Less : Interest payable at the end of the year 4 6 Notes :

1. It has been assumed that dividends for the year, Rs. 15 lakhs have been paid off.

13.8

2. It has been assumed that foreign exchange gain represents the effect of changes in exchange rates on cash and cash equivalents held in a foreign currency.

Question 2

The following are the changes in the account balances taken from the Balance Sheets of PQ Ltd. as at the beginning and end of the year. :

Changes in Rupees in

debt or [credit]

Equity share capital 30,000 shares of Rs. 10 each issued and fully paid 0

Capital reserve ]49,200]

8% debentures [50,000]

Debenture discount 1,000

Freehold property at cost/revaluation 43,000

Plant and machinery at cost 60,000

Depreciation on plant and machinery [14,400]

Debtors 50,000

Stock and work-in-progress 38,500

Creditors [11,800]

Net profit for the year [76,500]

Dividend paid in respect of earlier year 30,000

Provision for doubtful debts [3,300]

Trade investments at cost 47,000

Bank [64,300]

0

You are informed that.

(a) Capital reserve as at the end of the year represented realised profits on sale of one freehold property together with surplus arising on the revaluation of balance of freehold properties.

(b) During the year plant costing Rs. 18,000 against which depreciation provision of Rs. 13,500 was lying, was sold for Rs. 7,000.

(c) During the middle of the year Rs. 50,000 debentures were issued for cash at a discount of Rs. 1,000.

(d) The net profit for the year was after crediting the profit on sale of plant and charging debenture interest.

You are required to prepare a statement which will explain, why bank borrowing has increased by Rs. 64,300 during the year end. Ignore taxation. (15 marks)(Final Nov. 1998)

Answer PQ Ltd.

Cash Flow Statement for the year ended... Rs. Cash flows from operating activities Net profit 76,500 Adjustments for : Depreciation 27,900 Profit on sale of plant (2,500) Interest expense 2,000

13.9

Operating profit before working capital changes 1,03,900 Increase in debtors (less provision) (46,700) Increase in stock and work-in-progress (38,500) Increase in creditors 11,800 Net cash operating activities 30,500 Cash flows from investing activities Purchase of plant and machinery (78,000) Proceeds from sale of plant 7,000 Proceeds from sale of freehold property 6,200 Increase in trade investments (47,000) Net cash used in investing activities (1,11,800) Cash flows from financing activities Proceeds from issuance od debentures at discount 49,000 Debenture interest paid (2,000) Dividend paid in financing activities (30,000) Net cash from financing activities 17,000 Excess of outflows over inflows 64,300 Thus the shortfall of Rs. 64,300 was made up through borrowing from bank.

Working Notes :

(1) Plant and Machinery Rs Amount of increase (at cost) 60,000 Add : Disposal (at cost) 18,000

Acquisition during the year 78,000

Disposal of plant : proceeds from sale 7,000 Net book value (18,000 – 13,500) 4,500 Profit on sale 2,500

(2) Freehold property Capital Reserve 49,200 Less : Increase in freehold property (closing balance minus opening balance) 43,000 Proceeds from sale of freehold property 6,200

Memorandum Accounts

(a) Plant and Machinery Account

Rs. Rs.

To Balance b/d By Bank (Sale proceeds) 7,000

To Profit and Loss A/c 2,500 By Provision for Depreciation 13,500

(Profit on sale) By Balance c/d 60,000

To Bank (Balancing figure) 78,000

80,500 80,500

(b) Provision for Depreciation (Plant and Machinery) Account

To Plant and Machinery A/c 13,500 By Balance b/d

To Balance c/d 14,400 By Profit and Loss A/c 27,900

(Balancing figure)

13.10

27,900 27,900

(c) Freehold Property Account

To Balance b/d — By Bank A/c 6,200

To Capital reserve 49,200 (Balancing figure)

By Balance c/d 43,000

49,200 49,200

In the absence of information about the opening balances, the entire amount of change has been considered under the closing balances for the purpose of calculation of missing figures.

Notes :

(1) Investment income and dividend pertaining to the current year have not been considered in the absence of any related information.

(2) Debenture interest has been calculated for 6 months @ 8% on Rs. 50,000.

Question 3

Examine the following schedule prepared by K Ltd.

K Ltd.

Schedule of funds provided by operations for the year ended 31st July, 1999

(Rs.’000) (Rs.’000)

Sales 32,760

Add : Decrease in bills receivable. 1,000

Less : Increase in accounts receivable (626)

Inflow from operating revenues 33,134

Cost of goods sold 18,588

Less : Decrease in inventories (212)

Add : Decrease in trades payable 81 18,457

Wages and Salaries 5,284

Less : Increase in wages payable (12) 5,272

Administrative Expenses 3,066

Add : Increase in prepaid expenses 11 3,077

Property taxes 428

Interest expenses 532

Add : Amortisation of premium on bonds payable 20 552

Outflow from operating expenses 27,786

From operations 5,348

Rent Income 207

Add : Increase in unearned rent 3 210

5,558

Income tax 1,330

Less : Increase in deferred tax 50 1,280

13.11

Funds from operations 4,278

Required :

(i) What is the definition of funds shown in the schedule?

(ii) What amount was reported as gross margin in the income statement?

(iii) How much cash was collected from the customers?

(iv) How much cash was paid for the purchases made?

(v) As a result of change in inventories, did the working capital increase or decrease and by what amount?

(vi) How much rent was actually earned during the year?

(vii) What was the amount of tax expenses reported on the income statement?

Can you reconcile the profit after tax-with the funds provided by the operations?

(16 marks)(Final May 2000)

Answer

(i) ‘Funds’ shown in the schedule refer to the cash and cash equivalents [as defined in AS 3 (Revised) on Cash Flow Statements].

(ii) Gross margin in the income statement :

Rs. (’000)

Sales 32,760

Cost of goods sold 18,588

14,172

(iii) Cash collected from the customers 33,134

(iv) Cash paid for purchases made 18,457

(v) Change in inventories would reduce the working capital by 212

(vi) Rental income earned during the year 207

(vii) Tax expenses reported in the income statement 1330

(Viii) Reconciliation Statement Rs.(’000)

Profit after tax (See W.N.) 3,719

Decrease in bills receivable 1,000

Increase in accounts receivable (626)

Decrease in inventories 212

Decrease in trades payable (81)

Increase in wages payable 12

Increase in prepaid expenses (11)

Increase in unearned rent 3

Increase in deferred tax 50

Funds from operations as shown in the schedule 4,278

(i.e. cash and cash equivalents)

Working Note :

13.12

Calculation of Profit after Tax Rs. (’000)

Sales 32,760

Less : Cost of goods sold 18,588

Gross margin 14,172

Add : Rental income 207

14,379

Less : Wages and salaries 5,284

Administrative expenses 3,066

Property taxes 428

Interest expenses 532

Amortisation of premium on bonds payable 20

9,330

Profit before tax 5,049

Less : Income tax 1,330

Profit after tax 3,719

Question 4

Ms. Joyti of Star Oils Limited has collected the following information for the preparation of cash flow statement for the year 2000 :

(Rs. in Lakhs)

Net Profit 25,000

Dividend (including dividend tax) paid 8,535

Provision for Income tax 5,000

Income tax paid during the year 4,248

Loss on sale of assets (net) 40

Book value of the assets sold 185

Depreciation charged to Profit & Loss Account 20,000

Amortisation of Capital grant 6

Profit on sale of Investments 100

Carrying amount of Investment sold 27,765

Interest income on investments 2,506

Increase expenses 10,000

Interest paid during the year 10,520

Increase in Working Capital (excluding Cash & Bank Balance) 56,075

Purchase of fixed assets 14,560

Investment in joint venture 3,850

Expenditure on construction work in progress 34,740

Proceeds from calls in arrear 2

Receipt of grant for capital projects 12

Proceeds from long-term borrowings 25,980

Proceeds from short-term borrowings 20,575

Opening cash and Bank balance 5,003

13.13

Closing cash and Bank balance 6,988

Required :

Prepare the Cash Flow Statement for the year 2000 in accordance with AS 3, Cash Flow Statements issued by the Institute of Chartered Accounants of India. (make necessary assumptions).(16 marks)(Final May 2001)

Answer

Star Oils Limited

Cash Flow Statement

for the year ended 31st December, 2000

(Rs. in lakhs)

Cash flows from operating activities

Net profit before taxation (25,000 + 5,000) 30,000

Adjustments for :

Depreciation 20,000

Loss on sale of assets (Net) 40

Amortisation of capital grant (6)

Profit on sale of investments (100)

Interest income on investments (2,506)

Interest expenses 10,000

Operating profit before working capital changes 57,428

Changes in working capital (Excluding cash and bank balance) (56,075)

Cash generated from operations 1,353

Income taxes paid (4,248)

Net cash used in operating activities (2,895)

Cash flows from investing activities

Sale of assets 145

Sale of investments (27,765 + 100) (27,865)

Interest income on investments 2,506

Purchase of fixed assets (14,560)

Investment in joint venture (3,850)

Expenditure on construction work-in progress (34,740)

Net cash used in investing activities (22,634)

Cash flows from financing activities

Proceeds from calls in arrear 2

Receipts of grant for capital projects 12

Proceeds from long-term borrowings 25,980

Proceed from short-term borrowings 20,575

Interest paid (10,520)

Dividend (including dividend tax) paid (8,535)

27,514

Net increase in cash and cash equivalents 1,985

Cash and cash equivalents at the beginning of the period 5,003

Cash and cash equivalents at the end of the period 6,988

13.14

Working note :

Book value of the assets sold 185

Less : Loss on sale of assets 40

Proceeds on sale 145

Assumption :

Interest income on investments Rs. 2,506 has been received during the year.

Question 5

From the following Summary Cash Account of X Ltd. prepare Cash Flow Statement for the year ended 31st March, 2001 in accordance with AS 3 (Revised) using the direct method. The company does not have any cash equivalents.

Summary Cash Account for the year ended 31.3.2001

Rs. ’000 Rs. ’000

Balance on 1.4.2000 50 Payment to Suppliers 2,000

Issue of Equity Shares 300 Purchase of Fixed Assets 200

Receipts from Customers 2,800 Overhead expense 200

Sale of Fixed Assets 100 Wages and Salaries 100

Taxation 250

Dividend 50

Repayment of Bank Loan 300

Balance on 31.3.2001 150

3,250 3,250

(8 marks)(Final Nov. 2001)

Answer

X Ltd.

Cash Flow Statement for the year ended 31st March, 2001

(Using the direct method)

Rs. ’000 Rs.’000

Cash flows from operating activities

Cash receipts from customers 2,800

Cash payments to suppliers (2,000)

Cash paid to employees (100)

Cash payments for overheads (200)

Cash generated from operations 500

Income tax paid (250)

Net cash from operating activities 250

Cash flows from investing activities

Payments for purchase of fixed assets (200)

Proceeds from sale of fixed assets 100

Net cash used in investing activities (100)

Cash flows from financing activities

13.15

Proceeds from issuance of equity shares 300

Bank loan repaid (300)

Dividend paid (50)

Net cash used in financing activities (50)

Net increase in cash 100

Cash at beginning of the period 50

Cash at end of the period 150

13.16

Question 6

From the following details relating to the Accounts of Grow More Ltd. prepare Cash Flow Statement:

Liabilities 31.03.2002 (Rs.) 31.03.2001 (Rs.)

Share Capital 10,00,000 8,00,000

Reserve 2,00,000 1,50,000

Profit and Loss Account 1,00,000 60,000

Debentures 2,00,000 –

Provision for taxation 1,00,000 70,000

Proposed dividend 2,00,000 1,00,000

Sundry Creditors 7,00,000 8,20,000

25,00,000 20,00,000

Assets

Plant and Machinery 7,00,000 5,00,000

Land and Building 6,00,000 4,00,000

Investments 1,00,000 –

Sundry Debtors 5,00,000 7,00,000

Stock 4,00,000 2,00,000

Cash on hand/Bank 2,00,000 2,00,000

25,00,000 20,00,000

(i) Depreciation @ 25% was charged on the opening value of Plant and Machinery.

(ii) During the year one old machine costing 50,000 (WDV 20,000) was sold for Rs. 35,000.

(iii) Rs. 50,000 was paid towards Income tax during the year.

(iv) Building under construction was not subject to any depreciation.

Prepare Cash flow Statement. (16 marks) (PE-II–Nov. 2002)

Answer

Grow More Ltd Cash Flow Statement

for the year ended 31st March, 2002 Cash Flow from Operating Activities

Net Profit 40,000

Proposed Dividend 2,00,000

Provision for taxation 80,000

Transfer to General Reserve 50,000

Depreciation 1,25,000

Profit on sale of Plant and Machinery (15,000)

Operating Profit before Working Capital changes 4,80,000

Increase in Stock (2,00,000)

Decrease in debtors 2,00,000

Decrease in creditors (1,20,000)

13.17

Cash generated from operations 3,60,000

Income tax paid (50,000)

Net Cash from operating activities 3,10,000 Cash Flow from Inventing Activities

Purchase of fixed assets (3,45,000)

Expenses on building (2,00,000)

Increase in investments (1,00,000)

Sale of old machine 35,000

Net Cash used ininvesting activities (6,10,000) Cash Flow from financing activities:

Proceeds from issue of shares 2,00,000

Proceeds from issue of debentures 2,00,000

Dividend paid (1,00,000)

Net cash used in financing activities 3,00,000

Net increase in cash or cash equivalents NIL

Cash and Cash equivalents at the beginning of the year 2,00,000

Cash and Cash equivalents at the end of the year 2,00,000

Working Notes:

Provision for taxation account

Rs. Rs.

To Cash (Paid) 50,000 By Balance b/d 70,000

To Balance c/d 1,00,000 By Profit and Loss A/c 80,000

(Balancing figure)

1,50,000 1,50,000

13.18

Plant and Machinery account

Rs. Rs.

To Balance b/d 5,00,000 By Depreciation 1,25,000

To Cash (Balancing figure) 3,45,000 By Cash (sale of machine) 20,000

_______ By Balance c/d 7,00,000

8,45,000 8,45,000

Question 7

From the following Balance Sheet and information, prepare Cash Flow Statement of Ryan Ltd. for the year ended 31st March, 2003:

Balance Sheet

31st March, 2003

31st March, 2002

Rs. Rs. Liabilities Equity Share Capital 6,00,000 5,00,000 10% Redeemable Preference Capital

2,00,000

Capital Redemption Reserve 1,00,000 – Capital Reserve 1,00,000 – General Reserve 1,00,000 2,50,000 Profit and Loss Account 70,000 50,000 9% Debentures 2,00,000 – Sundry Creditors 95,000 80,000 Bills Payable 20,000 30,000 Liabilities for Expenses 30,000 20,000 Provision for Taxation 95,000 60,000 Proposed Dividend 90,000 60,000 15,00,000 12,50,000

31st March,

2003 31st March,

2002 Rs. Rs. Assets Land and Building 1,50,000 2,00,000 Plant and Machinery 7,65,000 5,00,000 Investments 50,000 80,000 Inventory 95,000 90,000 Bills Receivable 65,000 70,000 Sundry Debtors 1,75,000 1,30,000 Cash and Bank 65,000 90,000 Preliminary Expenses 10,000 25,000 Voluntary Separation Payments 1,25,000 65,000 15,00,000 12,50,000

13.19

Additional Information:

(i) A piece of land has been sold out for Rs. 1,50,000 (Cost – Rs. 1,20,000) and the balance land was revalued. Capital Reserve consisted of profit on sale and profit on revaluation.

(ii) On 1st April, 2002 a plant was sold for Rs. 90,000 (Original Cost – Rs. 70,000 and W.D.V. – Rs. 50,000) and Debentures worth Rs. 1 lakh was issued at par as part consideration for plant of Rs. 4.5 lakhs acquired.

(iii) Part of the investments (Cost – Rs. 50,000) was sold for Rs. 70,000.

(iv) Pre-acquisition dividend received Rs. 5,000 was adjusted against cost of investment.

(v) Directors have proposed 15% dividend for the current year.

(vi) Voluntary separation cost of Rs. 50,000 was adjusted against General Reserve.

(vii) Income-tax liability for the current year was estimated at Rs. 1,35,000.

(viii) Depreciation @ 15% has been written off from Plant account but no depreciation has been charged on Land and Building. (20 marks) (PE-II–May 2003)

Answer

Cash Flow Statement of Ryan Limited

For the year ended 31st March, 2003

Cash flow from operating activities Rs. Rs.

Net Profit before taxation 2,45,000

Adjustment for

Depreciation 1,35,000

Preliminary expenses 15,000

Profit on sale of plant (40,000)

Profit on sale of investments (20,000)

Interest on debentures 18,000

Operating profit before working capital changes 3,53,000

Increase in inventory (5,000)

Decrease in bills receivable 5,000

Increase in debtors (45,000)

Increase in creditors 15,000

Decrease in bills payable (10,000)

Increase in accrued liabilities 10,000

Cash generated from operations 3,23,000

Income taxes paid (1,00,000)

2,23,000

Voluntary separation payments (1,10,000)

Net cash from operating activities 1,13,000

Cash flow from investing activities

Proceeds from sale of land 1,50,000

Proceeds from sale of plant 90,000

Proceeds from sale of investments 70,000

13.20

Purchase of plant (3,50,000)

Purchase of investments (25,000)

Pre-acquisition dividend received 5,000

Net cash used in investing activities (60,000)

Cash flow from financing activities

Proceeds from issue of equity shares 1,00,000

Proceeds from issue of debentures 1,00,000

Redemption of preference shares (2,00,000)

Dividends paid (60,000)

Interest paid on debentures (18,000)

Net cash used in financing activities (78,000)

Net decrease in cash and cash equivalents (25,000)

Cash and cash equivalents at the beginning of the year 90,000

Cash and Cash equivalents at the end of the year 65,000

Working Notes:

1. Rs. Net profit before taxation Retained profit 70,000 Less: Balance as on 31.3.2002 (50,000) 20,000 Provision for taxation 1,35,000 Proposed dividend 90,000 2,45,000

2. Land and Building Account

Rs. Rs. To Balance b/d 2,00,000 By Cash (Sale) 1,50,000 To Capital reserve (Profit on sale) 30,000 By Balance c/d 1,50,000 To Capital reserve

(Revaluation profit)

70,000

_______ 3,00,000 3,00,000

3. Plant and Machinery Account Rs. Rs.

To Balance b/d 5,00,000 By Cash (Sale) 90,000 To Profit and loss account 40,000 By Depreciation 1,35,000 To Debentures 1,00,000 By Balance c/d 7,65,000 To Bank 3,50,000 9,90,000 9,90,000

4. Investments Account

Rs. Rs.

13.21

To Balance b/d 80,000 By Cash (Sale) 70,000 To To

Profit and loss account Bank (Balancing figure)

20,000 25,000

By Dividend (Pre-acquisition)

5,000

_______ By Balance c/d 50,000 1,25,000 1,25,000

5. Capital Reserve Account

Rs. Rs.

To Balance c/d 1,00,000 By Profit on sale of land 30,000

_______ By Profit on revaluation

of land

70,000 1,00,000 1,00,000

6. General Reserve Account

Rs. Rs. To Voluntary separation cost 50,000 By Balance b/d 2,50,000 To To

Capital redemption reserve Balance c/d

1,00,000 1,00,000

_______

2,50,000 2,50,000

7. Proposed Dividend Account

Rs. Rs. To Bank (Balancing figure) 60,000 By Balance b/d 60,000 To Balance c/d 90,000 By Profit and loss account 90,000 1,50,000 1,50,000

8. Provision for Taxation Account

Rs. Rs. To Bank (Balancing figure) 1,00,000 By Balance b/d 60,000 To Balance c/d 95,000 By Profit and loss account 1,35,000 1,95,000 1,95,000

9. Voluntary Separation Payments Account

Rs. Rs. To Balance b/d 65,000 By General reserve 50,000 To Bank (Balancing figure) 1,10,000 By Balance c/d 1,25,000 1,75,000 1,75,000

Note: Cash Flow statement has been prepared using ‘indirect method’.

Question 8

The Balance Sheet of New Light Ltd. for the years ended 31st March, 2001 and 2002 are as follows:

Liabilities 31st March 2001

31st March 2002

Assets 31st March 2001

31st March 2002

13.22

(Rs.) (Rs.) (Rs.) (Rs.)

Equity share capital 12,00,000 16,00,000 Fixed Assets 32,00,000 38,00,000 10% Preference share capital

4,00,000

2,80,000

Less: Depreciation 9,20,000 22,80,000

11,60,000 26,40,000

Capital Reserve – 40,000 Investment 4,00,000 3,20,000 General Reserve 6,80,000 8,00,000 Cash 10,000 10,000 Profit and Loss A/c 2,40,000 3,00,000 Other current assets 11,10,000 13,10,000 9% Debentures 4,00,000 2,80,000 Preliminary expenses 80,000 40,000 Current liabilities 4,80,000 5,20,000 Proposed dividend 1,20,000 1,44,000 Provision for Tax 3,60,000 3,40,000 Unpaid dividend – 16,000 ________ ________ 38,80,000 43,20,000 38,80,000 43,20,000 Additional information:

(i) The company sold one fixed asset for Rs. 1,00,000, the cost of which was Rs. 2,00,000 and the depreciation provided on it was Rs. 80,000.

(ii) The company also decided to write off another fixed asset costing Rs. 56,000 on which depreciation amounting to Rs. 40,000 has been provided.

(iii) Depreciation on fixed assets provided Rs. 3,60,000.

(iv) Company sold some investment at a profit of Rs. 40,000, which was credited to capital reserve.

(v) Debentures and preference share capital redeemed at 5% premium.

(vi) Company decided to value stock at cost, whereas previously the practice was to value stock at cost less 10%. The stock according to books on 31.3.2001 was Rs. 2,16,000. The stock on 31.3.2002 was correctly valued at Rs. 3,00,000.

Prepare Cash Flow Statement as per revised Accounting Standard 3 by indirect method.

(16 marks) (PE-II–Nov. 2003)

Answer New Light Ltd.

Cash Flow Statement for the year ended 31st March, 2002

A. Cash Flow from operating activities Rs. Rs. Profit after appropriation Increase in profit and loss A/c after inventory

adjustment [Rs.3,00,000 – (Rs.2,40,000 + Rs.24,000)]

36,000

Transfer to general reserve 1,20,000 Proposed dividend 1,44,000 Provision for tax 3,40,000 Net profit before taxation and extraordinary item 6,40,000 Adjustments for: Preliminary expenses written off 40,000 Depreciation 3,60,000 Loss on sale of fixed assets 20,000 Decrease in value of fixed assets 16,000

13.23

Premium on redemption of preference share capital 6,000 Premium on redemption of debentures 6,000 Operating profit before working capital changes 10,88,000 Increase in current liabilities

(Rs.5,20,000 –Rs.4,80,000)

40,000

Increase in other current assets [Rs.13,10,000 – (Rs.11,10,000 + Rs.24,000)]

(1,76,000)

Cash generated from operations 9,52,000 Income taxes paid (3,60,000) Net Cash from operating activities 5,92,000

B. Cash Flow from investing activities Purchase of fixed assets (8,56,000) Proceeds from sale of fixed assets 1,00,000 Proceeds from sale of investments 1,20,000 Net Cash from investing activities (6,36,000)

C. Cash Flow from financing activities Proceeds from issuance of share capital 4,00,000 Redemption of preference share capital

(Rs.1,20,000 + Rs.6,000) (1,26,000)

Redemption of debentures (Rs. 1,20,000 + Rs. 6,000) (1,26,000) Dividend paid (1,04,000) Net Cash from financing activities 44,000 Net increase/decrease in cash and cash equivalent

during the year

Nil Cash and cash equivalent at the beginning of the year 10,000 Cash and cash equivalent at the end of the year 10,000

Working Notes:

1. Revaluation of stock will increase opening stock by Rs. 24,000.

24,000 Rs. 10 90

2,16,000

Therefore, opening balance of other current assets would be as follows:

Rs. 11,10,000 + Rs. 24,000 = Rs. 11,34,000

Due to under valuation of stock, the opening balance of profit and loss account be increased by Rs. 24,000.

The opening balance of profit and loss account after revaluation of stock will be

Rs. 2,40,000 + Rs. 24,000 = Rs. 2,64,000

2. Investment Account

Rs. Rs. To To

Balance b/d Capital reserve A/c (Profit on sale of investment)

4,00,000

40,000

By

By

Bank A/c (balancing figure being investment sold) Balance c/d

1,20,000

3,20,000 4,40,000 4,40,000

13.24

3. Fixed Assets Account

Rs. Rs. Rs. To Balance b/d 32,00,000 By Bank A/c (sale of assets) 1,00,000 To Bank A/c

(balancing figure being assets purchased)

8,56,000 By

By

Accumulated depreciation A/c Profit and loss A/c(loss on sale of assets)

80,000

20,000

2,00,000 By Accumulated

depreciation A/c

40,000

By Profit and loss A/c (assets written off)

16,000

56,000

By Balance c/d 38,00,000 40,56,000 40,56,000

4. Accumulated Depreciation Account Rs. Rs.

To Fixed assets A/c 80,000 By Balance b/d 9,20,000 To Fixed assets A/c 40,000 By Profit and loss A/c To Balance c/d 11,60,000 (depreciation for the period) 3,60,000

12,80,000 12,80,000 5. Unpaid dividend is taken as non-current item and dividend paid is shown at Rs. 1,04,000 (Rs.1,20,000

– Rs.16,000).

Note: Alternatively, unpaid dividend can be assumed as current liability and hence, dividend paid can be shown at Rs. 1,20,000. Due to this assumption cash flow from operating activities would be affected. The cash flow from operating activities will increase by Rs. 16,000 to Rs. 6,08,000 and cash flow from financing activities will get reduced by Rs. 16,000 to Rs. 28,000.

Question 9

ABC Ltd. gives you the following informations. You are required to prepare Cash Flow Statement by using indirect methods as per AS 3 for the year ended 31.03.2004:

Balance Sheet as on Liabilities 31st March

2003 31st March

2004 Assets 31st March

2003 31st March

2004

Rs. Rs. Rs. Rs.

Capital 50,00,000 50,00,000 Plant & Machinery 27,30,000 40,70,000

Retained Earnings 26,50,000 36,90,000 Less: Depreciation 6,10,000 7,90,000

Debentures ― 9,00,000 21,20,000 32,80,000

Current Liabilities Current Assets

Creditors 8,80,000 8,20,000 Debtors 23,90,000 28,30,000

Bank Loan 1,50,000 3,00,000 Less: Provision 1,50,000 1,90,000

Liability for expenses 3,30,000 2,70,000 22,40,000 26,40,000

Dividend payable 1,50,000 3,00,000 Cash 15,20,000 18,20,000

Marketable securities

11,80,000 15,00,000

Inventories 20,10,000 19,20,000

Prepaid Expenses 90,000 1,20,000

13.25

91,60,000 1,12,80,000 91,60,000 1,12,80,000 Additional Information:

(i) Net profit for the year ended 31st March, 2004, after charging depreciation Rs. 1,80,000 is Rs. 22,40,000.

(ii) Debtors of Rs. 2,30,000 were determined to be worthless and were written off against the provisions for doubtful debts account during the year.

(ii) ABC Ltd. declared dividend of Rs. 12,00,000 for the year 2003-2004.

(16 marks) (PE-II–May 2004)

Answer

Cash flow Statement of ABC Ltd. for the year ended 31.3.2004

Cash flows from Operating activities Rs. Rs.

Net Profit 22,40,000

Add: Adjustment for Depreciation (Rs.7,90,000 – Rs.6,10,000)

1,80,000

Operating profit before working capital changes 24,20,000

Add: Decrease in Inventories (Rs.20,10,000 – Rs.19,20,000)

90,000

Increase in provision for doubtful debts (Rs. 4,20,000 – Rs.1,50,000)

2,70,000

27,80,000

Less: Increase in Current Assets:

Debtors (Rs. 30,60,000 – Rs.23,90,000) 6,70,000

Prepaid expenses (Rs. 1,20,000 – Rs.90,000) 30,000

Decrease in current liabilities:

Creditors (Rs. 8,80,000 – Rs. 8,20,000) 60,000

Expenses outstanding (Rs. 3,30,000 – Rs.2,70,000)

60,000

8,20,000

Net cash from operating activities 19,60,000

Cash flows from Investing activities

Purchase of Plant & Equipment (Rs. 40,70,000 – Rs.27,30,000)

13,40,000

Net cash used in investing activities (13,40,000)

Cash flows from Financing Activities

Bank loan raised (Rs. 3,00,000 – Rs. 1,50,000) 1,50,000

Issue of debentures 9,00,000

Payment of Dividend (Rs. 12,00,000 – Rs. 1,50,000) (10,50,000)

Net cash used in financing activities NIL

Net increase in cash during the year 6,20,000

Add: Cash and cash equivalents as on 1.4.2003

13.26

(Rs. 15,20,000 + Rs.11,80,000) 27,00,000

Cash and cash equivalents as on 31.3.2004 (Rs. 18,20,000 + Rs.15,00,000)

33,20,000

Note: Bad debts amounting Rs. 2,30,000 were written off against provision for doubtful debts account during the year. In the above solution, Bad debts have been added back in the balances of provision for doubtful debts and debtors as on 31.3.2004. Alternatively, the adjustment of writing off bad debts may be ignored and the solution can be given on the basis of figures of debtors and provision for doubtful debts as appearing in the balance sheet on 31.3.2004.

Question 10

From the following balance sheets of Sneha Ltd. as on 31.3.2003 and 31.3.2004 prepare a statement of sources and applications of fund and a schedule of changes in working capital for the year ending 31.3.2004:

Balance Sheets Liabilities 31.3.2003 31.3.2004 Assets 31.3.2003 31.3.2004

Rs. Rs. Rs. Rs.

Equity share capital 13,00,000 16,90,000 Goodwill 65,000 42,500

Profit and loss account 4,90,100 8,77,500 Building 11,70,000 11,37,500

10% Debentures 16,25,000 13,00,000 Machinery 16,18,500 21,38,500

Creditors 9,00,000 10,00,000 Non-trade investments 5,07,000 3,93,250

Bills payable 42,500 1,70,000 Debtors 4,16,000 11,70,000

Provision for tax 2,60,000 9,75,000 Stock 5,07,000 7,99,500

Dividend payable 42,250 Cash 2,60,000 2,92,500

Prepaid expenses 42,250 52,000

Debenture discount 31,850 29,000

46,17,600 60,54,750 46,17,600 60,54,750

The following additional information is given:

(i) Building Machinery Rs. Rs. Accumulated depreciation 31.3.2003 4,87,500 15,92,500 Accumulated depreciation 31.3.2004 5,20,000 15,66,500 Depreciation for 2003-2004 32,500 1,36,500

(ii) Profit and loss account for 2003-2004 is as follows: Rs. Balance as on 31.3.2003 4,90,100 Add: Profit for 2003-2004 4,71,900 9,62,000 Less: Dividend 84,500 8,77,500

(iii) During 2003-2004 machinery costing Rs. 2,92,500 was sold for Rs. 97,500.

(iv) Investments which were sold for Rs. 1,17,000 had cost Rs. 97,500.

(v) Provision for Taxation and Dividend are to be taken as Non-current liabilities.

13.27

(20 marks) (PE-II–Nov. 2004)

Answer (a) Sneha Ltd.

Fund Flow Statement for the year ended 31st March, 2004

Amount (Rs.) Sources of funds

Share capital

(Rs. 16,90,000 Rs. 13,00,000)

3,90,000

Sale of machinery 97,500 Sale of investments 1,17,000 Funds from operation (W.N. 1) 16,70,500 22,75,000 Applications of funds

Debentures redeemed

(Rs. 16,25,000 Rs. 13,00,000)

3,25,000

Machinery purchased (W.N. 4) 7,86,500

Tax paid 2,60,000

Dividend (Rs. 84,500 Rs. 42,250) 42,250

Increase in working capital 8,61,250 22,75,000

Schedule of Changes in Working Capital for the year ended 31st March, 2004

Balance as on Changes in working capital

1.4.2003 31.3.2004 Increase Decrease

Rs. Rs. Rs. Rs.

Current Assets:

Debtors 4,16,000 11,70,000 7,54,000

Stock 5,07,000 7,99,500 2,92,500

Cash 2,60,000 2,92,500 32,500

Prepaid expenses 42,250 52,000 9,750

A 12,25,250 23,14,000

Current Liabilities:

Creditors 9,00,000 10,00,000 1,00,000

Bills payable 42,500 1,70,000 1,27,500

B 9,42,500 11,70,000 10,88,750 2,27,500

Working capital (A – B) 2,82,750 11,44,000

Increase in working capital

________

8,61,250

10,88,750 10,88,750

The provision for taxation has been treated as a non-current liability as per the requirement of the question. Last year’s provision for taxation amounting Rs. 2,60,000 has been assumed to be paid in the current year ended 31st March, 2004.

13.28

Working Notes: 1. Statement showing funds generated from operations

(Rs.)

Increase in profit and loss account during the year (Rs. 8,77,500 – Rs. 4,90,100)

3,87,400

Add: Non-cash expenditures

(1) Loss on sale of machinery (W.N. 4) 32,500

(2) Investments written off (W.N. 2) 16,250

(3) Provision for tax 9,75,000

(4) Depreciation on building (Rs. 11,70,000 – Rs. 11,37,500) 32,500

on machinery (W.N. 3) 1,36,500 1,69,000

(5) Goodwill written off (Rs. 65,000 – Rs. 42,500) 22,500

(6) Debenture discount written off (Rs. 31,850 – Rs. 29,000) 2,850

(7) Dividend 84,500 13,02,600

16,90,000

Less: Non-cash incomes

(1) Profit on sale of investments (Rs. 1,17,000 – Rs. 97,500) 19,500

Funds from operations 16,70,500 2. Non Trade Investment Account Dr. Cr. Rs. Rs. To Balance b/d 5,07,000 By Bank -Sale 1,17,000 To Profit on sale

(Rs. 1,17,000 Rs. 97,500) 19,500

By

Profit and loss account – written off (balancing figure)

16,250

_______ By Balance c/d 3,93,250 5,26,500 5,26,500

13.29

3. Provision for Depreciation on Machinery Account Dr. Cr. Rs. Rs. To Machinery -sale (balancing

figure) 1,62,500 By

By Balance b/d Depreciation

15,92,500 1,36,500

To Balance c/d 15,66,500 17,29,000 17,29,000

4. Machinery Account Dr. Cr. Rs. Rs. To Balance b/d 16,18,500 By Bank (sale) 97,500 Add: Provision for

depreciation

15,92,500

32,11,000 By By

Depreciation Loss on sale

1,62,500 32,500

To Bank -purchase (balancing figure)

7,86,500

By Balance c/d W.D.V.

21,38,500

________ Add: Provision for depreciation

15,66,500

37,05,000

39,97,500 39,97,500

Question 11

The following figures have been extracted from the Books of X Limited for the year ended on 31.3.2004. You are required to prepare a cash flow statement.

(i) Net profit before taking into account income tax and income from law suits but after taking into account the following items was Rs. 20 lakhs:

(a) Depreciation on Fixed Assets Rs. 5 lakhs.

(b) Discount on issue of Debentures written off Rs. 30,000.

(c) Interest on Debentures paid Rs. 3,50,000.

(d) Book value of investments Rs. 3 lakhs (Sale of Investments for Rs. 3,20,000).

(e) Interest received on investments Rs. 60,000.

(f) Compensation received Rs. 90,000 by the company in a suit filed.

(ii) Income tax paid during the year Rs. 10,50,000.

(iii) 15,000, 10% preference shares of Rs. 100 each were redeemed on 31.3.2004 at a premium of 5%. Further the company issued 50,000 equity shares of Rs. 10 each at a premium of 20% on 2.4.2003. Dividend on preference shares were paid at the time of redemption.

13.30

(iv) Dividends paid for the year 2002-2003 Rs. 5 lakhs and interim dividend paid Rs. 3 lakhs for the year 2003-2004.

(v) Land was purchased on 2.4.2003 for Rs. 2,40,000 for which the company issued 20,000 equity shares of Rs. 10 each at a premium of 20% to the land owner as consideration.

(vi) Current assets and current liabilities in the beginning and at the end of the years were as detailed below:

As on 31.3.2003 As on 31.3.2004 Rs. Rs. Stock 12,00,000 13,18,000 Sundry Debtors 2,08,000 2,13,100 Cash in hand 1,96,300 35,300 Bills receivable 50,000 40,000 Bills payable 45,000 40,000 Sundry Creditors 1,66,000 1,71,300 Outstanding expenses 75,000 81,800

(20 marks) (PE-II – May 2005)

Answer

X Ltd.

Cash Flow Statement

for the year ended 31st March, 2004

Rs. Rs.

Cash flow from Operating Activities

Net profit before income tax and extraordinary items: 20,00,000

Adjustments for:

Depreciation on fixed assets 5,00,000

Discount on issue of debentures 30,000

Interest on debentures paid 3,50,000

Interest on investments received (60,000)

Profit on sale of investments (20,000) 8,00,000

Operating profit before working capital changes 28,00,000

Adjustments for:

Increase in stock (1,18,000)

Increase in sundry debtors (5,100)

Decrease in bills receivable 10,000

Decrease in bills payable (5,000)

Increase in sundry creditors 5,300

Increase in outstanding expenses 6,800 (1,06,000)

Cash generated from operations 26,94,000

Income tax paid (10,50,000)

16,44,000

Cash flow from extraordinary items:

Compensation received in a suit filed 90,000

Net cash flow from operating activities 17,34,000

13.31

Cash flow from Investing Activities

Sale proceeds of investments 3,20,000

Interest received on investments 60,000

Net cash flow from investing activities 3,80,000

Cash flow from Financing Activities

Proceeds by issue of equity shares at 20% premium 6,00,000

Redemption of preference shares at 5% premium (15,75,000)

Preference dividend paid (1,50,000)

Interest on debentures paid (3,50,000)

Dividend paid (5,00,000 + 3,00,000) (8,00,000)

Net cash used in financing activities (22,75,000)

Net decrease in cash and cash equivalents during the year (1,61,000)

Add: Cash and cash equivalents as on 31.3.2003 1,96,300

Cash and cash equivalents as on 31.3.2004 35,300 Note: Purchase of land in exchange of equity shares (issued at 20% premium) has not been considered in the cash flow statement as it does not involve any cash transaction.

Question 12

Raj Ltd. gives you the following information for the year ended 31st March, 2006:

(i) Sales for the year Rs.48,00,000. The Company sold goods for cash only.

(ii) Cost of goods sold was 75% of sales.

(iii) Closing inventory was higher than opening inventory by Rs.50,000.

(iv) Trade creditors on 31.3.2006 exceed the outstanding on 31.3.2005 by Rs.1,00,000.

13.32

(v) Tax paid during the year amounts to Rs.1,50,000.

(vi) Amounts paid to Trade creditors during the year Rs.35,50,000.

(vii) Administrative and Selling expenses paid Rs.3,60,000.

(viii) One new machinery was acquired in December, 2005 for Rs.6,00,000.

(ix) Dividend paid during the year Rs.1,20,000.

(x) Cash in hand and at Bank on 31.3.2006 Rs.70,000.

(xi) Cash in hand and at Bank on 1.4.2005 Rs.50,000.

Prepare Cash Flow Statement for the year ended 31.3.2006 as per the prescribed Accounting standard. (12 Marks) (PE-II – May 2006)

Answer

Cash flow statement of Raj Limited for the year ended 31.3.2006

Direct Method

Cash flow from operating activities:

Rs. Rs.

Cash receipt from customers (sales) 48,00,000

Cash paid to suppliers and expenses (Rs.35,50,000 + Rs.3,60,000)

39,10,000

Cash flow from operation 8,90,000

Less: Tax paid 1,50,000

Net cash from operating activities 7,40,000

Cash flow from investing activities:

Purchase of fixed assets (6,00,000)

Net cash used in investing activities (6,00,000)

Cash flow from financing activities:

Dividend Paid (1,20,000)

Net cash from financing activities (1,20,000)

20,000

Add: Opening balance of Cash in Hand and at Bank 50,000

Cash in Hand and at Bank on 31.3.2006 70,000

13.33

Question 13

The following are the summarized Balance Sheets of ‘X’ Ltd. as on March 31, 2005 and 2006:

Liabilities As on 31.3.2005 (Rs.)

As on 31.3.2006 (Rs,.)

Equity share capital 10,00,000 12,50,000

Capital Reserve --- 10,000

General Reserve 2,50,000 3,00,000

Profit and Loss A/c 1,50,000 1,80,000

Long-term loan from the Bank 5,00,000 4,00,000

Sundry Creditors 5,00,000 4,00,000

Provision for Taxation 50,000 60,000

Proposed Dividends 1,00,000 1,25,000

25,50,000 27,25,000

Assets Year 2005 (Rs.)

Year 2006 (Rs.)

Land and Building 5,00,000 4,80,000

Machinery 7,50,000 9,20,000

Investment 1,00,000 50,000

Stock 3,00,000 2,80,000

Sundry Debtors 4,00,000 4,20,000

Cash in Hand 2,00,000 1,65,000

Cash at Bank 3,00,000 4,10,000

25,50,000 27,25,000 Additional Information:

(i) Dividend of Rs.1,00,000 was paid during the year ended March 31, 2006.

(ii) Machinery during the year purchased for Rs.1,25,000.

(iii) Machinery of another company was purchased for a consideration of Rs.1,00,000 payable in equity shares.

(iv) Income-tax provided during the year Rs.55,000.

13.34

(v) Company sold some investment at a profit of Rs.10,000, which was credited to Capital reserve.

(vi) There was no sale of machinery during the year.

(vii) Depreciation written off on Land and Building Rs.20,000.

From the above particulars, prepare a cash flow statement for the year ended March, 2006 as per AS 3 (Indirect method). (16 Marks) (PE-II - Nov. 2006)

Answer

Cash Flow Statement for the year ending on March 31, 2006

Rs. Rs.

I. Cash flows from Operating Activities

Net profit made during the year (W.N.1) 2,60,000

Adjustment for depreciation on Machinery (W.N.2) 55,000

Adjustment for depreciation on Land & Building 20,000

Operating profit before change in Working Capital 3,35,000

Decrease in Stock 20,000

Increase in Sundry Debtors (20,000)

Decrease in Sundry Creditors (1,00,000)

Income-tax paid (45,000)

Net cash from operating activities 1,90,000

II. Cash flows from Investing Activities

Purchase on Machinery (1,25,000)

Sale of Investments 60,000 (65,000)

III. Cash flows from Financing Activities

Issue of equity shares (2,50,000-1,00,000) 1,50,000

Repayment of Long term loan (1,00,000)

Dividend paid (1,00,000) (50,000)

Net increase in cash and cash equivalent 75,000

Cash and cash equivalents at the beginning of the period 5,00,000

Cash and cash equivalents at the end of the period 5,75,000

13.35

Working Notes:

(i) Net Profit made during the year ended 31.3.2006

Increase in P & L (Cr.) Balance 30,000

Add: Transfer to general reserve 50,000

Add: Provision for taxation made during the year 55,000

Add: Provided for proposed dividend during the year 1,25,000

2,60,000 (ii) Machinery Account

Rs. Rs.

To Balance b/d 7,50,000 By Depreciation (Bal. Fig.)

55,000

To Bank 1,25,000 By Balance c/d 9,20,000

To Equity share capital 1,00,000

9,75,000 9,75,000 (iii) Provision for Taxation Account

Rs. Rs.

To Cash (Bal. Fig.) 45,000 By Balance b/d 50,000

To Balance c/d 60,000 By P & L A/c 55,000

1,05,000 1,05,000

(iv) Proposed Dividend Account

Rs. Rs.

To Bank 1,00,000 By Balance b/d 1,00,000

To Balance c/d 1,25,000 By P & L A/c (Bal. Fig.) 1,25,000

2,25,000 2,25,000

(v) Investment Account

Rs. Rs.

To Balance b/d 1,00,000 By Bank A/c 60,000

To Capital Reserve A/c (Profit on sale of investment)

10,000

(Balancing figure for investment sold)

By Balance c/d 50,000

1,10,000 1,10,000

PCC/IPCC_Accounts_theory_______________________________________1

MEC/CEC, CA/CWA & B.Com By Mattupalli Associates for Master Minds

THE WAY OF ASKING THEORY QUESTIONS IN EXAMINATION POINT OF VIEW FOR PCC /IPCC

FUNDAMENTALS OF ACCOUNTING:

1. What are Fundamental Accounting Assumptions? Write short notes on them. Ans: The Fundamental Accounting Assumptions are­

a. Going Concern: The enterprise is normally viewed as a going concern, that is as continuing in operation for the foreseeable future. It is assumed that the enterprise has neither the intention nor the necessity of liquidation or of curtailing materially the scale of operation.

b. Consistency: It is assumed that accounting policies are consistent from one period to another.

c. Accrual: Revenues and Costs are accrued, that is recognized as they are earned or incurred and recorded in the Financial Statements of the periods to which they relate.

AVERAGE DUE DATE:

1. What is ADD, areas where ADD is calculated? Ans: Definition: Average Due Date (ADD) is an equated date of payment on which a single payment can be made in lieu of several payments due for payment on different dates, without loss of interest to either party. Thus, ADD is the Arithmetic Average of various payments.

Areas where ADD method is followed: a. For calculation of ADD when various payments are due on different dates and single payment

is to be made by debtor. (Including the settlement of various bills due on different dates).

b. For calculating Interest on drawings made by partners on different dates.

c. For settlement of Contra Accounts. E.g. X & Y sells goods to each other on different dates.

d. For calculation of ADD when amount is lent by the creditor in one installment and repayment of the amount lent is to be made in various installments.

ACCOUNT CURRENT:

1. What is an Account Current? Explain Briefly. Ans: Account Current: Account current is a statement in the debit and credit form i.e., in the ledger form recording the transactions between the two parties in a chronological order or time sequence order. It is the copy of the accounts appearing in the books of sender with an additional column for interest. It is sent by one party to another usually by the agent to his principal or by the banker to his client.

An account current bears the following characteristics:­ a. It is an ordinary form of ledger account. b. The transactions are arranged in a sequential manner. c. There is an additional column of interest on each side of the account. d. It is the copy of accounts of one party in the books of another party. e. Any of the two parties can prepare this account. f. The interest columns are purely on the memorandum basis and are not a part of double entry.

2. What is meant by Red Ink Interest? Ans: Sometimes, the due date of the transactions fall beyond the date of settlement i.e., the date on which the account is prepared, in such cases, the days are counted from the settlement date to the date of transaction. These days are written with a negative sign in the days column or with a positive sign on the opposite side where the transaction does not appears. The

PCC/IPCC_Accounts_theory_______________________________________2

Ph: 0863 – 22 42 355 www.gntmasterminds.com

products were written with Red Ink. So the interest on such products is called as Red Ink Interest. This Red Ink Interest is treated as a negative interest.

3. What are various methods of accounting in an account current? Ans: There are three ways of preparing an Account Current: a. Interest Tables method b. The Method of Products c. The Method of product of balances.

a. Interest Tables method: i. Format: According to this method, all the transactions are arranged in the form of an

account. There are two additional columns on both the sides of such an account.

Ø One Column is meant to indicate the number of days counted from the due date of each transaction to the date of rendering the account. If no specific date is mentioned as the date on which payment is due, the date of the transactions is presumed to be the due date.

Ø The other column is meant for writing interest.

ii. Calculation of Interest: With the help of ready made tables (Simple Interest Tables), interest due on different amounts at given rates for different periods of time is found out and this is entered against each item separately. The interest columns of both the sides are totaled up and the balance is drawn.

b. The Method of Products: i. Format: The method of preparing the Account Current is the same as in the method of

interest tables, the method of calculating the interest alone remains the same but for minor modifications.

ii. Product Column: In this method the Product Column replaces the Interest Column of the Interest Method. The product is obtained by multiplying the amount of the transaction by the number of days from the Due Date to the date of the Account. The Product column is then balanced so as to ascertain the figure on which the interest (net) is to be calculated.

iii. Calculation of Interest: The interest is then calculated for a single day on the balance in the product column and is posted to the Account Current on the side opposite to the side where the “Product Balance” stands.

iv. Net Interest: This method calculates the net interest directly, i.e., interest payable and interest due are mutually set off and only the net interest due/receivable is reflected in the Account Current. Interest on the individual transactions is not reflected as in the Interest Method.

v. Procedure: Steps involved in this method is summarized as follows:

Ø Find out the balance of the products as in point (iii).

Ø Calculate Interest at the given rate on the balance of the products for a single day,

Ø Enter the interest on the appropriate side in the amount column. This entry is made on the side opposite to the side on which the balance of product appears.

c. The Method of product of balances: i. Meaning: This method is also known as periodic balance method and is usually adopted

in the case of banks where the balance of the account is taken out after every transaction.

ii. Format: The format of preparation of Account Current is as given below. Date Particulars Dr. Cr. Nature of

Balance (Dr./Cr.) Balance Amount Days Dr.

Product Cr.

Product

PCC/IPCC_Accounts_theory_______________________________________3

MEC/CEC, CA/CWA & B.Com By Mattupalli Associates for Master Minds iii. Purpose of Columns: The usage of the date and the particulars column are to denote the

date and the details of the transactions. The other columns are for the following purposes:

Ø The debits/credits to the account are entered in the Dr./Cr. Columns respectively. Ø The Nature of the balance i.e. whether the account has a Dr./Cr. balance is reflected

the “Nature of Balance” Column. Ø The Amount of balance is reflected in the “Balance Amount” Column. Ø In the “Days” Column the no. of days from the date of the transaction to that of the

next transactions in recorded. Ø “Dr. Product”/ “Cr. Product” consists of the product of the Dr. /Cr. column and the

Days column.

iv. Calculation of Interest: The Interest is calculated as under:

Ø The Columns are filled up to the date of the account as specified in point (c).

Ø The total of the Dr. Product and the Cr. Product Columns are arrived at.

Ø The interest for one day is calculated for the Dr./Cr. Products, at the appropriate rates and netted off to arrive at the net interest.

v. Posting of Interest: The interest amount is posted as follows: If the Dr. Product is greater than the Cr. Product the interest is posted to the Debit of the account, else it is posted to the Credit of the Account.

STATUTORY REPORT:

1. Write short note on Contents of Statutory report? Ans: Contents of Statutory Report [Sec. 165 (3)] a. The total number of shares allotted, distinguishing shares allotted as fully paid or partly

paid, otherwise than in cash and in case of partly paid up share stating the extent to which they are so paid up and in both cases the consideration for which they have been allotted.

b. The total amount of cash received in respect of shares allotted for cash.

c. An abstract of the Receipts and Payment A/c up to date within 7 days of the report, showing under the different headings the receipts of the company from shares, debentures and other sources, payments made, balance in hand and an account or estimate of the preliminary expenses of the company, showing separately any commission or discount paid, or to be paid on the issue or sale of shares or debentures.

d. The names, addresses and occupations of the director, auditor, manager and secretary and the change if any.

e. The particulars of any contract which, or the modification or the proposed modification of which, is to be submitted to the meeting for its approval together in the latter case with the particulars of the modification or the proposed modification.

f. The extent, if any, to which each underwriting contract has not been carried out and the reasons thereof.

g. The call­in­arrears from director and manager of the company.

h. The particulars of any commission or brokerage paid or payable in connection with the issue or sale of shares and debentures to any director or manager.

DEPARTMENTAL ACCOUNTS: 1. Write short notes on Basis of allocation of common expenditure among different

departments? Ans: Apportionment of Common Expenses:

No. Items of Expenses and Income Basis of Apportionment a. Salesmen’s commission, discounts allowed Sales (turnover) of each

PCC/IPCC_Accounts_theory_______________________________________4

Ph: 0863 – 22 42 355 www.gntmasterminds.com

(including provision for such discounts), bad debts, carriage outwards, advertisement, packing expenses etc.

department.

b. Discounts received (including provision for such discounts) etc.

Purchases of each department

c. Rent, rates and taxes, repairs and maintenance of building etc.

Floor space of each department

d. Depreciation of assets, fire insurance premium etc.

Asset values of each department

e. Workmen’s compensation insurance, employer’s contribution to Employees State Insurance etc.

Wages of each department

f. Canteen expenses, medical benefits, safety measures and such other labour welfare expenses etc.

No. of workers of each department.

Notes: a. Expenses incurred for the direct benefit of a particular department should be allocated

to the department concerned. E.g.: Special Advertisement, Insurance of Stock, Departmental Salaries.

b. If expenses incurred for the benefit of more than one department are not capable of accurate measurement, should be distributed on arbitrary basis (i.e., either in turnover ratio or in the cost of goods sold etc.) E.G., Salary paid to the General Manager, Expenses of Accounts dept. etc.

c. Expenses which cannot be apportioned satisfactorily should be left intact and finally to be debited to General P & L A/c.

FINANCIAL STATEMENT OF NOT FOR PROFIT ORGANISATION:

1. Explain the accounting treatment of donation received for specific purpose in the case of charitable society?

Ans: a. Donation may have been raised either for meeting some revenue or capital expenditure.

b. When expenditure intended for the revenue purpose are credited directly to the Income and Expenditure Account but others, if the donors have declared their specific intention, are credited to special fund account and in the absence thereof, to the Capital Fund Account.

c. When any investments are purchased out of a special fund or an asset is acquired there from, these are disclosed separately.

d. Any income received from such investments or any donations collected for a special purpose are credited to an account indicating the purpose and correspondingly the expenditure incurred in carrying out the purpose of the fund is debited to this account. In such case expenses are not charged to Income & Expenditure Account.

e. The term ‘Fund’ is strictly applicable to the amounts collected for a special purpose when these are invested, e.g. scholarship fund, recreation fund etc

f. Other wise, when the amounts collected are not invested in securities or assets distinguishable from those belonging to the institution, the word “Account” is more appropriate e.g. Building Account, Sports Goods Account etc.

UNDER WRITING: 1. What are the terms used in under writing? Ans: a. Marked Applications:

PCC/IPCC_Accounts_theory_______________________________________5

MEC/CEC, CA/CWA & B.Com By Mattupalli Associates for Master Minds i. The application forms bearing the stamp of the underwriter are termed as “Marked

Application forms.

ii. The benefit of marked applications is given to the concerned underwriters in whose favour application forms have been marked.

b. Unmarked Applications: i. The application forms which do not bear the stamp of the underwriter are termed as

‘Unmarked Application forms’.

ii. The benefit of unmarked applications is given first to the company to the extent of issue not underwritten by underwriters (in case any part of the issue is not underwritten).

iii. In case there is surplus, the benefit of surplus unmarked applications will be given to the underwriters in the ratio of their gross liability.

c. Full Underwriting: i. When the entire issue is underwritten such underwriting is termed as ‘full underwriting’.

For example, X Ltd. decided to make a public issue of 1,00,000 Equity Shares of Rs. 10 each which is entirely underwritten by A,B,C and D in the ratio of 2:2:1:1.

ii. In such a case the benefit of unmarked applications is given to the underwriter in the ratio of their gross liability i.e., 2:2:1:1.

d. Partial Underwriting: i. When only a part of issue is underwritten, such underwriting is termed as ‘Partial

Underwriting’. For example, X Ltd., decided to make a public issue of 1,00,000 Equity Shares of Rs. 10 each out of which 90,000 shares are underwritten by A, B, C and D in the ratio of 2:2:1:1. It means 10,000 shares are underwritten by the company itself.

ii. In such a case, the benefit of unmarked applications will first be given to the company.

iii. In case there is surplus, such surplus will be distributed among other underwriters in the ratio of their gross liability.

e. Sole Underwriting: i. When the issue is underwritten only by one underwriter, such underwriting is termed as

‘Sole Underwriting’. For example, if an issue of 1,00,000 shares of Rs. 10 each of X Ltd., is underwritten by A, it is a case of sole underwriting.

ii. In such a case, the distinction between marked and unmarked applications is not of such significance.

f. Joint Underwriting: i. When the issue is underwritten by two or more underwriters, such underwriting is termed

as ‘Joint Underwriting’. For Example, if an issue of 1,00,000 shares of Rs. 10 each of X Ltd., is underwritten by A, B, C and D in the ratio 2:2:1:1, it is a case of joint underwriting.

ii. In such a case the benefit of unmarked applications is given to the underwriters in the ratio of their gross liability.

iii. The benefit of marked applications is given to the concerned underwriters in whose favour applications have been marked.

g. Firm Underwriting: i. Meaning: Firm underwriting refers to a definite commitment by the underwriter to take

up a specified number of securities irrespective of the number of securities subscribed by the public.

For example, the entire issue of X Ltd., is underwritten as follows:

A. 1,60,000 shares (firm underwriting 3,600 shares) B. 1,60,000 shares (firm underwriting 2,000 shares)

PCC/IPCC_Accounts_theory_______________________________________6

Ph: 0863 – 22 42 355 www.gntmasterminds.com

C. 80,000 shares (firm underwriting 1,200 shares) D. 80,000 shares (firm underwriting 10,000 shares)

In this case only 4,63,200 shares (i.e., 4,80,000 shares – firm underwriting of 16,800 shares) will be offered to public and 16,800 shares will be taken up by the underwriters even if the issue is oversubscribed.

ii. Treatment: The benefit of firm underwriting may be given either.

Ø To an individual underwriter on the basis of his individual firm underwriting, or

Ø To all the underwriters in the ratio of their gross liability

In other words, firm underwriting shares may be treated at par with either ‘Marked Applications’ or ‘Unmarked Applications’.

2. Write a short note on firm underwriting and partial underwriting along with firm underwriting?

Ans: In firm underwriting the underwriter decides to subscribe upto a certain number of shares /debentures irrespective of the nature of public response to issue of securities. He gets these securities even if the issue is fully subscribed or over­subscribed. These securities are taken over by the underwriter in addition to his liability for securities not subscribed by the public.

Under partial underwriting along with firm underwriting, unless otherwise agreed, individual underwriters does not gets the advantage of firm underwriting in determination of number of shares/debentures to be taken up by him.

3. Pass the accounting entries relating to firm underwriting in the books of Company and Underwriter. Ans:

Entries in the books of AB Co.Ltd (Company) Particulars Dr.

(Rs.) Cr. Rs.)

a. A’s Account Dr. To Equity Share Capital Account.

(Being allotment of underwritten equity shares in pursuance of firm underwriting contract, vide Board’s resolution).

XXX XXX

b. Underwriting commission on issue of Shares Account Dr. To A’s Account

(Being underwriting commission due to the underwriter under the firm underwriting contract)

XXX XXX

c. Bank Account Dr. To A’s Account

(Being money received in full settlement of account from under writer)

XXX XXX

Entries in the books of A (Underwriter)

Particulars Dr. (Rs.)

Cr. (Rs.)

a. Underwriting Account Dr. To AB Co. Ltd Account

(Being the liability to take up necessary number of shares of the company in pursuance of firm under­ writing contract recorded)

XXX XXX

b. AB Co. Ltd Account Dr. To Underwriting Account

(Being underwriting commission income credited to underwriting A/c)

XXX XXX

PCC/IPCC_Accounts_theory_______________________________________7

MEC/CEC, CA/CWA & B.Com By Mattupalli Associates for Master Minds

c. AB Co. Ltd Account Dr. To Bank Account

(Being balance money paid to the co. in full settlement of account)

XXX XXX

SELF BALANCING LEDGERS (SBS):

1. What are the advantages of Self­balancing Ledger system? Ans: Advantages of Self­balancing ledger:­ a. A number of book keepers can work on different ledgers.

b. Arithmetic accuracy of each ledger can be proved independently.

c. Each ledger is of a suitable size.

d. A complete trial balance can be prepared without balancing subsidiary ledgers, thus facilitating the quick assessment.

e. Since error can be localized, delay in detection is minimized, there by saving labour and time of the book keepers.

LIQUIDATION OF COMPANIES: 1. Write short notes on Over­riding Preferential Creditors? Ans: Overriding Preferential payment (Section 529A): This section gives priority in payment to workmen’s dues and debts due to secured creditors to the extent they could not be paid because of the former ranking pari passu with the later.

Example: The following details have been extracted from the books of a company at the time of the liquidation:

Secured creditors (with assets charged in their favour Rs.2,00,000) Workmen’s dues Preferential creditors (excluding workmen’s dues) Unsecured creditors Other assets

3,00,000 1,00,000

50,000 2,00,000 2,50,000

The assets available will be used as follows: a. Assets charged in favour of secured creditors worth Rs.2,00,000 will be shared by Secured

Creditors and workers in the ratio of 3:1. i. Share of secured creditors 2,00,000 x ¾ = 1,50,000 ii. Share of workers 2,00,000 x ¼ = 50,000

b. Over­riding preferential payments amount to:

50,000 50,000

Secured creditors to the extent of their security being used for workmen’s dues: Balance of workmen’s dues (1,00,000 – 50,000)

1,00,000 c. Other Assets will be used as follows:

1,00,000 50,000

1,00,000

Over­riding preferential payments Preferential creditors Unsecured creditors

2,50,000

2. What are the contents of “Liquidators’ Statement of Account”? How frequently does a liquidator has to submit such statement?

Ans: It is a final statement A/c that is to be submitted by official Liquidator/Liquidator to the court/members creditors as the case may be in the event the company is finally being wound up.

PCC/IPCC_Accounts_theory_______________________________________8

Ph: 0863 – 22 42 355 www.gntmasterminds.com

PROFORMA Name of the company

Nature of Winding Up – voluntary/ compulsory Liquidator Final A/c

[In pursuance of sec. 497 & 509 of the companies Act] Receipts Rs. Payments Rs.

XXX

XXX XXX

XXX XXX XXX XXX XXX XXX XXX

XXX

To Realisation from sale of Assets [not specifically pledged]

To Realisation from Assets specifically pledged XXX Less: amount paid to

Secured creditors XXX To Receipts from contributory

(to the extent of uncalled capital)

XXX

By Legal Charges By Liquidator remuneration By Expenses of Liquidation By Amt. paid to Debenture holders By Preferential Creditors By Unsecured Creditors By Preference Share Holders

(at the rate of ­­­per share) By Equity Share Holders

(at the rate of ­­­ per share) XXX

Liquidator’s statement of account of the winding up is prepared for the period starting from the commencement of winding up to the close of winding up. If winding up process is not completed within one year after its commencement, Liquidator’s Statement of account pursuant to Section 551 of the Companies Act, 1956 (Form No.153) is to be filed by a Liquidator within a period of two months of the conclusion of one year and thereafter on interval of six months.

3. What is meant by B list of contributories”? What is the liability of contributories included in this list?

Ans: B list Contributories: The shareholders who transferred partly paid shares (otherwise than by operation of law or by death) within one year, before the date of winding up may be called upon to pay an amount (not exceeding the amount not called up when the shares were transferred) to pay off such creditors as existed on the date of transfer of shares.

a. When the existing assets available with the liquidator are not sufficient to cover the liabilities.

b. When the existing shareholders fail to pay the amount due on the shares to the liquidator.

AMALGAMATION OF COMPANIES: 1. What are the two main methods of accounting amalgamation of Companies? Ans: Method of accounting of amalgamation: a. Purchase method b. Pooling of interest method 2. Distinguish between Pooling of interest method and purchase method of recording

transactions relating to amalgamation. Ans:

Sl. No

Basis Pooling of interest method Purchase method of recording transaction

a. Applicability The pooling of interest method is applied in case of an amalgamation in the nature of merger.

Purchase method is applied in the case of an amalgamation in the nature of purchase.

b. Recording In the pooling of interest method all the reserves of the transferor Co. are also recorded by the transferee Co. in its books of account

In the purchase method the transferee Co. records in its books of accounts only the assets and liabilities taken over the reserves, except the statutory reserves of the transferor company are not aggregated with

PCC/IPCC_Accounts_theory_______________________________________9

MEC/CEC, CA/CWA & B.Com By Mattupalli Associates for Master Minds

those of the transferee Co.

c. Adjustment of the differences

Under the pooling of interest method, the difference between the consideration paid and the share capital of the transferor company is adjusted in the general reserve or other reserves of the transferee company.

Under the purchase method, the difference between the consideration and net assets taken over is treated by the transferee company as goodwill or capital reserve.

d. Statutory reserves

In this method, the statutory reserves are recorded by the transferee co. like all other reserves without opening Amalgamation and Adjustment A/c.

In the purchase method, while incorporating the statutory reserves, the transferee Co. has to open amalgamation adjustment account debiting it with the amt. of the statutory reserves being incorporated.

3. What are the conditions that are to be satisfied for ‘Amalgamation in the nature of Merger’ in an Amalgamation?

Ans: According to AS­14 on Accounting for Amalgamation, the following conditions must be satisfied for an amalgamation in the nature of merger: a. After amalgamation, all the assets and liabilities of the transferor company becomes the

assets and liabilities of the transferee company. b. Shareholders holding not less than 90% of the face value of the equity shares of the

transferor company becomes the equity shareholders of the transferee company by virtue of amalgamation.

c. The business of the transferor company is intended to be carried on after the amalgamation by the transferee company.

d. Purchase consideration should be discharged only by issue of equity shares in the transferee company except that cash may be paid in respect of any fractional shares.

e. No adjustments are required to be made in the book values of the assets and liabilities of the transferor company, when they are incorporated in the financial statements of the transferee company.

If any one of the condition is not satisfied in a process of amalgamation, it will not be considered as amalgamation in the nature of merger. 4. What are the methods for calculating purchase consideration? Ans: Different methods in computing the “Purchase Consideration”. a. Lumpsum Method: Under this method purchase consideration will be paid in lump sum as

per the valuation of purchasing companies valuationer. E.g., If it is stated that A Ltd. takes over the business of B Ltd. for Rs.15,00,000 here the sum of the Rs.15,00,000 is the Purchase Consideration.

b. Net Assets Method: Under this method P.C. shall be computed as follows: Particulars Rs.

XXX XXX

Agreed value of assets taken over Less: Agreed value of Liabilities taken over Purchase Consideration XXX

Note: i. The term “agreed value” means the amount at which the transferor company has

agreed to sell and the transferee company has agreed to take over a particular assets or a liability Otherwise book value will be the agreed value.

ii. Fictitious assets (i.e., preliminary expenses, underwriting commission, discount on issue of shares, discount on issue of debentures and debit balance in P & L A/c) are not taken over.

PCC/IPCC_Accounts_theory_______________________________________10

Ph: 0863 – 22 42 355 www.gntmasterminds.com

c. Payment Method: Under this method P.C. should be calculated by aggregating total payments made by the purchasing company. E.g.: A Ltd. had taken over B Ltd. and for that it agreed to pay Rs.5,00,000 in cash 4,00,000 Equity Shares of Rs.10 each fully paid at an agreed value of Rs.15 per share then the P.C. will be ascertained as follows:

Particulars Rs. 5,00,000

60,00,000

Cash 4,00,000 E. Shares of Rs.10 each fully paid, at Rs.15 per share Purchase Consideration 65,00,000

Note: A modified method of indicating consideration is to say how much a shareholder get per share on the transfer of the company’s business to transferee company.

5. Distinguish between Amalgamation in the nature of purchase and Amalgamation in the nature of merger?

Ans: Basis of

Distinction Amalgamation in Nature

of Merger Amalgamation in Nature of

Purchase a. T/f of

Assets & Liabilities

There is transfer of all assets & liabilities.

There need not be transfer for all assets & liabilities.

b. Equity Shareholder s holding 90%

Equity shareholders holding 90% equity shares in transferor company become shareholders of transferee company.

Equity shareholders, need not become shareholders of transferee company.

c. Purchase Considerati­ on

Purchase consideration is discharged wholly by issue of equity shares (except cash for fractional shares)

Purchase consideration need not be discharged wholly by issue of equity shares.

d. Same Business

The same business of the transferor company is intended to be carried on by the transferee company.

The business of the transferor company need not be intended to be carried on by the transferee company.

e. Recording of Assets & Liabilities

The assets & liabilities taken over are recorded at their existing carrying amounts except where adjustment is required to ensure uniformity of accounting policies.

The assets & liabilities taken over are recorded at their existing carrying amounts or the basis of their fair values.

f. Recording of Reserves ofTransferor Co.

All reserves are recorded at their existing carrying amounts and in the same form.

(a) Only statutory reserves are recorded at their existing carrying amounts as follows:

Amalgamation adjustment A/c Dr.

To Statutory Reserve A/c (b) Other reserves are not

recorded at all. g. Recording

of Balance of Profit & Loss A/c of Transferor

The balance of P&L A/c should be aggregated with the corresponding balance of the transferee co. or transferred to the General

The balance of P&L A/c losses its identify and is not recorded at all.

PCC/IPCC_Accounts_theory_______________________________________11

MEC/CEC, CA/CWA & B.Com By Mattupalli Associates for Master Minds

Co. Reserve. h. Difference

between the Purchase Considerati on and Share Capital/Net Assets of transfer co.

The excess of the purchase consideration over the share capital of transferor company is debited to Reserves and the excess of share capital over purchase consideration is credited to reserves.

The excess of purchase consideration over the net assets is treated as Goodwill and the excess of net assets over purchase consideration is treated as Capital Reserve.

HIRE PURCHASE:

1. Pass Accounting Entries for Repossessed goods under HP system of Debtors method.? Ans: The following entries will be made in respect of repossessed goods.

1. For goods repossessed on default of Purchaser. Goods repossessed A/c Dr.

To HP Debtor A/c (installment due) To HP Stock A/c (installment not due)

2. For difference between installment the unpaid and value of repossessed goods or loading amount only.

In Case of Loss:­ HP Adjustment A/c Dr.

To Goods Repossessed A/c In Case of Profit:­

The above entry will be reversed

3. For sale of goods repossessed Bank A/c Dr.

To Goods repossessed A/c

4. For profit on sale of goods repossessed. Goods repossessed A/c Dr.

To H.P. Adjustment A/c (in case of loss entry will be reversed)

ISSUE & REDEMPTION OF DEBENTURES:

1. State the guidelines of SEBI regarding issue of convertible debentures for disclosure and investor protection?

Ans: SEBI has prescribed the following guidelines for the issue of convertible debentures for disclosure and investor protections:

a. Issue of fully Convertible Debentures, having a conversion period of more than 36 months will not be permissible, unless the conversion is made optional with “put” and “call” option.

b. Premium amount on conversion, time of conversion, in stages, if any shall be determined in advance and must be stated in the prospectus. Interest rates for the above debentures will be fixed by the issue.

c. Any public or right issue of debt instruments shall have to be compulsorily rated by the approved credit rating agencies, irrespective of their maturity or conversion period.

d. Any conversion in part or whole of the debenture will be optional at the hands of the debenture­holders, if the alteration takes place at or after 18 months, from the date of allotment, but before 36 months.

e. Issue of debentures with maturity of 18 months or less are exempted from the necessity of appointment of Debenture Trustee or creating a Debenture Redemption Reserve.

PCC/IPCC_Accounts_theory_______________________________________12

Ph: 0863 – 22 42 355 www.gntmasterminds.com

f. Premium amount at the time of conversion for the Partly Convertible Debenture (PCD) should also be determined in advance and must be stated in the prospectus. Redemption amount, period of maturity, yield on redemption for the PCD’s or NCD’s must be shown in the prospectus.

g. The discount on the non­convertible portion of the PCD’s in case they are traded and procedure for their purchase on spot trading basis must be disclosed in the prospectus.

h. Before roll over of any NCDs or non­convertible portion of the PCD’s fresh credit rating shall be optioned within a period of six months to the due date of redemption, and communicated to debenture holder before roll over and fresh trust deed shall be made.

i. Letter of information regarding all over should be vetted by SEBI with regard to the credit rating, debenture holder resolution, option for conversion and such other item which SEBI may prescribe from time to time.

j. In case, the non­convertible portions of PCD’s or NCD’s are to be rolled over with or without change in the interest rate, a compulsory option should be given to those debenture holders who want to withdraw and in case from the debenture programme.

k. SEBI may prescribe additional disclosure requirement from time to time after due notice.

PARTNERSHIP ACCOUNTS: 1. Explain Garner Vs. Murray Rule applicable in case of partnership firms? Ans: a. Loss of Insolvent Partner: If upon dissolution, a Partner has become insolvent and the

debit Balance in his Capital Account is remaining unpaid, such loss should be borne by the Solvent Partners in the proportion of their Capital at the date of dissolution.

b. Capital: The Capital for this purpose is the balance in Partner’s Capital Account before adjusting therein the amount of profit or loss on the realization of assets.

c. Making Good the Realisation Loss: The loss on the realization should be contributed in cash by the solvent partners and the profit, will not be taken into account for determining the proportion in which the loss of insolvent partner should be borne by the solvent partners.

2. How to calculate Interest on Deceased partners loan as per Sec. 37 of Indian Partnership Act?

Ans: Disposal of the amount due to the Retiring on Deceased Partner: In the absence of an agreement in this regard, the outgoing partner at his option is entitled to receive either interest @ 6% p.a. till the amount is paid off or a share of the profit which has been earned by using the amount due to him. [Sec. 37] Which ever is lower.

Application on Section 37 of Indian Partnership Act, 1932 For e.g.: A, B & C were partners sharing profits and losses in the ratio of 2:2:1. C retired on 1 st

July, 2003 on which date the capitals of A, B and C after all necessary adjustments stood at Rs. 73,875, Rs. 63,875 and Rs. 42,250 respectively. A and B continued to carry on the business for six months without settling the A/c of C. During the period of six months from 01.07.2003, a profit of Rs. 20,500 is earned by the use of the firm’s property. State which of the two options available under section 37 of the Indian Partnership Act, 1932 should be exercised by C.

Solution: a. Share in the subsequent profits attributable to the use of his property;

= Rs. 42,250 x 2,500 = Rs. 4,812 Rs. 1,80,000

b. Interest @ 6% p.a. on the use of his property = Rs. 42,250 x 6 x 6 = Rs. 1,267.50 12 100

C should exercise option. Since the amount payable to him under this option is more as compared to the amount payable to him under option.

PCC/IPCC_Accounts_theory_______________________________________13

MEC/CEC, CA/CWA & B.Com By Mattupalli Associates for Master Minds

BANK ACCOUNTS:

1. Write shorts notes on Slip system of posting and double voucher system. Ans: Slip system of posting: This system is used in the case of banking companies, in this entries in the personal ledgers are made directly from vouchers, instead of being posted from day book. Pay­in­slips and cheques are used as slips. This become the basis of most of the transactions directly recorded in the account of the customer’s. Lots of time and labour of the bank staff is saved, because the slips are filled in by the customers themselves. The vouchers entered into different personal ledgers are summarized on summary sheets every day, total of which are posted to the different control accounts which are maintained in general ledger.

Double Voucher System: For the transaction not involving cash, two vouchers are prepared by the bank one debit voucher and the other credit voucher. This system of maintaining two vouchers are called double voucher system. The vouchers are sent to different clerks who make entries in book under their charge.

2. Write shorts notes on Acceptances and Endorsements? Ans: a. A bank has a more acceptable credit as compared to that of its customers. On the basis of

this, it is often called upon to, accept or endorse bills on behalf of its customers.

b. In such a case, the bank undertakes a liability towards the party which agrees to receive a bill in payment of a debt or agreed to discount the bill after the same has been accepted by the bank.

c. As against this liability, the bank has a correspondence claim against the customer on whose behalf it has undertaken to be a party to the bill, either as an acceptor or as an endorser.

d. The liabilities which are outstanding at the end of the year, and the corresponding assets are disclosed as contingent liability in the financial statement.

e. To be on the safer side, usually the bank asks the customers to deposit a security equivalent to the amount of the bill accepted on his behalf.

f. A record of the particulars of the bills accepted as well as of the securities collected from the customers is kept in the Bill Accepted Register.

g. A bank may not treat this book as part of system of its accounts.

h. In these cases, no further record of the transaction is kept until the bill matures for payment.

i. If the bill, at the end of its term, has to be retired by the bank, and the amount cannot be collected from the customer on demand. The bank reimburses itself by disposing of the security deposited by the customer.

3. Write a short notes on Assets Classification borrower­wise in Bank Accounts? Ans: The classification of advances as performing and non­performing and is borrower­wise and non­account wise. If one of the accounts of the borrower is non­performing (NPA), then other accounts which are otherwise performing, have to be classified as NPA only. The Reserve Bank of India has made clear that for purpose of classification as NPA, the most adverse category in any borrower’s account should be adopted as a prudential measure.

For example, if any borrower has three days of facilities, one of which is classified as standard, second sub­standard and the third is classified as doubtful., all the outstandings in the said borrowers account should be classified as doubtful assets under prudential norms.

4. What do you mean by Standard assets, Sub­standard assets and Doubtful assets of a bank?

Ans: a. Standard Assets: Standard Assets are those assets which does not show any problem and

also does not carry more than normal risk attached to the business. They are not non­ performing assets.

PCC/IPCC_Accounts_theory_______________________________________14

Ph: 0863 – 22 42 355 www.gntmasterminds.com

The provision of 0.25% is made on these standard assets as a matter of abundant caution, thought there could be no risk of non­recovery/default.

b. Sub­standard Assets: These are the assets which are classified as non­performing assets for a period not exceeding eighteen months. These assets have a well defined credit weaknesses that prejuicide the liquidation of the debt and are characterized by the distinct possibility that the bank will bear some loss, if deficiency are not corrected.

A provision of 10% of total outstanding should be made on such assets.

c. Doubtful Assets: The assets which remained a non­performing assets for a period exceeding 18 months are called doubtful assets. These advances are so weak that its collection in full is difficult. Depending on the age of the doubtful assets and the security, various percentage of provisioning is recommended for doubtful assets.

5. What is the % of NPA provision to be made by banks in respect of fully secured doubtful advances of more than 3 years old?

Ans: % of NPA provision for doubtful of advance of more than 3 years is 100%.

ELECTRICITY COMPANIES:

1. Write a short notes on Disposal of Surplus in Electricity Company? Ans: Disposal of Surplus: The law intends to prevent an electricity undertaking from earning unreasonable profits. That is why, it has defined Reasonable Return, Clear Profit, Capital Base. Excess of Clear Profit over Reasonable Return is called as ‘Surplus’ required to be credited to Customers Rebate Reserve and Surplus upto 20% of Reasonable Return is to be disposed off as follows:

a. 1/3 rd of such surplus not exceeding 5% of Reasonable Return at the disposal of the Company,

b. ½ of the balance to be credited to Tarrifs, Dividend Control Reserve.

c. ½ of the balance to be credited to Customers Rebate Reserves.

2. Write a short notes on Accounting treatment for replacement of an asset in Electricity Company?

Ans: Journal Entries:

1. To Record the Total Current Cash Cost of Old Asset: Asset A/c Dr. [with Current Cash Cost to be

Capitalised (as per Step 2)] Replacement A/c Dr. [Current Cost of old Asset (as per Step 1)]

To Bank A/c (Current Cash Cost of New Asset) 2. To Record the Value of Old Materials reused:

Asset A/c Dr. (with the given value of old materials reused) To Replacement A/c

3. To Record the sale proceeds of Old Materials sold: Bank A/c Dr. (with the given Sale Proceeds of

Old Materials Sold) To Replacement A/c

4. To Record the amount to be written off to Revenue: Revenue A/c Dr. [with the amount to be written off to

Revenue A/c (as per Step 3)] To Replacement A/c

Note: Hence, the total amount capitalized is calculated as follows: A. Current Cash Cost to be Capitalized xxx

B. Add: Value of Old Materials reused xxx

C. Total amount Capitalized (A + B) xxx

PCC/IPCC_Accounts_theory_______________________________________15

MEC/CEC, CA/CWA & B.Com By Mattupalli Associates for Master Minds 3. Write short notes on Reasonable returns in electricity supply companies.? Ans: Reasonable return in electricity supply companies: The law seeks to prevent an electricity undertaking from earning to high a profit. For this purpose, concept of Reasonable Return has been propounded. Reasonable Return is the normal return which Electricity Company can earn. The following is the procedure to compute the Reasonable Return and disposal of any surplus profits earned.

Computation of Reasonable Return: Particulars Rs.

Yield on Capital Base = Capital Base X Standard Rate of Return (Note) Add: Income on Investments other than Investment against Contingencies Reserve Add: ½ % of Loans advanced by the Electricity Boards Add: ½ % of amount borrowed from State Government approved

Organisation/Institutions Add: ½% of amount raised by the Issue of Debentures Add: ½ % on balance in Development Reserve

XXX

XXX XXX

XXX XXX XXX

Reasonable Return XXX Note: Standard Rate = Reserve Bank of India Rate + 2%. The term “Capital Base” used above can be identified as: a. The original cost of fixed assets available for use and necessary for the purpose of the

undertaking less contributions, if any made by the consumers for constructions of service lines and also amounts written off.

b. The original cost of work in progress. c. The cost of intangible assets. d. The amounts of investments compulsory made against contingencies reserve; and e. The monthly average of the stores, materials supplies and cash and bank balances held at

the end of each month of the year of account. 4. Main features of ‘double account’ system of presentation of financial information in

the case of public utility concern? Ans: Double accounts system is the name given to the system of preparing the final accounts of certain statutory companies formed by special Acts of Parliament, usually public utility undertakings (for example Electricity Companies). This method is not a special method of keeping accounts but it is a special method of presenting the accounts under the normal double entry system. In this system, separate accounts in respect of capital and revenue are prepared in order to show clearly the capital receipts and the manner in which the amounts thereof have been invested. The final accounts prepared under the double accounts system normally consists of: a. Revenue Account. b. Net Revenue Accounts. c. Capital Account (Receipts and Expenditure). d. General Balance Sheet. The Revenue account is certain to the Profit and Loss Account of a company with some exception. The Net Revenue Account resembles with appropriation portion of the Profit and Loss Account of the Company. The Capital Account shows the total amount of capital raised and its sources and also the manner and the extent to which this capital has been applied in the acquisition of fixed assets for the purpose of carrying on the business. Other items are included in the General Balance Sheet. The Double Account system in its pure form does no longer exist but the statements submitted to State Government by electricity companies generally follow the principle of double account system. For presenting accounts to the shareholders, electricity companies normally follow Schedule VI of the Companies Act, 1956.

PCC/IPCC_Accounts_theory_______________________________________16

Ph: 0863 – 22 42 355 www.gntmasterminds.com

5. Write short notes on “Receipt and payment on capital account” and “General balance sheet” of a public utility?

Ans: Receipt and payment on capital account and “General balance sheet” of a public Utility: Under the double accounts system, the balance sheet is split into two parts: a. Receipts and expenditure (payment) on capital account and b. General balance sheet. The main object of the former is to show the total amount of capital raised with its sources and the way and the extent to which this capital has been applied in the acquisition of fixed assets for the purpose of carrying on the business of the undertaking. It thus discloses the receipt and expenditure on capital account, that is the receipt from issue of shares, debentures and loans and the expenditure out of such receipts on acquisition of and addition of fixed assets. The receipt and expenditure on capital account is shown in a columnar form: There are three columns: a. One showing the amount at the commencement of the period. b. Another disclosing the amount received or spent during the period. c. The third showing the balance at the end of the period.

The general balance sheet contains other assets and liabilities and the balance of the receipt and expenditure on capital account. It is drawn up in the usual way, showing on the liabilities side, resources, depreciation fund, current liabilities and other credit balances and total of receipts as per capital accounts, on the assets side total of expenditure as per capital account, floating assets and other debit balances.

6. What is meant by Clear Profits? Ans: Clear Profit: It is the difference between the total income and total expenditure including specific appropriations. The following are the specific appropriations for computing clear profit.

a. Brought Forward Losses of Previous years to the extent permitted by State Government.

b. Taxes on Income and Profits

c. Amount written off in respect of Intangible Assets and New Issue Expenses (note)

d. Contribution to the Contingencies Reserve

e. Contribution towards arrears of Depreciation

f. Contribution to Development Reserve

g. Other Special Appropriations permitted by the State Government.

INSURANCE COMPANY ACCOUNT:

1. Computation of “premium income”, “claims expense” and “commission expense” in the case of an insurance company?

Ans: a. Premium: It is the consideration paid by insured to get his risk covered. The aggregate of

premium received together with premium receivable (outstanding) as reduced by premium on re­insurance ceded, if any, is shown in the credit side of revenue account. Thus the net figure of premium to be shown in revenue account is arrived in the following manner. Premium Received (direct) Rs. Add: Premium Received (Reinsurance accepted) Rs. Add: Premium outstanding Rs. ………………. Add: Bonus in Reduction of premium Rs. ………………. Less: Premium of Reinsurance ceded Rs. ……………….

Net Premium Rs. ……………….

PCC/IPCC_Accounts_theory_______________________________________17

MEC/CEC, CA/CWA & B.Com By Mattupalli Associates for Master Minds

The reserve for unexpired risk and additional reserve are created on the net premium only.

b. Claims: The insurance company covers various risks for a consideration called ‘Premium and in case the risk covered falls with the insured, the insurance company is liable to compensate. The amount paid as claim is written in the debit side of revenue account.

In addition to this, insurance companies, are required to incur expenditure as ‘legal expenses’ and ‘medical expenses’ in connection with claim. This expenditure since directly related to claim are basically incurred to reduce the liability of claim are also accounted for under the head ‘claims’. Thus following shall be treatment of various adjustments under the head ‘Claim’. Claim:

Particulars Rs. Rs. Claim paid during the year

Add: i Claim outstanding at, the closer of year, ii Medical Expenses re­claims,

iii Legal expenses re­claims, iv Survey expenses.

Less: i Claims outstanding at the beginning of the year ii Re insurance claim recoveries.

­ ­ ­ ­ ­ ­

­

­

c. Commission: When making final accounts of an Insurance Company is prepared terms with respect to commission are:

a. Commission on direct business.

b. Commission on reinsurance ceded and

c. Commission on reinsurance accepted.

a. The commission paid by an insurance company for carrying out work by its agents is called commission on Direct Business and shown in ‘Direct side of Revenue Account.

b. Where reinsurance premium is payable then commission is recoverable in such reinsurance business, the commission so paid by re­insurer is ‘Commission on Re­ insurance business ceded’ for the reinsured Company and is shown on ‘Credit’ side of revenues account.

c. There is ‘Commission on Reinsurance Accepted’ for the insurance company accepting the risk of other insurance company and is reflected in ‘Debit’ side of revenue account. The presentation is as follows:

Dr. Revenue Account Cr.

Debit Rs. Credit Rs. ­ ­

­ Commission on direct business Commission on re­insurance accepted

Commission on reinsurance ceded

2. Write short notes on Re­insurance? Ans: a. When an Insurance Company wants to part with some risk on an insurance policy, a part of

the risk may be insured with some other insurer, is called, ‘re­insurance ceded’ and original company is called ‘reinsured’. In this case reinsured company surrender proportionate premium to the other insurer and receive proportionate commission to re­insurance ceded.

b. In case of claim, original company pay to the policy holder and in turn receives proportionate claim from the other company. For the insurer with whom risk has been reinsured, this is called ‘re­insurance accepted.

c. There are two types of re­insurances. (a) Fluctuative Re­insurance and (b) Treaty Re­ insurance.

i. Fluctuative Re­insurance: In this type of re­insurance each risk is negotiated separately. Re­insurance of each risk is affected by ceding company and accepted by the

PCC/IPCC_Accounts_theory_______________________________________18

Ph: 0863 – 22 42 355 www.gntmasterminds.com

accepting company. Both companies have free choice for offer and acceptance. Since each transaction is negotiated separately, this involves large volume of work.

ii. Treaty Re­insurance: Under this type of re­insurance an agreement (Treaty) is entered between insurance companies to offer re­insurance and accept re­insurance compulsorily. The treaty may be for a specific type of risk or insurances of a specific geographical area. It is mandatory for both, ceding company to offer and insurer to accept within the limits of Treaty.

3. Write short notes on Reserve for Unexpired Risks in an insurance Company? Ans: Insurance policies are usually issued for a period of 1 year. However, at the year end, risks remain unexpired on most of the policies. Thus, total premium received cannot be taken as income of the current year. Since, risk is not reduced with passage of time, the premium relating to the next year is not calculated in proportion to unexpired period there is an unexpired liability under various policies. In order to cover this unexpired risk a reserve is created. This reserve is known as Unexpired Risk Reserve.

As per recommendation of the Executive Committee of the General insurance Council, the reserve for unexpired risk has to be created at following rates:

a. For Marine Business – 100% of Net Premium

b. For Other Business – 40% of Net Premium

However, as per Income Tax Rules, a provision of 50% of net premium may be created for other business. The journal entry for creating provision at the end of year is:

Revenue A/c …. Dr. To Reserve for Unexpired Risk A/c.

Reserve for unexpired risk account is shown in Balance Sheet (liabilities side) Next year, opening balance of Reserve for Unexpired Risk Account will be transferred to credit side of revenue account by making following entry:

Reserve for Unexpired Risk A/c ….Dr. To Revenue A/c.

4. What are the accounting entries pertaining to re­insurance business ceded to and by an insurance company? What are the corresponding commission entries?

Ans: When an insurance company wants to part with some risk on an insurance policy, a part of the risk be insured with some other insurer. This is called Re­insurance ceded. The original company is called Re­insured. In this case re­insured company surrenders proportionate premium to the other insurer and receives proportionate commission to re­insurance ceded.

In case of claim, original company: Pays to the policy holder and in turn recovers proportionate claim from the other company. For the answer with whom risk has been re­insured, this is called re­insurance accepted.

A Insurance Company cedes re­insurance business to ‘B’ Insurance Company. ‘C’ Insurance company further cedes re­insurance business to A Insurance company. Accounting entries pertaining to re­insurance business ceded to and by A Insurance Company as follows:

1. Re­insurance Premium (or re­insurance ceded ) A/c Dr. To B Insurance Company A/c

(Being premium on re­insurance business ceded to B insurance co. recorded 2. C Insurance Company A/c Dr.

To Reinsurance Premium (or reinsurance accepted) A/c (Being premium on business ceded by C insurance company recorded) 3. B Insurance company A/c Dr.

To claims (On re­insurance ceded) A/c (Being claim receivable from B company for part of insurance business ceded)

PCC/IPCC_Accounts_theory_______________________________________19

MEC/CEC, CA/CWA & B.Com By Mattupalli Associates for Master Minds

4. Claims (on re­insurance accepted) A/c Dr. To C Insurance Company A/c

(Being claims on re­insurance business accepted from Z company recorded) 5. B Insurance Company A/c Dr.

To Commission (on reinsurance ceded) A/c (Being commission due on re­insurance business ceded to B insurance company recorded) 6. Commission (on re­insurance accepted) A/c Dr.

To C Insurance Co. A/c (Being commission due on re­insurance business ceded to C company debited)

5. Explain the significance of ‘average clause’ in a Fire Insurance Policy? Ans: In the interest of the business, business units take a fire insurance policy to indemnify itself against the loss of stock and other assets resulting from the fire.

A fire insurance policy generally includes an average clause to discourage the under insurance of stock or any other asset.

The impact of this clause is that: a. If the value of stock or any other asset insured on the date of fire is more than the amount

of policy taken, the full value of stock or any other asset destroyed does not become payable to the business unit.

b. But, insurance company agrees to pay the proportion of the loss which the amount of policy taken bears to the total value of stock or any other assets on the date of the fire.

Following formula is used for computing average clause: = Value of stock destroyed x Value of Insurance Policy

Value of stock on the date of fire

Thus, Insurance Company accept the insurance claim in a proportionate reduction to the actual loss and stock or any asset on the date of such loss.

SHARE CAPITAL: 1. Describe the conditions, which have to be fulfilled by a Joint Stock Company to buy­

back its equity shares? Ans: According to Section 77A of the companies Act, 1956 a joint stock company has to fulfill the following conditions to buy­back its own equity shares:

a. There must be an authorization in the articles for the buy­back.

b. A special resolution must be passed in general meeting of the company for authorizing the buy­back and it must be completed within 12 months of passing S.R.

c. The buy­back should not exceed 25% of the total paid­up capital and free­reserves of the company in particular financial year.

d. All the shares of the company must be fully paid­up.

e. The ratio of the debt owned by the company is not more than twice the capital and its free reserves after such buy back.

f. The buy­back is made out of the free­reserves or out of the proceeds of the fresh issue.

g. The buy­back of the shares listed on any recognized stock exchange is in accordance with the regulation made by the SEBI on this behalf.

2. What are Sweat Equity Shares? What are the conditions, which must be fulfilled by a Joint Stock Company to issue these shares?

Ans: The Companies (Amendment) Act, 1999 has introduced with the help of Section 79A a new type of equity shares called “Sweat Equity Shares”. “Sweat equity shares are the equity shares issued by a company to its employees or director at a discount or for consideration other than

PCC/IPCC_Accounts_theory_______________________________________20

Ph: 0863 – 22 42 355 www.gntmasterminds.com

cash for providing know­how or making available right in the nature of intellectual property right or value addition by whatever name called”.

When the following conditions are fulfilled by the company, than only a company may issue sweat equity shares:

a. Sweat equity shares will be issued only when it is authorized and a special resolution is passed by the company in general meeting.

b. The resolution specifies the number of shares, current market price, the consideration if any and the class or classes of directors or employees to whom such equity shares are to be issued.

c. Not less than one year has, at the time of the issue, elapsed since the date on which the company was entitled to start the business.

d. The sweat equity shares of company, whose equity shares are listed on a stock exchange are issued in reference with the regulation made by the Securities and Exchange Board of India on this behalf.

e. In case of company whose equity shares are not listed on any stock exchange, the Sweat equity shares are issued on reference with the guidelines as may be prescribed.

3. Write short notes on Dividend on partly paid shares? Ans: a. As per the provisions given in the articles of the company, in case of partly paid­up shares,

the dividends are payable either on the nominal, called up or the paid up amount of shares.

b. In the absence of any such provision, Table A would be applied.

c. In this case the amount to be paid as dividend will be calculated on the amount paid­up on the shares and at the time of calculation, the date on which the amount were paid must be taken into account.

d. Calls paid in advance do not rank for payment of dividend.

e. But the interest may be paid on such calls, the rate of interest is 6% p.a as per Table A; different rates may be prescribed by the articles of the company.

f. According to Section 93, of the companies Act, 1956 a company may if so authorized by its article, pays a dividend in proportion to the amount paid on each share, where a larger amount is paid on some shares than on others.

g. But where the articles are silent and Table A has been excluded, the amount of dividend payable will have to be calculated on the nominal amount of shares.

h. According to Clause 88 to Table A dividends are to be declared and paid according to the amounts paid or credited as paid on the shares in respect where of the dividend is paid.

i. But in case nothing is paid upon any of the shares of the company, dividends may be declared and paid as per the nominal amount of the shares.

INTERNAL RECONSTRUCTION: 1. What is meant by Internal Reconstruction? Ans: In case of internal reconstruction, the company’s existing financial structure is reorganized without dissolving the existing company and without forming a new company. Taking a wider meaning of the term ‘Internal Reconstruction’. It includes:

a. Alteration of Share Capital under Section 94 to 97.

b. Reduction of Share Capital under Section 100 to 105.

c. Variation of Shareholders’ Rights under Section 106.

d. Scheme of Compromise/Arrangement under Sections 391 to 393 and 394A.

PCC/IPCC_Accounts_theory_______________________________________21

MEC/CEC, CA/CWA & B.Com By Mattupalli Associates for Master Minds

PREPARATION OF FINANCIAL STATEMENTS:

1. Which parties are Interested in Financial Statements? Ans: The users of financial statements include present and potential investors, employees, lenders, supplier and other trade creditors, customers, governments and their agencies and the public. They use financial statements in order to satisfy some of their information needs. These needs include the following:

a. Investors: The providers of risk capital are concerned with the risk inherent in, and return provided by, their investments. They need information to determine whether they should buy, hold or sell. They are also interested in information which enables them to assess the ability of the enterprise to pay dividends.

b. Employees: Employees and their representative groups are interested in information about the stability and profitability of their employers. They are also interested in information which enables them to assess the ability of the enterprise to provide remuneration, retirement benefits and employment opportunities.

c. Lenders: Lenders are interested in information which enables them to determine whether their loans, and the interest attaching to them, will be paid when due.

d. Suppliers and other trade Creditors: Suppliers and other creditors are interested in information which enables them to determine whether amounts owing to them will be paid when due. Trade creditors are likely to be interested in an enterprise over a shorter period than lenders unless they are dependent upon the continuance of the enterprise as a major customer.

e. Customers: Customers have an interest in information about the continuance of an enterprise, especially when they have a long­term involvement with, or are dependent on, the enterprise.

f. Governments and their agencies: Governments and their agencies are interested in the allocation of resources and therefore, the activities of enterprises. They also require information in order to regulate the activities of enterprises and determine taxation policies, and to serve as the basis for determination of national income and similar statistics.

g. Public: Enterprises affect members of the public in a variety of ways. For example, enterprises may make a substantial contribution to the local economy in many ways including the number of people they employ and their patronage of local suppliers. Financial statements may assist the public by providing information about the trends and recent developments in the prosperity of the enterprise and the range of its activities.

While all of the information needs to these users cannot be met by financial statements, there are needs which are common to all users. As providers of risk capital to the enterprise, investors need more comprehensive information than other users. The provision of financial statements that meet their needs will also meet most of the needs of other users that financial statements can satisfy.

2. Give the Four qualitative Characteristics which the financial statements should observe? Ans: a. Understandability: Financial Statements should be readily understandable by the users.

This means that all required information should be disclosed, clearly and properly.

b. Relevance: Financial Statements should contain only relevant information. Information, which is likely to influence the economic decisions of the users, is said to be relevant. Relevance of an item of information is related to its materiality.

c. Reliability: Financial Statements should be reliable, i.e. free from material error and bias. This means that the transactions and events in Financial Statements are­

i. Are faithfully represented.

ii. Reported in terms of its substance and economic reality, rather than merely their legal form.

iii. Reported in a neutral fashion, i.e. free from bias.

PCC/IPCC_Accounts_theory_______________________________________22

Ph: 0863 – 22 42 355 www.gntmasterminds.com

iv. Reported based on the principle of prudence.

d. Comparability: Financial Statements should be useful for inter­firm comparison (i.e. between different Firms in the same industry), and intra­firm comparison (i.e. within the Firm for different periods, branches/divisions, products, etc.). Comparability can be achieved only if the financial effect of any change in accounting policies is disclosed properly.

e. True and Fair View: Financial Statements are required to show an true and fair view of the performance, financial position and cash flows of the enterprise.

3. One of the Characteristics of Financial Statements is neutrality­ Do you agree with this statement?

Ans: a. Yes. One of the essential characteristics of Financial Statement is Neutrality.

b. It means the Financial Statements are free from Bias­impartial.

c. In order to make the Financial Statements more Reliable­Neutrality is more important. The Financial Statements prepared and reported should be from Material misstatements and bias.

d. Hence Neutrality is one of the important characteristics of Financial Statements.

4. Write the Procedure for Calculation of Managerial Remuneration (or) How to calculate Profits as per Sec. 349 for the sale of calculating Managerial Remuneration?

Ans: Calculation of Net Profits [Sec. 349]: Sec. 349 lays down the manner in which net profits for the purposes of calculation of managerial remuneration shall be calculated.

The various items which are required to be included or excluded or deducted or not to be deducted as per Sec. 349 are given below:

Items to be included in/excluded from Profits for Computing Manager’s Remuneration Items to be included in profits Items to be excluded from profits a. Bounties and subsidies received

from any Government or any public authority constituted or authorized in this behalf by any government.

a. Premium on shares or debentures. b. Profit on sale of forfeited shares. c. Profits of capital nature including

those from the sale of the undertaking of the company.

d. Profits of capital nature from the sale of any immovable property or fixed assets.

Items to be / Not to be deducted from Profit for Computing Manager’s Remuneration Item to be deducted from Profits Item not to be deducted from Profits a. All the usual working charges; b. Directors’ remuneration; c. Bonus of Commission paid or payable to

any person employed or engaged by the company;

d. Excess Profit Tax and Business Tax; e. Interest on Debentures, Mortgages,

Loans and Advances. f. Expenses of Repairs not of Capital

Nature g. Contribution to Charitable and other

funds, not directly related to the business of the company not exceeding Rs. 50,000 or 5% of the average net profit during the three financial years immediately preceding, whichever is greater. However, these limits can be exceeded with the consent of the company in general meeting;

a. Income­tax and super tax paid by the company or any other tax on the income of the company not falling under Clause (d) above.

b. Any Compensation, Damages or Payments made voluntarily and not paid due to any legal liability.

c. Capital Losses.

PCC/IPCC_Accounts_theory_______________________________________23

MEC/CEC, CA/CWA & B.Com By Mattupalli Associates for Master Minds h. Depreciation to the extent specified in

Sec. 350; i. Past losses arising after 1 st April, 1956 to

the extent not already deducted in any subsequent year;

j. Any compensation or damages to be paid due to any legal liability including the one arising from a breach of contract;

k. Any sum paid by way of insurance to cover any liability discussed in above clause.

l. Bad debts written off or adjusted during the accounting year.

5. Write a short notes on Transfer of Profit to Reserve Rules? Ans: a. Yes, a Company can transfer more than 10% of its profits to reserves. In such an event a

minimum dividend has to be declared as stated below:

b. Minimum dividend rate should be equal to average rate of dividend declared in preceding three years.

c. Where bonus shares have been issued in the preceding three years, the distribution of dividend, a minimum distribution of dividend equal to the average amount (quantum and not the rate) of the dividend declared for the three years is ensured.

d. However, the minimum distribution in the above two cases need not be ensured if net profits after tax have fallen by at least 20% of the net profits of the average net profits after tax of the two preceding financial years.

e. Where no dividend is declared the amount proposed to be transferred to its reserves from the current profits must be less than the average amount of dividends declared to the shareholders over the three preceding financial years.

ACCOUNTS IN COMPUTERISED ENVIRONMENT:

1. What are the advantages and disadvantages of outsourcing the accounting functions? Ans: a. Advantages are:

i. Organisation can concentrate on their core area Less number of human resource is required.

ii. Organisation is able to utilized the expertise knowledge and experience of the outsourcing agency.

iii. It is economical for the organisation.

iv. Labour turnover does not effects the functioning of accounts department.

v. Accounts are maintain and stored in the hand of professionals.

b. Disadvantages are: i. Outsourcing agency is unable to meet the standard desirable.

ii. Various hidden costs are involved, which was not initially envisaged.

iii. There is a fear of confidentiality loss and security of accounts.

iv. Delay in obtaining services from third party.

v. Organisation Looses their control from various financial activities.

PCC/IPCC_Accounts_theory_______________________________________24

Ph: 0863 – 22 42 355 www.gntmasterminds.com

2. Explain the factors to be considered before selecting the pre­packaged accounting software?

Ans: It is very difficult task for an organisation is choose appropriate accounting software from the bundle of software available in the market. Some basic criteria should be considered while selecting the software.

a. Fulfillment of business needs: Buyers try to match his own requirement with the available softwares.

b. Easy to use:­ Such software which is easily operative should be selected.

c. Provides maximum reports: Some software packages are available in the market which might provide extra reports or such report as they wants.

d. Goodwill of the Vendor: A stable vendor with good past records will always be preferred because their continuous support is essential for any software.

e. Cost comparison: First analyse various software the select most economic software.

f. Regular update: Vendor normally provides regular updates to take care of the changes of law as well as add new feature to the existing software. So, select the vendor whose past record in this context is good.

3. What are the advantages and disadvantages of customized accounting packages? Ans: a. Advantages are:

i. Such softwares suitable match with the organizational structures.

ii. It covers all functional areas of the organizations.

iii. The input screens can be tailor made so that is matches with the input documents for data entry.

iv. Reports are available as per the specification of the organisation.

v. Various tools such as Barcode can used as input devices.

b. Disadvantages are: i. Development of such software is a time taking process. ii. Of input specifications are incomplete or improper resulting in a defecting or

inappropriate system. iii. Lack of documentation. iv. Inadequate control measures. v. Regular undates are not possible. vi. It is costlier than pre­packaged accounting. vii.Less reliability. viii. Gestation period is very high.

4. What are the advantages and disadvantages of an ERP package ? Ans: a. Advantages are:

i. Large volume of information are available through such package.

ii. It is an integrated package so it reduces the possibility of duplication of data entry.

iii. It is a generalised package which covers most of the common functionalities of any specific module.

iv. Reports or ERP are standardized across industry and are generally acceptable to the user.

PCC/IPCC_Accounts_theory_______________________________________25

MEC/CEC, CA/CWA & B.Com By Mattupalli Associates for Master Minds v. Various reports are available in such package. These reports are standardized and

generally acceptable to the user.

b. Disadvantages are: i. It is less flexible: In which user may have to modify their business procedure at times

to be able to effectively use the ERP.

ii. Implementation is very difficult: Many of the consultants doing he implementation of the ERP may not be able to fully appreciate the business procedure to be able to do a good implementation of an ERP.

iii. It is very expensive: ERP are normally priced at an amount which is often beyond the reach of small and medium sized organisation. However, there are some ERP coming into the market which are moderately priced and may be useful to the small businesses.

iv. It is a very complex software: Generally an ERP package has large number of options to choose from. Further the parameter settings and configuration market it a little complex for the common users.

The End